Download as pdf or txt
Download as pdf or txt
You are on page 1of 120

JEE Companion

For JEE MAINS and ADVANCED

PHYSICS
MODULE-1
CONTENTS

CHAPTER – 01 : Vector & Basic Mathematics


THEORY 01-15
EXERCISE 16-33
ANSWER KEY 34

CHAPTER – 02-Unit & Dimension


THEORY 35-47
EXERCISE 48-70
ANSWER KEY 71

CHAPTER – : 03-Motion in One Dimension


THEORY 72-88
EXERCISE 89-116
ANSWER KEY 117-118
VECTOR AND BASIC MATH EMATICS PHYSICS-X I-IIT-JEE

CHAPTER

VECTOR AND BASIC MATHEMATICS


1. VECTOR 4. Collinear vectors:When the vectors under
Introduction of Vector: consideration can share the same support or have a
Physical quantities having magnitude, direction and common support then the considered vectors are
obeying laws of vector algebra are called vectors. collinear.

Example: Displacement, velocity, acceleration, 5. Zero vector (0) : A vector having zero magnitude
momentum, force, impulse, weight, thrust, torque, and arbitrary direction (not known to us) is a zero
angular momentum, angular velocity etc.
vector.
If a physical quantity has magnitude and direction both, 6. Unit vector:A vector divided by its magnitude is a
then it does not always imply that it is a vector. For it to 
unit vector. Unit vector for A is  (read as A cap or
be a vector the third condition of obeying laws of vector
A hat).
algebra has to be satisfied.  
A ˆ
Since, Â   A  AA
Example: The physical quantity current has both A
magnitude and direction but is still a scalar as it Thus, we can say that unit vector gives us the
disobeys the laws of vector algebra. direction.
Types of Vector:
  7. Orthogonal unit vectors ˆi , ˆj and k̂ are called
1. Equal vectors:Two vectors A and B are said to be orthogonal unit vectors. These vectors must form a
equal when they have equal magnitudes and same Right Handed Triad (It is a coordinate system such
direction. that when we Curl the fingers of right hand from
 xto y then we must get the direction of z along
A thumb). The
 y
B
  ˆj
2. Parallel vector: Two vectors A and B are said to
be parallel when x
(i) Both have same direction. k̂ î
(ii) One vector is scalar (positive) non-zero z
multiple of another vector.   
  x ˆ y z
3. Anti-parallel vectors:Two vectors A and B are î  , j  , k̂ 
x y z
said to be anti-parallel when   
(i) Both have opposite direction.  x  xiˆ , y  yjˆ , z  zkˆ
(ii) One vector is scalar non-zero negative multiple 8. Polar vectors: These have starting point or point of
of another vector. application .Example displacement and force etc.
9. Axial Vectors:These represent rotational effects
 and are always along the axis of rotation in
A
accordance with right hand screw rule. Angular
 velocity, torque and angular momentum, etc., are
B
example of physical quantities of this type.

1
PHYSICS-X I-IIT-JEE VECTOR AND BASIC MATHEMATICS
Axial vector (iii) Direction of resultant vectors: If  is angle
 
between A and B, then
 
| A  B |  A 2  B2  2ABcos 
Anticlockwise rotation  
Axis of rotation If R makes an angle  with A, then in OBN,
BN BN
Axis of rotation tan   
ON OA  AN
Bsin 
tan  
A  B cos 
Clock wise rotation
Example-1
Axial vector   
Given: R  A  B and R = A = B. The angle between
 
10. Coplanar vector: Three (or more) vectors A and B is -
arecalled coplanar vector if they lie in the same (a) 60º (b) 90º (c) 120º (d) 180º
plane. Two (free) vectors are always coplanar. Solution:[c]
R2 = A2 + B2 + 2AB cos ........(i)
 Addition of vectors:
R=A=B
(i) Triangle Law of Vector Addition of Two Vectors
A2 = A2 + A2 + 2A2cos
If two non zero vectors are represented by the two
sides of a triangle taken in same order then the 1
cos = 
resultant is given by the closing side of triangle in 2
  
opposite order. i.e. R  A  B  = 120º
B
Example-2
R  AB    
If the angle between A and B is 270º, then A .B is-
B 1
O A (a) 0 (b) 1 (c) –1 (d)
A 3
   Solution:[a]
 OB  OA  AB  
A .B = AB cos 270º = 0

(ii) Magnitude of resultant vector Example-3


AN     
In  ABN, cos    AN  B cos  Given: C  A  B . Also, the magnitudes of A , B and
B 
BN C are 12, 5 and 13 units respectively. The angle
sin    BN  Bsin   
B between A and B is -
In OBN, we have OB2  ON 2  BN 2  
(a) 0º (b) (c) (d)
B 4 2
Solution:[c]
R
B B sin 

R 

O A B
A N
  90º
B cos 
A
 R 2  (A  Bcos )2  (Bsin ) 2 (iv) Parallelogram Law of Vector Addition
 R 2  A 2  B2 cos2   2ABcos   B2 sin 2  If two non zero vectors are represented by the two
adjacent sides of a parallelogram then the resultant
 R 2  A 2  B2 (cos2   sin 2 )  2ABcos 
is given by the diagonal of the parallelogram
 R 2  A 2  B2  2ABcos  passing through the point of intersection of the two
 R  A2  B2  2ABcos  vectors.

2
VECTOR AND BASIC MATH EMATICS PHYSICS-X I-IIT-JEE
(1) Magnitude
D D C
Since, R 2  ON 2  CN 2 
 R 2  (OA  AN) 2  CN2 E C

 R 2  A 2  B2  2ABcos  E B
  
 R  | R |  | A  B |  A 2  B2  2ABcos 
R B
B C

O A
R  AB A
B B sin
B Note 

   Resultant of two unequal vectors cannot be zero.
O A N  Resultant of three co-planar vectors may or may not
A be zero
B cos  Resultant of three non co- planar vectors cannot be
Specialcases: R  A  B when  = 0o zero.
R = A – Bwhen  = 180o Subtraction of vectors
   
2 2 o
Since, A  B  A  (B) and
R  A  B when = 90  
(2) Direction | A  B |  A 2  B2  2ABcos 
CN Bsin   
tan     | A  B |  A 2  B2  2ABcos (180o  )
ON A  Bcos 
Since, cos (180  )   cos 
Example-4  
 | A  B |  A 2  B2  2ABcos 
Two forces of magnitude F and 3 F act at right angles
R sum  A  B
to each other. Their resultant makes an angle  with F.
The value of  is – B
(a) 30º (b) 45º (c) 60º (d)135º
1
Solution:[c] 
2
A
3F 180 – 
Resultant  B

Rdiff  A  (B)

F Bsin 
3 Fsin 90º tan 1 
tan = A  Bcos 
F  3 Fcos 90º B sin (180  )
and tan  2 
3F A  B cos (180  )
= = 3
F But sin(180  )  sin  and cos(180  )   cos 
 = 60° Bsin 
 tan  2 
A  Bcos 
(v) Polygon Law of Vector Addition
Example-5
If a number of non zero vectors are represented by
the (n – 1) sides of an n-sided polygon then the A particle moving with velocity v towards northward
resultant is given by the closing side or the nth side direction changes its direction and moves towards
of the polygon taken in opposite order. So, eastward with the same speed. Find the change in its
      velocity.
R  A BCD E
      (a) v 2 (b) v 3 (c) 2v (d) v
OA  AB  BC  CD  DE  OE

3
PHYSICS-X I-IIT-JEE VECTOR AND BASIC MATHEMATICS

Solution:[a] (1) The magnitude of the vector R is obtained by
 
v1 = (ON) where |v1| = v squaring and adding equation (ii) and (iii), i.e.
 
v2 = (NE) where |v2 = v R  R 2x  R 2y
Change in velocity 
    (2) The direction of the vector R is obtained by
 v = v2  v1 + (– v1 )
   dividing equation (iii) by (ii), i.e.
= (NE) + (ON) = (NA)
 tan   (R y / R x ) or   tan 1 (R y / R x )
and | v | = v12  v22  2v1v2 cos 90º

= v2  v2 = v 2 Example-6

|v | v A displacement vector, at an angle of 30º with y-axis
tan =  1 = = 1 or  = 45º
| v2 | v has an x-component of 10 units. Then the magnitude of
N
the vector is -
W E (a) 5.0 (b) 10 (c) 11.5 (d) 20
v2 E v2
N
S Solution:[d]
y
-v1
v1 
R

O
fig (a) fig (b) 30º
 Resolution of Vector into Components:
 60º
Consider a vector R in X-Y plane as shown in fig. x
 
If we draw orthogonal vectors R x and R y along x  
Let R be the given vector. X-component of the vector R
and y axes respectively, by law of vector addition,
   
R  Rx  R y = | R | cos 60º = 10
Y  10
|R |= = 20
cos 60º

Ry R
 Rectangular Components of 3-D Vector
    

X
R  R x  R y  R z q or R  R x ˆi  R y ˆj  R z kˆ
Rx
  Y
ˆ
Now as for any vector A  Anˆ so, R x  iR x and

R y  ˆjR y

so R  ˆiR x  ˆjR y …(i) Ry
R
Rx

But from figure R x  R cos  … (ii)


X
and R y  R sin  …(iii) Rz

Z
Since R and  are usually known, Equation (ii) and

(iii) give the magnitude of the components of R 
If R makes an angle  with x axis,  with y axis
along x and y-axes respectively.
and  with z-axis, then
Here it is worthy to note once a vector is resolved Rx Rx
 cos    l
into its components, the components themselves R R 2x  R 2y  R 2z
can be used to specify the vector as

4
VECTOR AND BASIC MATH EMATICS PHYSICS-X I-IIT-JEE
   
Ry Ry ii. It is commutative, i.e. A.B  B.A
 cos    m
R       
R 2x  R 2y  R 2z iii. It is distributive, i.e. A.(B  C)  A.B  A.C
 
 cos   R z  Rz
n iv. As by definition A.B  ABcos 
R R 2x  R 2y  R z2  
1  A.B 
The angle between the vectors   cos  
Where l, m, n are called Direction Cosines of the  AB 

vector R and v. Scalar product of two vectors will be maximum
l 2  m2  n 2  when cos   max  1, i.e.   0o , i.e., vectors are
 
R 2x  R 2y  R 2z parallel (A.B)max  AB
cos 2   cos 2   cos 2   1
R 2x  R 2y  R 2z vi. Scalar product of two vectors will be minimum
o  
when | cos  |  min  0, i.e.   90 (A.B)min  0
Note: i.e.if the scalar product of two nonzero
vectorsvanishes the vectors are orthogonal.
 When a point P have coordinate (x, y, z) then its
 vii. The scalar product of a vector by itself is termed as
position vector OP  xiˆ  yjˆ  zkˆ self dot product and is given by
  
 When a particle moves from point (x1, y1, z1) to (A) 2  A.A  AA cos   A 2
(x2, y2, z2) then its displacement  
i.e. A  A.A

vector r  (x 2  x1 ) iˆ  (y 2  y1 )ˆj  (z 2  z1 )kˆ viii. In case of unit vector n̂
 ˆ ˆ  1 1 cos0  1
n.n
 Scalar Product of Two Vectors ˆ ˆ  ˆi.iˆ  ˆj. ˆj  k.k
So n.n ˆ ˆ 1
 Definition: The scalar product (or dot product) of ix. In case of orthogonal unit vectors ˆi, ˆj and k̂,
two vectors is defined as the product of the ˆi. ˆj  ˆj.kˆ  k.i
ˆ ˆ  11cos90  0
magnitude of two vectors with cosine of angle
x. In terms of components
between them.
  A . B  (iA x  jA y  kA z ) . (iB x  jB y  kB z )
Thus if there are two vectors A and B having
 [Ax Bx  AyBy  AZBz ]
angle  between them, then their scalar product
   
written as A.B is defined as A.B  ABcos   Example:
i. Work W: In physics for constant force work is
defined as, W  Fs cos  ... (i)
 Properties: But by definition of scalar product of two vectors,

F.s  Fs cos  ...(ii)
B

So from eqn (i) and (ii) W  F.s i.e. work is the
 scalar product of force with displacement.

ii. Power P:
A
 
dW  ds 
As W  F.s or  F. [As F is constant]
i. It is always a scalar which is positive if angle dt dt
between the vectors is acute (i.e., < 90°) and  
or P  F.v i.e., power is the scalar product of force
negative if angle between them is obtuse (i.e.

90°<< 180°).  dW ds  
with velocity.  As  P and  v 
 dt dt 

5
PHYSICS-X I-IIT-JEE VECTOR AND BASIC MATHEMATICS

iii. Magnetic Flux  : Example-9



ds  The vector 5iˆ  2jˆ  kˆ is perpendicular to the vector
B
3iˆ  ˆj  2kˆ for  =
(a) 1 (b) 4.7

(c) 6.3 (d) 8.5
O Solution:[d]
 
A . B = 15 + 2 – 2 = 0
Magnetic flux through an area is given by 2 = 17
d  B ds cos  .......(i) 17
 = = 8.5
But by definition of scalar product 2
 
B.ds  Bdscos  .......(ii)
n
So from eq (i) and (ii) we have Example-10
   
d  B.d s or    B.ds Angle between two vectors (iˆ  ˆj) and (iˆ  ˆj) is-
(a) 30º (b) 60º
iv. Potential energy of a dipole U: If an electric dipole (c) 45º (d)90º
 
of moment p is situated in an electric field E or a Solution:[d]
  
magnetic dipole of moment M in a field of A.B  (iˆ  ˆj).(iˆ  ˆj) = 0
  
induction B, the potential energy of the dipole is So A  B
given by:
     Vector Product of Two Vectors
UE  p.E and UB  M.B  Definition:
Example-7 The vector product or cross product of two vectors
 
If A is parallel to B , then is defined as a vector having a magnitude equal to
      the product of the magnitudes of two vectors with
(a) A.(A  B)  A 2  AB (b) A  (A  B)  AB
     the sine of angle between them, and direction
(c) A.B  0 (d) A  (A  B)  9 perpendicular to the plane containing the two
Solution:[a] vectors in accordance with right hand screw rule.
     
A . (A  B)
  C  A B
 
= A2 + A . B Thus, if A and B are two vectors, then their
= A2 + AB cos 0º   
= A2 + AB vector product written as A  B is a vector C
defined by
  
Example-8 C  A  B  ABsin  nˆ
A force of 4iˆ  5jˆ Newton displaces a body through

3iˆ  6kˆ metre. The work done is–


(a)12 J (b) 30 J
(c) 42 J (d) 0 J
Solution:[c]
 
W = F – S = 12 + 30
= 42 J

6
VECTOR AND BASIC MATH EMATICS PHYSICS-X I-IIT-JEE
   
The direction of A  B, i.e. C is perpendicular to vii. In case of unit vector nˆ  nˆ  0 so that
  
the plane containing vectors A and B and in the ˆi  ˆi  ˆj  ˆj  kˆ  kˆ  0
sense of advance of a right handed screw rotated
  ˆ ˆj,kˆ in
viii. In case of orthogonal unit vectors, i,
from A (first vector) to B (second vector) through
the smaller angle between them. Thus, if a right accordance with right hand screw rule:
handed screw whose axis is perpendicular to the
  
plane framed by A and B is rotated from A to ĵ
 ĵ
B through the smaller angle between them, then
the direction of advancement of the screw gives the
   k̂
ˆi
direction of A  B i.e. C
ˆi

 Properties k̂
i. Vector product of any two vectors is always a
vector perpendicular to the plane containing these
 ˆi  ˆj  k,
ˆ ˆj  kˆ  ˆi and kˆ  ˆi  ˆj
two vectors, i.e., orthogonal to both the vectors A
   And as cross product is not commutative,
and B, though the vectors A and B may or may
ˆj  ˆi  kˆ , kˆ  ˆj  ˆi and ˆi  kˆ  ˆj
not be orthogonal.
ii. Vector product of two vectors is not commutative, ix. In terms of components
     
i.e., A  B  B  A [but  B  A] iˆ ˆj kˆ
 
Here it is worthy to note that A  B  Ax Ay Az
   
| A  B || B  A | ABsin  Bx By Bz
   
i.e. in case of vector A  B and B  A magnitudes
 î(A y B z  A z B y )
are equal but directions are opposite.
iii. The vector product is distributive when the order of  ˆj(A z B x  A x Bz )  k(A
ˆ
x By  A y Bx )
the vectors is strictly maintained, i.e.
      
A  (B  C)  A  B  A  C  Example:
iv. The vector product of two vectors will be Since vector product of two vectors is a vector,
o
maximum when sin   max  1, i.e.,   90 vector physical quantities (particularly representing
  rotational effects) like torque, angular momentum,
[A  B]max  AB nˆ
velocity and force on a moving charge in a magnetic
i.e. vector product is maximum if the vectors are
field and can be expressed as the vector product of
orthogonal.
two vectors. It is well – established in physics that:
v. The vector product of two non- zero vectors will be   
minimum when | sin  |  minimum = 0, (i) Torque   r  F
  
i.e.,   0o or 180o (ii) Angular momentum L  r  p
    
[A  B]min  0
(iii) Velocity v   r
i.e.if the vector product of two non-zero vectors
(iv) Force on a charged particle q moving with velocity
vanishes, the vectors are collinear.     
vi. The self cross product, i.e., product of a v in a magnetic field B is given by F  q(v  B)
vectorbyitself vanishes, i.e., is null vector (v) Torque on a dipole in a field
        
A  A  AAsin 0o nˆ  0 E  p  E and B  M  B

7
PHYSICS-X I-IIT-JEE VECTOR AND BASIC MATHEMATICS

Example-11 [All three sides are taken in order] ...... (i)


  
The adjacent sides of a parallelogram are represented by  a  b  c ...... (ii)

co-initial vectors 2iˆ  3jˆ and ˆi  4jˆ . The area of the Pre-multiplying both sides by a
         
parallelogram is–
a  (a  b)  a  c  0  a  b  a  c
(a) 5 units along z-axis (b) 5 units in x-y plane    
(c) 3 units in x-z plane (d) 3 units in y-z plane  a  b  ca ....... (iii)

Solution:[a] Pre-multiplying both sides of (ii) by b
ˆi ˆj kˆ           
  b  (a  b)   b  c  b  a  b  b  b  c
A × B = 2 3 0 = k̂ (8 – 3) = 5 k̂        
  a  b  b  c  a  b  b  c ...... (iv)
1 4 0      
From (iii) and (iv), we get a  b  b  c  c  a
Taking magnitude, we get
Example-12      
| a  b | | b  c | | c  a |
 
A vector A of magnitude 5 3 units, another vector B  ab sin(180   )  bcsin(180  )
of magnitude 10 units are inclined to each other at an  ca sin(180  )
angle of 30º. The magnitude of the vector product of the  absin   bc sin   ca sin 
two vectors is – Dividing throughout by abc, we have
(a) 1 units (b) 5 3 units sin  sin  sin 
  
a b c
(c) 75 units (d) 25 3 units
Example-13
Solution:[d] A rope is stretched between two poles. A 50 N boy
  hangs from it, as shown in Fig. Find the tensions in the
A × B = 5 3 × 10 × sin 30º
two parts of the rope.
3 units A B
= 25
30º 15º

T1 T2

2. LAMI’S THEOREM 
  W
In any  A BC with sides a,b,c (a) T1 = 68.3 N, T2 = 61.24 N
sin  sin  sin  (b) T1 = 61.24 N, T2 = 68.3 N
 
a b c (c) T1 = 62.3 N, T2 = 65.24 N
(d) T1 = 60 N, T2 = 62.7 N
Solution:[a]
180 – 
In Fig.  = 90º + 15º = 105º
 = 90º + 30º = 120º
And  = 180º – (30º + 15º) = 135º
c b Using Lami's Theorem, we have
T1 T W
 180 – 
 1 
 sin  sin  sin 
sin  sin105
180 –  a  T1 = W × = 50 ×
sin  sin135
i.e. for any triangle the ratio of the sine of the angle
sin 75 50  0.9659
containing the side to the length of the side is a = 50 × = = 68.3 N
constant. sin 45 0.7071
For a triangle whose three sides are in the same W sin  50sin120 50  sin 60
T2 = = =
order we establish the Lami's theorem in the sin  sin135 sin 45
following manner. For the triangle shown 50  0.8660
    = = 61.24 N
abc0 0.7071

8
VECTOR AND BASIC MATH EMATICS PHYSICS-X I-IIT-JEE

Example-14 If the position of frames S relative to S at any time


A weight mg is suspended from the middle of a rope    
whose ends are at the same level. The rope is no longer is r SS then from figure, rPS  rPS  rS S
horizontal. Find the minimum tension required to Differentiating this equation with respect to time
  
completely straighten the rope. drPS drPS drSS
A B  
dt dt dt
     
or v PS  v PS  vSS [as v  d r /dt ]
T 
   
or v PS  v PS  vSS
mg  General Formula: The relative velocity of a
(a) 0 (b)  (c) 1 (d) None 
Solution: particle P1 moving with velocity v1 with respect to
According to Lami'stheorm 
another particle P2 moving with velocity v 2 is
  
given by, v r12 = v1 – v 2
v1
T T mg v2
 
sin  sin  sin  P2

For straighten = 180º,  = 90º,  = 90º P1


T mg
 i. If both the particles are moving in the same direction
sin 90 sin180 then:
mg r12  1 – 2
T= = 
0 ii. If the two particles are moving in the opposite
So the minimum tension required to completely direction, then:
straithen will be infinity.
r12  1  2
3. RELATIVE VELOCITY iii. If the two particles are moving in the mutually
perpendicular directions, then:
 Introduction:When we consider the motion of a
particle, we assume a fixed point relative to which r12  12   22
the given particle is in motion. For example, if we  
say that water is flowing or wind is blowing or a iv. If the angle between 1 and  2 be , then
person is running with a speed v, we mean that 1/2
these all are relative to the earth (which we have r12   12  22 – 212 cos  .
 
assumed to be fixed).
 Relative velocity of satellite: If a satellite is
Y 
Y P
moving in equatorial plane with velocity vs and a

r point on the surface of earth with v e relative to the
PS'
r
PS
centre of earth, the velocity of satellite relative to
X the surface of earth
S   
r S' S vse  vs  v e
X
S So if the satellite moves from west to east (in the
Now to find the velocity of a moving object relative direction of rotation of earth on its axis) its velocity
to another moving object, consider a particle P relative to earth's surface will be vse  vs  ve
 And if the satellite moves from east to west, i.e.,
whose position relative to frame S is rPS while relative to
opposite to the motion of earth,

S is rPS . vse  vs  (ve )  vs  ve

9
PHYSICS-X I-IIT-JEE VECTOR AND BASIC MATHEMATICS

 Relative velocity of rain: 


v RM (Velocity of rain with respect to man) is vertical

If rain is falling vertically with a velocity v R and   
an observer is moving horizontally with downward v RM = v R – vM

speed vM the velocity of rain relative to observer
30º

   vR vRM
will be v RM vR  vM
Which by law of vector addition has magnitude
v RM  vR 2 2
 vM direction   tan 1 (vM / vR ) –vRM

with the vertical as shown in fig.  


| v R | sin 30º = | v M |
10Km / hr 10Km / hr
 vR = = = 20 Km/hr
sin 30º 1/ 2

vR

  Relative velocity of swimmer: If a man can swim


vM

relative to water with velocity v and water is
flowing relative to ground with velocity
 
vR velocity of man relative to ground v M will be

– vM
given by:
      
vR v  v M  v R , i.e., v M  v  v R
vR

 So if the swimming is in the direction of flow of
vM
water, v M  v  v R
And if the swimming is opposite to the flow of
water, v M  v  v R
 Crossing the river:Suppose, the river is flowing
Example-15 
with velocity r . A man can swim in still water
A man standing on a road has to hold his umbrella at
30º with the vertical to keep the rain away. He thrown 
with velocity m . He is standing on one bank of
the umbrella and starts running at 10 km/h. He finds
that rain drop is hitting his head vertically. Find the the river and wants to cross the river, two cases
speed of rain w.r.t. road-
(a) 10 km/s (b) 20 km/h arise.
(c) 10 3 km/s (d) 20 3 km/h (i) To cross the river over shortest distance: That is
Solution:[b]
to cross the river straight, the man should swim
making angle  with the upstream as shown.
 
v RM A vr B

30º vr 
10 Km/hr  
vr
w  v
vm


(1) (2)
Upstream O Downstream

10
VECTOR AND BASIC MATH EMATICS PHYSICS-X I-IIT-JEE
Here OAB is the triangle of vectors, in which (a)at right angle to the stream
   
(b)at an angle of sin–1 (2/5) with PQ up the stream
OA  v m , AB  r . Their resultant is given by (c)at an angle of sin–1 (2/5) with PQ down the stream
(d)at an angle cos–1 (2/5) with PQ down the stream
  Solution:[b]
OB   . The direction of swimming makes angle 
with upstream. From the triangle OBA, we find, Q

cos  
r 
Also sin   r vBR Vr = 4 m/s
m m 
Where  is the angle made by the direction of
P
swimming with the shortest distance (OB) across Velocity of boat with respect to ground must be along
the river. line]
vBR = Velocity of boat w.r.t river
Time taken to cross the river:If wbe the width of      
v BR = v B – v R  v B = v BR + v R
the river, then time taken to cross the river will be

given by As v B is along PQ.
w w vBR sin  = vr
t1  
  v  4
2m – 2r or = sin–1  R  = sin  
 vBR   10 
(ii) To cross the river in shortest possible time: The
2
 angle sin   with PQ up the stream
man should swim perpendicular to thebank. 5
The time taken to cross the river will be: 4. BASIC MATHEMATICS
w 1. Trigonometry functions
t2  180º
m 1.1 1 rad = = 57.3º


A vr B 
1º = = 0.0174 rad.
180
1.2 Trigonometric ratio of certain allied angles
 
w vm vr
sin (– ) = – sin  cos (– ) = cos
sin (90 – ) = cos cos (90 – ) = sin 

Upstream O Downstream
sin (90 + ) = cos cos (90 + ) = – sin 

In this case, the man will touch the opposite bank sin (180 – ) = sin  cos (180 – ) = – cos
at a distance ABdownstream. This distancewill sin (180 + ) = – sin cos (180 + ) = – cos
be given by: sin (270 – )= – cos cos (270 – ) = – sin 
w 
AB   r t 2   r or AB  r w sin (270 + ) = – cos cos (270 + ) = sin 
m m
sin (360 – ) = – sin  cos (360 – ) = cos
Example-16 1.3 Relation involving one angle
A boat man could row his boat with a speed 10m/sec. (i) sin2 + cos2 = 1
He wants to take his boat from P to a point Q just (ii) sec2 = 1 + tan2
opposite on the other bank of the river flowing at a (iii) cosec2 = 1 + cot2
speed 4m/sec. He should row his boat –

11
PHYSICS-X I-IIT-JEE VECTOR AND BASIC MATHEMATICS
1.4 Sum and difference of angles Note:
(i)sin (A ± B) = sin A cos B ± cos A sin B (a) When n is a positive integer, then expansion will
(ii) cos (A ± B) = cos A cos B  sin A sin B have (n + 1) terms
(b) When n is a negative integer, expansion will have
tan A  tan B infinite terms.
(iii) tan (A ± B) =
1  tan A tan B (c) When n is a fraction, expansion will have infinite
1.5 Conversion of sum intoproduct& vice-versa terms.
(i)sin (A + B) + sin (A – B) = 2 sin A cos B 4. Algebraic equations & Solutions
(ii) sin (A + B) – sin (A – B) = 2 cos A sin B 4.1 Linear equation
(iii) cos (A + B) + cos (A – B) = 2 cos A cos B If ax + b = 0
(iv) cos (A + B) – cos (A – B) = – 2sin A sin B b
(v) 2 sin C cos D = sin(C + D) + sin(C – D) then x =  ; one real solution
a
(C  D) (C  D)
(vi)sinC + sin D = 2sin cos 4.2 Quadratic equation
2 2 Ifax2 + bx + c = 0
(C  D) (C  D)
(vii) sin C – sin D = 2cos sin  b  b2  4ac
2 2 thenx =
(C  D) (C  D) 2a
(viii) cos C + cos D = 2cos cos Ifb2 – 4ac > 0,two real solutions
2 2 b2 – 4ac = 0, one real solution
(C  D) (D  C) b2 – 4ac < 0,no real solutions
(ix) cos C – cos D = 2sin sin
2 2 5. Determinant
1.6 Multiplesformulae a b
(i)sin2 = 2 sincos 5.1 D = = ad – bc
c d
(ii)sin3= 3 sin – 4 sin3
(iii)cos2 = cos2 – sin2 a1 a 2 a 3
= 1 – 2sin2 = 2cos2 – 1 5.2 D = b1 b2 b3
(iv)cos3= 4 cos3 – 3 cos c1 c 2 c3
2 tan 
(v)tan2= = a1 (b2 c3 – b3 c2) – a2 (b1 c3 – b3 c1)
1  tan 2  + a3 (b1 c2 – b2 c1)
(vi)2 cot2= cot – tan 6. Straight line, Circle, Ellipse, Parabola,
2. Logarithm Hyperbola
2.1 Base 10 6.1 Straight line
If 10n = x; then log10x = n Equation of straight line: y = mx + c
2.2 Base e y
wherem = slope of line = = tan ;
If em = y; then logey = m x
2.3 Change of base Intercept on y-axis = c
1 c
(i)logba = The intercept on x-axis =
log a b m
y
1 y m=
(ii)logex = log10x m=0 (B)
log10 e (A)
= 2.303 log10x x
x
3. Binomial Theorem y y
m=1 m=–3
n(n  1)x 2 (C) 45º (D)
(i) (1 ± x)n = 1 ± nx + … 120º
2!
x x
n(n  1) 2
(ii) (1 ± x)–n = 1 nx + x …
2! y (1)
(iii) If x <<1 ; then(1 ± x)n = 1 ± nx (E) m1=m2
(Neglecting higher terms) (2)
(1 ± x)–n =1 ± (–n)x = 1 nx x

12
VECTOR AND BASIC MATH EMATICS PHYSICS-X I-IIT-JEE

6.2 Circle (ii) x2 = 4ay


(i) Equation of a circle of centre (0, 0) and Y
radius r
x2 + y2= r2
X
y

y r (x,y)
x 7. Calculus
(0,0) x
7.1 Differentiation
(ii) Equation of a circle of centre at (a, b) and 7.1.1 Geometrical meaning
radius r dy
Let y = f (x), be a function of x then
(x – a)2 + (y – b)2 = r2 dx
showsthe slope of the tangent at any point of
y
the curve.
r
b (a,b)
7.1.2 Physical meaning
a x
(0,0) d
is a symbol for rate of change w.r.t. x
6.3 Ellipse dx
orderivative w.r.t. x.
x 2 y2
Equation of a ellipse is given by 2  2 = 1
a b 7.1.3 Differential coefficient of some
Y standardfunctions
B
(x, y) d n

b (i) (x ) = nxn–1
O F A dx
c a X
d x
(ii) (e ) = ex
dx
F and F´ = foci = 2c d x
(iii) (a ) = axloge a
Where c = a 2  b2 dx
a = OA = semi major axis d 1
(iv) (logex) =
b = OB= semi minor axis dx x
c d 1
Eccentricity () = (v) (logax) = logae
a dx x
Area of ellipse = ab
d
(vi) (sinx) = cos x
dx
6.4 Parabola
d
(i) y2 = 4ax (vii) (cosx) = –sin x
dx
Y
d
(viii) (tanx) =sec2 x
dx
A F
X
(0, 0) (a, 0) d
(ix) (cotx) = – cosec2 x
dx
d
F = focus = (a, 0); A = Vertex = (0, 0) (x) (sec x) = sec x tan x
dx

13
PHYSICS-X I-IIT-JEE VECTOR AND BASIC MATHEMATICS

d dy
(xi) (cosec x) = –cosec x. cotx =0
dx dx
d 1 This is the condition for maxima or minima
(xii) (sin–1x) = ; – 1< x < 1
dx 1  x2 A
d 1
(xiii) (cos–1x) = ; – 1< x < 1
dx 1  x2
B
d 1
(xiv) (tan–1x) =
dx 1  x2 Note:
d 1
(xv) (cot–1 x) = dy
dx 1  x2 To check whether the point where = 0 is a
dx
d 1
(xvi) (sec–1 x) = |x| >| d2y
dx x x2 1 maximum or minimum, one must evaluate .
dx 2
d 1
(xvii) (cosec–1 x) = |x| >|
dx x x2 1 d2y
If < 0 (negative), then the point is maximum.
d dx 2
(xviii) (c) = 0, where c is a constant
dx d2y
If > 0 (positive), then the point is minimum
7.1.4 Rules for finding derivatives of composite dx 2
functions
d dy 7.2 Integration
(i) (cy) = c 7.2.1Meaning:
dx dx
d d
(ii) (u ± v ± w ± …) If [F(x)] = f(x), then F(x) called the
dx dx
du dv dw integral of f(x). It is written as . It is
= ±
dx dx
±
dx
±…  f (x) dx
where u, v, w … are all functions of x read as integral of f(x) w.r.t. x.
dy dv du
(iii) (uv) = u +v
dx dx dx 7.2.2 Integral of some standard function
du du
v u n x n 1
d u dx dx (i)  x dx = + C (n  –1)
(iv)   = 2 n 1
dx  v  v
(v) 1
(ii)  x . dx = loge| x | + C
(a) chain rule of differentiation
dy dy du (iii)
dx
= .
du dx
 k dx = kx + C
x
(b) Extended chain rule (iv) e dx = ex + C
dy dy dx1 dx 2 dx n
= …
dx dx1 dx 2 dx 3 dx (vi)  sin x dx = – cosx + C

7.1.5 Maxima & Minima (vii)  cos x dx = sinx + C


In fig. at point A the curve has a
maximumvalue, and at point B, the curve has a (viii)  tan x dx = log secx + C
minimum. At these points the tangent to the curve 2
(dotted lines) is parallel to the X-axis. If the curve is (ix)  sec x .dx = tanx + C
described by y = f(x) then, both at maximum and at Where C is the constant of integration.
minimum the slope of tangent is zero.

14
VECTOR AND BASIC MATH EMATICS PHYSICS-X I-IIT-JEE

eax  b eax b 1
7.2.3  eax  b .dx = = +C or
d a x(a  x)
(ax  b)
dx 2
x = a sec 
x xa
7.2.4 Rules for finding the integration of (v) or
xa x
composite functions.
or x(x  a)
(i)  k f (x) dx = k.  f(x) dx,where k is a
1
constant or
x(x  a)
(ii) If  f(x) dx = F(x) + C;then 2
x xa x = a tan 
1 (vi) or
 f(kx) dx = F(kx) + C xa x
K
or x(a  x)
(iii)  [f(x)±g(x)]dx=  f(x)dx±  g(x)dx
1
or
(iv) Integration by parts: x(a  x)
Integration of a product of two functions is
x = a cos2
given by: (first function) × (integral of ax ax
(vii) or
second function) – integral of [(differential ax ax
coefficient of first) × (integral of second)]

 f(x) (gx) dx = f(x).  g(x)dx


7.2.6 Definite Integral:
–  [f´(x) g(x).dx]dx b
b
 f (x) dx =  F(x)a = F(b) – F(a)
7.2.5 Integration by substitution method a
Standard Substitution: following standard Note:
substitutions will be useful. The constant of integration vanishes in definite
integration
Integrand form substitution Some properties of definite integral:
1 x = a sin b a
(i) a2  x2 or (i)  f (x) dx = –  f (x) dx
a2  x2 a b
1 x = a tan b c b
(ii) a2  x2 or (ii)  f (x) dx =  f (x) dx +  f (x) dx
x2  a 2 a a c
1 x = a sec Where a < c < b
(iii) x2  a2 or
x2  a2
2
x ax x = a sin 
(iv) or
ax x
or x(a  x)

15
PHYSICS-X I-IIT-JEE VECTOR AND BASIC MATHEMATICS

EXERCISE # 1
Based On Basics of Vector

1. The vector projection of a vector 3iˆ  4kˆ on 8. Five equal forces of 10 N each are applied at one
y-axis is
point and all are lying in one plane. If the angles
(A)5 (B) 4 (C)3 (D) Zero
between them are equal, the resultant force will
2. If a particle moves from point P (2,3,5) to point be
Q (3,4,5). Its displacement vector be
(A)Zero (B)10 N (C)20 N (D) 10 2N
(A) ˆi  ˆj  10kˆ (B) ˆi  ˆj  5kˆ
(C) ˆi  ˆj (D) 2iˆ  4jˆ  6kˆ 9. A boy walks uniformally along the sides of a
rectangular park of size 400 m× 300 m, starting
3. If A  3iˆ  4jˆ and B  7iˆ  24j,
ˆ the vector having from one corner to the other corner diagonally
the same magnitude as B and parallel to A is opposite. Which of the following statement is
(A) 5iˆ  20 ˆj (B) 15 ˆi  10 ˆj incorrect
(A) He has travelled a distance of 700 m
(C) 20iˆ  15 ˆj (D) 15iˆ  20 ˆj
(B) His displacement is 700 m

4. Vector A makes equal angles with x,y and z axis. (C) His displacement is 500 m
Value of its components (in terms of magnitude (D) His velocity is not uniform throughout the

of A ) will be walk
A A 3
(A) (B) (C) 3 A (D) 10. The unit vector parallel to the resultant of the
3 2 A  
 vectors A  4iˆ  3jˆ  6kˆ and B  ˆi  3jˆ  8kˆ is
5. If A  2iˆ  4jˆ  5kˆ the direction of cosines of the
 1 1
(A) (3iˆ  6ˆj  2k)
ˆ (B) (3iˆ  6ˆj  2k)
ˆ
vector A are 7 7
2 4 5 1 ˆ ˆ 1 ˆ ˆ
(A) , and ˆ ˆ
45 45 45 (C) (3i  6 j  2k) (D) (3i  6j  2k)
49 49
1 2 3
(B) , and
45 45 45 11. With respect to a rectangular cartesian coordinate
4 4 
(C) , 0 and system, three vectors are expressed as a  4iˆ  ˆj ,
45 45 
3 2 5 b  3iˆ  2jˆ and c  kˆ where ˆi, ˆj,kˆ are unit
(D) , and
45 45 45 vectors, along the X, Y and Z-axis respectively.
The unit vectors r̂ along the direction of sum of
 1 ˆ 1 ˆ
6. The expression  i j  is a these vector is
 2 2 
1 ˆ ˆ ˆ 1 ˆ ˆ ˆ
(A)Unit vector (A) r̂  (i  j  k) (B) r̂  (i  j  k)
(B)Null vector 3 2
1 1 ˆ ˆ ˆ
(C)Vector of magnitude 2 (C) r̂  (iˆ  ˆj  k)
ˆ (D) r̂  (i  j  k)
(D)Scalar 3 2

7. A vector is represented by 3iˆ  ˆj  2kˆ . Its length 12. The angle between the two vectors
in XY plane is  
A  3iˆ  4jˆ  5kˆ and B  3iˆ  4jˆ  5kˆ will be
(A)2 (B) 14 (C) 10 (D) 5 (A)90° (B) 0° (C)60° (D) 45°

16
VECTOR AND BASIC MATH EMATICS PHYSICS-X I-IIT-JEE
  
Based On Addition and Subtraction of Vectors 19. The magnitude of vector A, B and C are
13. There are two force vectors, one of 5 N and other   
respectively 12, 5 and 13 units and A  B  C
of 12 N at what angle the two vectors be added to  
get resultant vector of 17 N, 7 N and 13 N then the angle between A and B is
respectively (A) 0 (B)  (C)  / 2 (D)  / 4
(A) 0°, 180° and 90° (B) 0°, 90° and 180°
20. A particle has displacement of 12 m towards east
(C) 0°, 90° and 90° (D) 180°, 0° and 90°
and 5 m towards north then 6 m vertically
 
14. If A  4iˆ  3jˆ and B  6iˆ  8jˆ then magnitude upward. The sum of these displacements is
  (A) 12 (B) 10.04 m
and direction of A  B will be (C) 14.31 m (D) None of these
(A) 5, tan 1 (3 / 4) (B) 5 5, tan 1 (1 / 2)   
21. Let C  A  B then
(C) 10, tan 1 (5) (D) 25, tan 1 (3 / 4)  
(A) | C | is always greater then | A |
15. A truck travelling due north at 20 m/s turns west    
and travels at the same speed. The change in its (B) It is possible to have | C | | A | and | C |  | B|
velocity be (C) C is always equal to A + B
(D) C is never equal to A + B
(A) 40 m/sN–W (B) 20 2 m/s N–W
22. The sum of two forces acting at a point is 16 N. If
(C) 40 m/s S–W (D) 20 2 m/sS–W the resultant force is 8 N and its direction is
16. An object of m kg with speed of vm/s strikes a perpendicular to minimum force then the forces
wall at an angle  and rebounds at the same speed are
and same angle. The magnitude of the change in (A) 6 N and 10 N (B) 8 N and 8 N
momentum of the object will be (C) 4 N and 12 N (D) 2 N and 14 N
(A) 2m v cos  (B) 2m vsin  23. If vectors P, Q and R have magnitude 5, 12 and
  
(C) 0 (D) 2m v 13 units and P  Q  R, the angle between Q and
 
17. Two vectors A and B lie in a plane, another R is
 5 5
vector C lies outside this plane, then the resultant (A) cos 1 (B) cos 1
   12 13
of these three vectors i.e., A  B  C 12 7
(C) cos 1 (D) cos 1
(A) Can be zero 13 13
(B) Cannot be zero   
  24. The resultant of P and Q is perpendicular to P .
(C) Lies in the plane containing A  B
  
(D) Lies in the plane containing C What is the angle between P and Q
 (A) cos 1 (P / Q) (B) cos 1 (P / Q)
18. Let the angle between two nonzero vectors A
 
and B be 120° and resultant be C (C) sin 1 (P / Q) (D) sin 1 (P / Q)
  
(A) C must be equal to | A  B | 25. Maximum and minimum magnitudes of the
   resultant of two vectors of magnitudes P and Q
(B) C must be less than | A  B | are in the ratio 3 : 1. Which of the following
   relations is true
(C) C must be greater than | A  B |
 (A) P  2Q (B) P  Q
 
(D) C may be equal to | A  B | (C) PQ  1 (D) None of these

17
PHYSICS-X I-IIT-JEE VECTOR AND BASIC MATHEMATICS
     32. Three concurrent forces of the same magnitude
26. Given that A  B  C and that C is  to A .
  are in equilibrium. What is the angle between the
Further if | A | | C |, then what is the angle between forces ? Also name the triangle formed by the
 
A and B forces as sides
  (A)60° equilateral triangle
(A) radian (B) radian
4 2 (B) 120° equilateral triangle
3 (C)120°, 30°, 30° an isosceles triangle
(C) radian (D)  radian
4 (D) 120° an obtuse angled triangle
    
27. A plane is revolving around the earth with a 33. If |A  B|  |A|  |B| , then angle between A and
speed of 100 km/hr at a constant height from the 
B will be
surface of earth. The change in the velocity as it
(A)90° (B) 120° (C)0° (D) 60°
travels half circle is
(A) 200 km/hr (B) 150 km/hr
34. The maximum and minimum magnitude of the
(C) 100 2 km / hr (D) 0 resultant of two given vectors are 17 units and 7
unit respectively. If these two vectors are at right
28. What displacement must be added to the angles to each other, the magnitude of their
displacement 25iˆ  6jˆ m to give a displacement resultant is
of 7.0 m pointing in the x-direction (A)14 (B) 16 (C)18 (D) 13

(A) 18iˆ  6ˆj (B) 32iˆ  13jˆ


35. y component of velocity is 20 and x component of
(C) 18iˆ  6ˆj (D) 25iˆ  13jˆ velocity is 10. The direction of motion of the
body with the horizontal at this instant is
29. While travelling from one station to another, a car
(A) tan 1 (2) (B) tan 1 (1 / 2)
travels 75 kmNorth, 60 km North-east and 20 km
East. The minimum distance between the two (C)45° (D) 0°
stations is
(A)72 km (B) 112 km 36. Two equal forces (P each) act at a point inclined
(C)132 km (D) 155 km to each other at an angle of 120°. The magnitude
of their resultant is
30. A scooter going due east at 10 ms–1 turns right (A) P / 2 (B) P / 4 (C) P (D) 2P
through an angle of 90°. If the speed of the
scooter remains unchanged in taking turn, the 37. A car travels 6 km towards north at an angle of
change is the velocity of the scooter is 45° to the east and then travels distance of 4 km
(A)20.0 ms–1 south eastern direction towards north at an angle of 135° to the east.
(B) Zero How far is the point from the starting point. What
(C)10.0 ms–1 in southern direction angle does the straight line joining its initial and
(D) 14.14 ms–1 in south-west direction final position makes with the east
(A) 50 km and tan 1 (5)
31. Two forces 3N and 2 N are at an angle  such that
the resultant is R. The first force is now increased (B) 10 km and tan 1 ( 5)
to 6N and the resultant become 2R. The value of
(C) 52 km and tan 1 (5)
 is
(A)30° (B) 60° (C)90° (D) 120° (D) 52 km and tan 1 ( 5)

18
VECTOR AND BASIC MATH EMATICS PHYSICS-X I-IIT-JEE
   42. Figure below shows a body of mass M moving
38. Given that A  B  C = 0 out of three vectors two
with the uniform speed on a circular path of
are equal in magnitude and the magnitude of third radius, R. What is the change in acceleration in
vector is 2 times that of either of the two going from P1 to P2
having equal magnitude. Then the angles between P2
vectors are given by v

(A) 30°, 60°, 90° (B) 45°, 45°, 90°


P1
(C) 45°, 60°, 90° (D) 90°, 135°, 135° R

39. At what angle must the two forces (x + y) and (x


2 2
(A) Zero (B) v 2 / 2R
– y) act so that the resultant may be (x  y )
v2
(C) 2v 2 / R (D)  2
 x 2  y2  R
(A) cos 1   
 2(x 2  y 2 )  43. A particle is moving on a circular path of radius r
 
with uniform velocity v. The change in velocity
 2(x 2  y 2 )  when the particle moves from P to Q is
(B) cos 1   
 x 2  y 2  ( POQ  40)

P
1
 x 2  y2  r
(C) cos  2 
 x  y2  o
  O 40 v

 x2  y2 
(D) cos 1    Q
 x 2  y2  v
 
(A) 2v cos 40 (B) 2vsin 40
(C) 2vsin 20 (D) 2v cos 20
40. Following forces start acting on a particle at rest
Based On Multiplication of Vectors
at the origin of the co-ordinate system
simultaneously 44. A particle moves from position 3iˆ  2jˆ  6kˆ to
 
F1  4iˆ  5jˆ  5kˆ , F2  5iˆ  8jˆ  6kˆ , 14iˆ  13jˆ  9kˆ due to a uniform force of
  (4iˆ  ˆj  3k)
ˆ N. If the displacement in meters
F3  3iˆ  4jˆ  7kˆ and F4  2iˆ  3jˆ  2kˆ then the
then work done will be
particle will move (A) 100 J (B)200 J (C)300 J (D)250 J
(A) In x – y plane (B) In y – z plane  
45. The angle between the vectors A and B is
(C) In x – z plane (D) Alongx -axis   
. The value of the triple product A.(B  A) is

   (A) A2B (B)Zero


41. The resultant of A  B is R1. On reversing the
(C) A 2 Bsin  (D) A 2 Bcos 
  
vector B, the resultant becomes R 2 . What is the 46. The torque of the force F  (2iˆ  3jˆ  4kˆ )N

value of R12  R 22 acting at the point r  (3iˆ  2jˆ  3k)
ˆ m about the
origin be
(A) A2  B2 (B) A2  B2
(A) 6iˆ  6jˆ  12kˆ (B) 17iˆ  6jˆ  13kˆ
2 2 2 2
(C) 2(A  B ) (D) 2(A  B )
(C) 6iˆ  6jˆ  12kˆ (D) 17iˆ  6jˆ  13kˆ

19
PHYSICS-X I-IIT-JEE VECTOR AND BASIC MATHEMATICS

47. If a particle of mass m is moving with constant 54. A vector F1 is along the positive X-axis. If its
velocity v parallel to x-axis in x-y plane as shown 
in fig. Its angular momentum with respect to vector product with another vector F2 is zero then

origin at any time t will be F2 could be
(A) mvbkˆ (B) mvb kˆ
(A) 4jˆ (B) (iˆ  ˆj)
(C) mvb ˆi (D) mv ˆi
 (C) (jˆ  k)
ˆ ˆ
(D) (4i)
48. Consider two vectors F1  2iˆ  5kˆ and    
 55. If for two vectors A and B, A  B  0, the vectors
F2  3jˆ  4k.
ˆ The magnitude of the scalar product
(A)Are perpendicular to each other
of these vectors is
(B)Are parallel to each other
(A)20 (B)23 (C) 5 33 (D)26
(C)Act at an angle of 60°
   
49. If | V1  V2 | | V1  V2 | and V2 is finite, then (D)Act at an angle of 30°

(A) V1 is parallel to V2 
  56. Let ˆ
A  iAcos   ˆjAsin 
be any vector.
(B) V1  V 2 
Another vector B which is normal to A is
(C) V1 and V2 are mutually perpendicular
  (A) î Bcos   jBsin  (B) ˆi B sin   j B cos 
(D) | V1 |  | V2 |
 (C) î Bsin   jBcos  (D) î Bcos   jBsin 
50. A force F  (5iˆ  3j)
ˆ Newton is applied over a
particle which displaces it from its origin to the 57. The angle between two vectors given by

point r  (2iˆ  1j)
ˆ metres. The work done on the
6 i  6 j  3k and 7i  4j  4k is
particle is
 1   5 
(A)– 7 J (B)+13 J (C)+7 J (D) +11 J (A) cos1   (B) cos1  
 3  3
51. A particle moves with a velocity
 2   5
6iˆ  4jˆ  3kˆ m / s under the influence of a (C) sin 1   (D) sin 1  
  3  3 
constant force F  20iˆ  15jˆ  5kˆ N. The
instantaneous power applied to the particle is
58. Angle between the vectors (iˆ  ˆj) and (jˆ  k)
ˆ is
(A)35 J/s (B)45 J/s (C)25 J/s (D)195 J/s
(A)90° (B)0° (C)180° (D)60°
52. A body, constrained to move in the Y-direction is
subjected to a force given by
 59. The position vectors of points A, B, C and D are
F  (2iˆ  15jˆ  6k)
ˆ N. What is the work done by
A  3iˆ  4jˆ  5k,
ˆ B  4iˆ  5jˆ  6k,
ˆ C  7iˆ  9jˆ  3kˆ
this force in moving the body a distance 10 m
along the Y-axis and D  4iˆ  6jˆ then the displacement vectors
(A)20 J (B)150 J (C)160 J (D)190 J AB and CD are
53. A particle moves in the x-y plane under the action (A)Perpendicular
 (B)Parallel
of a force F such that the value of its linear
 (C)Antiparallel
momentum (P) at is anytimet
 (D)Inclined at an angle of 60°
Px  2cos t,py  2sin t. The angle  between F
      
and P at a given time t. will be 60. If | A  B| | A.B |, then angle between A and B
(A)   0 (B)   30 will be
(C)   90 (D)   180  (A)30° (B)45° (C)60° (D)90°

20
VECTOR AND BASIC MATH EMATICS PHYSICS-X I-IIT-JEE
61. The linear velocity of a rotating body is given by 68. The position vectors of radius are 2iˆ  ˆj  kˆ and
    
v    r, where  is the angular velocity and r 2iˆ  3jˆ  kˆ while those of linear momentum are
is the radius vector. The angular velocity of a body
 2iˆ  3jˆ  k.
ˆ Then the angular momentum is
is   ˆi  2jˆ  2kˆ and the radius vector
  (A) 2iˆ  4kˆ (B) 4iˆ  8kˆ
r  4jˆ  3k,
ˆ then | v | is
(C) 2iˆ  4jˆ  2kˆ (D) 4iˆ  8kˆ
(A) 29 units (B) 31 units
 
(C) 37 units (D) 41 units 69. When A.B   | A || B |, then
 
   (A) A and B are perpendicular to each other
 
62. Three vectors a, b and c satisfy the relation (B) A and B act in the same direction
      
a.b  0 and a.c  0. The vector a is parallel to (C) A and B act in the opposite direction
 
      (D) A and B can act in any direction
(A) b (B) c (C) b.c (D) b  c
     
63. What is the unit vector perpendicular to the 70. If | A  B| 3A.B, then the value of | A  B | is
1/2
following vectors 2iˆ  2jˆ  kˆ and 6iˆ  3jˆ  2kˆ  2 2 AB 
(A)  A  B  
ˆi  10jˆ  18kˆ ˆi  10ˆj  18kˆ  3
(A) (B) (B) A  B
5 17 5 17
ˆi  10ˆj  18kˆ ˆi  10jˆ  18kˆ (C) (A2  B2  3AB)1/2
(C) (D)
5 17 5 17 (D) (A2  B2  AB)1/2

64. The area of the parallelogram whose sides are 71. A force F  3iˆ  cjˆ  2kˆ acting on a particle

represented by the vectors ˆj  3kˆ and ˆi  2ˆj  kˆ causes a displacement S  4iˆ  2jˆ  3kˆ in its
is own direction. If the work done is 6J, then the
(A) 61 sq.unit (B) 59 sq.unit value of c will be
(A)12 (B)6 (C) 1 (D) 0
(C) 49 sq.unit (D) 52 sq.unit

65. Two vectorA and B have equal magnitudes. Then 72. A force F  (5iˆ  3j)
ˆ N is applied over a particle
the vector A + B is perpendicular to which displaces it from its original position to the

(A) A  B (B)A – B point s  (2iˆ  1j)
ˆ m. The work done on the
(C)3A – 3B (D)All of these particle is
    (A)+ 11 J (B)+ 7 J (C) + 13 J (D) – 7 J
66. The value of (A  B)  (A  B) is
73. Two constant forces F1  2iˆ  3jˆ  3kˆ (N) and
(A)0 (B) A2  B2
    F2  ˆi  ˆj  2kˆ (N) act on a body and displace it
(C) B  A (D) 2(B  A)
from the position r1  iˆ  2jˆ  2kˆ (m) to the
67. Two adjacent sides of a parallelogram are
position r2  7iˆ  10jˆ  5kˆ (m). What is the work
represented by the two vectors ˆi  2jˆ  3kˆ and
done
3iˆ  2jˆ  kˆ . What is the area of parallelogram (A) 9 J (B) 41 J
(A)8 (B) 8 3 (C) 3 8 (D) 192 (C) – 3 J (D) None of these

21
PHYSICS-X I-IIT-JEE VECTOR AND BASIC MATHEMATICS

Based On Lami’s Theorem 79. A man standing on a road hold his umbrella at
30° with the vertical to keep the rain away. He
74. P, Q and R are three coplanar forces acting at a throws the umbrella and starts running at 10
point and are in equilibrium. Given P = 1.9318 kg km/hr. He finds that raindrops are hitting his head
wt, sin 1  0.9659, the value of R is ( inkg wt vertically, the speed of raindrops with respect to the
road will be
(A)10 km/hr (B)20 km/hr
P 150o Q (C)30 km/hr (D)40 km/hr
2 1
R 80. A boat is moving with a velocity 3i + 4j with
respect to ground. The water in the river is moving
1 with a velocity – 3i – 4jwith respect to ground.
(A)0.9659 (B)2 (C)1 (D) The relative velocity of the boat with respect to
2
water is
75. If a body is in equilibrium under a set of non- (A)8j (B)– 6i – 8j
collinear forces, then the minimum number of (C)6i +8j (D) 5 2
forces has to be
81. A 150 m long train is moving to north at a speed
(A)Four (B)Three (C)Two (D)Five
of 10 m/s. A parrot flying towards south with a
76. How many minimum number of non-zero vectors speed of 5 m/s crosses the train. The time taken
by the parrot the cross to train would be:
in different planes can be added to give zero
(A)30 s (B)15 s (C)8 s (D)10 s
resultant
(A)2 (B)3 (C)4 (D)5 82. A river is flowing from east to west at a speed of
5 m/min. A man on south bank of river, capable
77. As shown in figure the tension in the horizontal of swimming 10m/min in still water, wants to
cord is 30 N. The weight W and tension in the swim across the river in shortest time. He should
swim
string OA in Newton are
(A)Due north
A (B)Due north-east
30o (C)Due north-east with double the speed of river
30 N (D)None of these
O
83. A person aiming to reach the exactly opposite
W
point on the bank of a stream is swimming with a
speed of 0.5 m/s at an angle of 1200 with the
(A)30 3, 30 (B)30 3, 60
direction of flow of water. The speed of water in
(C)60 3, 30 (D)None of these the stream is
(A)1 m/s (B)0.5 m/s
Relative Velocity (C)0.25 m/s (D)0.433 m/s
Based On
78. Two cars are moving in the same direction with 84. A thief is running away on a straight road on a
the same speed 30 km/hr. They are separated by a jeep moving with a speed of 9 m/s. A police man
distance of 5 km, the speed of a car moving in the chases him on a motor cycle moving at a speed of
10 m/s. If the instantaneous separation of jeep
opposite direction if it meets these two cars at an
from the motor cycle is 100 m, how long will it
interval of 4 minutes, will be take for the policemen to catch the thief
(A)40 km/hr (B)45 km/hr (A)1 second (B)19 second
(C)30 km/hr (D)15 km/hr (C)90 second (D)100 second

22
VECTOR AND BASIC MATH EMATICS PHYSICS-X I-IIT-JEE
85. A man can swim with velocity v relative to water. 90. A steam boat goes across a lake and comes back
He has to cross a river of width d flowing with a (a) On a quiet day when the water is still and (b)
velocity u (u >v). The distance through which he On a rough day when there is uniform air current
is carried downstream by the river is x. Which of so as to help the journey onward and to impede
the following statement is not correct the journey back. If the speed of the launch on
(A) If he crosses the river in minimum time
both days was same, in which case it will
du
x complete the journey in lesser time
v
du (A) Case (a)
(B) xcannot be less than (B) Case (b)
v
(C) For x to be minimum he has to swim in a (C) Same in both
 1  v  (D) Nothing can be predicted
direction making an angle of  sin  
2 u
with the direction of the flow of water 91. A swimmer can swim in still water with speed 
(D) None of these
and the river is flowing with velocity v/2. To
86. A man sitting in a bus travelling in a direction cross the river in shortest distance, he should
from west to east with a speed of 40 km/h swim making angle  with the upstream. What is
observes that the rain-drops are falling vertically the ratio of the time taken to swim across the
down. To the another man standing on ground the shortest time to that is swimming across over
rain will appear
shortest distance
(A) To fall vertically down
(A)cos (B) sin 
(B) To fall at an angle going from west to east
(C) To fall at an angle going from east to west (C)tan (D) cot 
(D) The information given is insufficient to
decide the direction of rain. Based On Basic Mathematics
87. A boat takes two hours to travel 8 km and back in x dx
still water. If the velocity of water is 4 km/h, the
92.  (1  x)
time taken for going upstream 8 km and coming 2 3/2 2 5/2
(A) x + x +C
back is 3 5
(A)2h 2 3/2 2 5/2
(B) – x + x +C
(B)2h 40 min 3 5
(C)1h 20 min 2 3/2 2 5/2
(C) – x – x +C
(D) Cannot be estimated with the information 3 5
given 2 3/2 2 5/2
88. A 120 m long train is moving towards west with a (D) + x – x +C
3 5
speed of 10 m/s. A bird flying towards east with a
speed of 5 m/s crosses the train. The time taken
d
by the bird to cross the train will be 93. 2x 2  1
dx
(A)16 sec (B)12 sec (C) 10 sec (D) 8 sec
(A) 2x (2x2 + 1)1/2 (B) 2x (2x2 + 1)–1/2
89. A boat crosses a river with a velocty of 8 km/h. If (C) (2x2 + 1)1/2 (D) (2x2 + 1)–1/2
the resulting velocity of boat is 10 km/h then the
velocity of river water is 94 Amongst all pairs of positive numbers with
(A)4 km/h (B)6 km/h product 256. Find those whose sum is the least.
(C) 8 km/h (D) 10 km/h (A) 16 (2) 18 (C) 15 (D) 13

23
PHYSICS-X I-IIT-JEE VECTOR AND BASIC MATHEMATICS
95. Graph of y = 3x2 – 4x + 1 is - 1
1
y y 99. Value of  (3  2x)2 dx is -
0
1 2
(A) (B) (A) – (B) –
x x 9 9
4
(C) – (D) None of these
y y 9

100. sin 100  is equal to -


(C) x (D) x
(A) 1 (B) 100
1
(C)Zero (D)
2
96. Graph of x = 2(y – 1)2 is -
y y
1 x dy
101. y = then is equal to -
x dx
e
(A) x (B) x x x
(A) (B) –
x
e ex
y y (x  1)
(C) (D) None of these
ex

(C) x (D) x 102. Which of the following graphs has positive slope
(m) and negative intercept (C) on y-axis.

97. The minimum value of y = 2x2 – x + 1 is - y y


(A) (B)
3 5 7 9
(A) – (B) – (C) – (D) –
8 8 8 8 O x O x

y
2 2 dy  y
98. Ify = sin x – 2 tan x , then at x = is - (C) O x (D)
dx 4 x
(A) – 11 (B) – 7 (C) – 13 (D) – 15 O

24
VECTOR AND BASIC MATH EMATICS PHYSICS-X I-IIT-JEE

EXERCISE # 2
Question Multiple Correct Answer Type 5. The arrow shown below represent all the force
vectors that are applied to a single point. Select

1. Vector R is the resultant of the vectors the correct statements -
  
A and B . Ratio of maximum value of | R | to F
90º 3F


3 |A|
the minimum value of | R | is . The may 150º
1 
|B| 2F
be equal to -
2 1 4 3
(A) (B) (C) (D)
1 2 1 1 (A) The point may be moving at a constant
velocity
2. A man is walking toward east with a velocity of (B) The point may not moving
8 km/h. Wind is blowing toward north-east at
(C) The point is accelerating at a constant rate
angle of 45°. To the man wind appears to blow of
(D) The point is not accelerating
angle of 60° north of west –
8 6 6. If y = kx2 where k is positive non-zero constant,
(A) True velocity of wind is km/hr
1 3 then which of the following graphs is/are correct ?
(B) Velocity of wind relative to man is
16 y y
km/h (A) (B)
1 3
6 O x O x2
(C) True velocity of wind is km/h
1 3
(D) Velocity of wind relative to man is (C) y (D) y
8 3 x x
km/h O O
1 3
7. Which of the following statement is/are correct
 
3. Maximum value of resultant of A and B is 10 (Figure) ?
and minimum value of resultant of these two y

vectors is4. The value of | A | may be - 
(A) 10 (B) 4 (C) 7 (D) 3 d2

4. A particle is moving with acceleration d1

a  (iˆ  ˆj) m/s and its initial velocity (at t = 0) is
2
x
 
v 0  (iˆ  ˆj) m/s. Then select the correct statement– (A) The sign of the x-component and d1 is

(A) Magnitude of displacement of particle in first positive and that of d 2 is negative
 
5 (B) The signs of the y-components of d1 and d 2
second is m
2 are positive and negative respectively
(B) Rate of change of speed at t = 0 is zero (C) The signs of the x and y components of
(C) Rate of change of speed at t = 0 is 2 m/s2  
d1  d 2 are positive
(D) Speed of particle at t = 2s is 10 m/s (D) None of these

25
PHYSICS-X I-IIT-JEE VECTOR AND BASIC MATHEMATICS
 
8. Given two vectors A = 3iˆ  4jˆ and B = ˆi  ˆj ,  12. The vectors (x2 – 7) î + (x – 2y + 1) ĵ + (y2 – 3) k̂
 
is the angle between A and B . Which of the and 2 î + k̂ are parallel, then ordered pair
following statements is/are correct ? (x, y) can be
  ˆi  ˆj    (A) (3, 2) (B) (– 3, – 1)
(A) A cos 
 2  is the component of A & B (C) (–1 , 0) (D) (1, 0)
 
  ˆi  ˆj   
(B) A sin  13. A vector ( d ) is equally inclined to three vectors
 2  is the component of A 
    
 a  iˆ  ˆj  kˆ , b  2iˆ  ˆj and c  3jˆ  2kˆ . Let x
perpendicular to B
    
  ˆi  ˆj    , y , z be three vector in the plane of a , b ; b,
(C) A cos   is the component of A along B   
 2  c ; c , a respectively then
  ˆi  ˆj    
(A) x . d = 14
(D) A sin   is the component of A
 2   
 (B) y . d = 3
perpendicular to B  
(C) z . d = 0
9. Consider three vectors      
   (D) r . d = 0 where = r  x + µ y +  z
a = ˆi  ˆj  kˆ , b = ˆi  ˆj  kˆ and c = ˆi  ˆj .
   14. Which of the following statements is/are
If k1 a + k2 b + k3 c = 4iˆ  6jˆ  kˆ , where k1, k2
correct ?
and k3 are scalars, then-      
(A) if n . a = 0, n . b = 0 & n . c = 0 for
(A)k1 = 2 (B) k2 = 3 (C) k3 = –1 (D) k3 = 1
    
some non zero vector n , then  a b c  = 0
10. The vector c , directed along the internal bisector
of the angle between the vectors (B) there exist a vector making angles 30º and
 ˆ ˆ ˆ  
a  7i  4 j  4k & b  2iˆ  ˆj  2kˆ with | c | = 45º with x-axis and y-axis respectively.
(C) locus of point for which x = 3 & y = 4 is a
5 6 is:
line parallel to the z-axis whose distance
5 5
(A) (iˆ  7 ˆj  2k)
ˆ (B) (iˆ  7 ˆj  2k)
ˆ
from the z axis is 5
3 3
5 5 (D) the vertices of a regular tetrahedron are O, A,
(C) (  ˆi  7 ˆj  2k)
ˆ (D) (  ˆi  7 ˆj  2k)
ˆ
3 3 B, C where 'O' is the origin. Thevector
  
11. In a four - dimensional space where unit vectors OA  OB  OC is perpendicular to the
   
along axes are ˆi, ˆj, kˆ and ̂ and a1, a 2 , a 3 , a 4 are plane ABC.
four non-zero vectors such that no vector can be

expressed as linear combination of others and 15. Let a = x î + x2 ĵ + 2 k̂ , = –3 î + ĵ + k̂ ,
      
( – 1) (a1  a 2 ) + µ (a 2  a 3 ) + (a 3  a 4  2a 2 ) 
c = (3x + 11) î + (x–9) ĵ – 3 k̂ be three vectors.
 
+ a 3 +  a 4 = 0 then  
Then angle between a and b is acute and angle
(A) = 1 (B) µ = 
2  
between c and a is obtuse, if x lies in
3
2 1 (A) (– , 1)  (2, 3) (B) (– , 1)
(C) =   (D) =
3 3 (C) (2, 3) (D) none of these

26
VECTOR AND BASIC MATH EMATICS PHYSICS-X I-IIT-JEE

Question Passage Based Questions 20. The correct acceleration (A)vs time (t) graph is-

Passage # 1 (Q.No16 to 18) a 2 a


A man in a boat crosses a river from point A. If he rows (m/s2) (m/s2)
(A) (B)
perpendicular to the banks then, 10 minutes after he O t O t
starts, he will reach point C lying at a distance S = 120 (s) (s)
m downstream from point B. If the man heads at a
a a
certain angle  to the straight line AB (AB is (m/s2) (m/s2)
(C) (D)
perpendicular to the banks) against the current he will
O t O t
reach point B after 12.5 minutes. Assume the speed of (s) (s)
the boat relative to the water to be constant and of the
same magnitude in both cases. Passage #3 (Q.No21 to 23)
A particle of m = 5 kg is momentally at rest x = 0 at t = 0.
  
16. Width of the river  is – It is acted upon by two forces F1 an F2 . F1 = 70 ˆj N.k

The direction and magnitude of F2 are unknown. The
(A) 100 m (B) 50 m (C) 150 m (D) 200 m

particle experiences a constant acceleration a . In the
direction as shown. Neglect gravity.
17. Velocity of the boat u relative to the water is –
(A) 5 m/min (B) 20 m/min y

(C) 10 m/min (D) 25 m/min F1  70 ĵ N

a1  10m / s 2
18. Speed of the current v is – 53º
(A) 12 m/min (B) 6 m/min x

(C) 20 m/min (D) 15 m/min F2

Passage # 2(Q.No 19&20) 21. Find the missing force F2 ?

The position (x) of a moving particle along straight line (A) (20iˆ  30j)
ˆ N (B) (25iˆ  40j)
ˆ N
depends on time 't' according to given relation
(C) (30iˆ  30j)
ˆ N (D) (30iˆ  20j)
ˆ N
2
x = (t – t + 1), where x is in metre and t is in second. At
any time 't', the velocity (v) of the particle is defined as 22. What is the velocity vector of the particle at t =
the rate of change of its position (x) with time 't'. 10 sec ?
(A) (30iˆ  50j)m
ˆ /s (B) (50iˆ  75j)m
ˆ /s
19. Which of the following options is correct ? (C) (30iˆ  45j)m
ˆ /s (D) (60iˆ  80j)m
ˆ /s
(A) the velocity of the particle at t = 0 s is zero
(B) the acceleration of the particle at t = 1s is 
23. What third force. F3 is required to make the
1 m/s2 acceleration of the particle zero ?
1
(C)the velocity of the particle is zero at t = s (A) (20iˆ  30j)
ˆ N (B) (30iˆ  40j)
ˆ N
2
(D)the position of the particle is 2 m at t = 1s (C) (30iˆ  30j)
ˆ N (D) (30iˆ  20j)
ˆ N

27
PHYSICS-X I-IIT-JEE VECTOR AND BASIC MATHEMATICS

Passage # 4(Q.No24&25) 25. If the acceleration which is produced by these


   three forces, they are acting in the direction
Three forces F1 , F2 and F3 are acting on a particle of shown in the figure. The magnitude of forces F1
mass 10 kg. The particle moves in x-y plane such that and F2 are
y
F1
its coordinates are given by (5t2 m, 15t2 m). F2
120º
30º
24. Find acceleration vector of the particle -
F2 = 100 N
(A) (5iˆ  15ˆj) m / s 2 (B) (10iˆ  30ˆj) m / s 2
(A) 250 N, 450 N (B) 250 N, 500 N
(C) (10iˆ  25ˆj) m / s2 (D) (5iˆ  30ˆj) m / s2 (C) 300 N, 500 N (D) 300 N, 300 N

28
VECTOR AND BASIC MATH EMATICS PHYSICS-X I-IIT-JEE

EXERCISE # 3
Question Column Match Based Questions
3. Column-I contain graph of two f(x) and
1. Column-I contains vector diagram of three
  
Column-II contains difference values
vectors a , b , c & Column-II contains vector
corresponding to graphs. Match the following.
equation. Match them.
Column-I Column-II Column I Column II

 
c b   
(A) (p) a – ( b + c ) = 0 (A) (–5x) (p)0

a x

 Area under graph


c    
(B) b (q) b – c = a Enclosed between

a x and y-axis.


10

a b    f(x)
(C) (r) a + b = –c 1
(B) (q)
5 x  10 15 5
 –10
c
 15
b
     f (x)dx
(D) a (s) a + b = c 0

c
(C) sinx  (r)25
2
2. In column-I condition on velocity, force and x
2
acceleration of a particle is given. Resultant
motion is described in column-II. u =
  sin xdx
0
instantaneous velocity -
10
Column-I Column-II
  (D) (s) 66
(A) u × F = 0 and (p) path will be f(x)
 10 20
F = constant circular path
  17
(B) u . F = 0 and (q) speed will increase  f ( x)dx

F = constant 5

 4. Match thecolumn:
(C) v.F = 0 all the time (r) path will be

and | F | = constant straight line Column - II Column –II
and the particle (A) (sin  + cos)2 (p) 1 – sin 2
always remains in
(B) (sin  – cos)2 (q) 1 + sin 2
one plane
 4
(C) cos  – sin  
(r) cos 2
(D) u = 2 î – 3 ĵ and (s) path will be
acceleration at all parabolic (D) cos4 + sin4 (s) 1 + 2 sin2

time a  6iˆ  9jˆ (t) None

29
PHYSICS-X I-IIT-JEE VECTOR AND BASIC MATHEMATICS
  
5. Match thecolumn: 8. Three forces F1 , F2 and F3 are represented as
Column - II Column –II shown. Each of them is of equal magnitude.
2
Column I Column II
(A)  5 dx (p) 1
(Combination) (Approximate
0
Direction)
2   
(B)  x dx (q) 10 (A) F1  F2  F3 p.
1   
2
(B) F1  F2  F3 q.
(C)  x –1 dx (r) 1.5   
(C) F1  F2  F3 r.
1
   
(D)  e – x dx (s) loge 2 (D) F2  F1  F3 s.
0
(t) None Question Numerical Type Questions
6. Match the information given in Column I and 9. A swimmer jumps from a bridge over a canal and
with Column II. Select the correct option form swims 1 km up stream. After that first km, he
passes a floating cork. He continues swimming
the codes given below.
for half an hour and then turns around and swims
back to the bridge. The swimmer and the cork
Column I Column II reach the bridge at the same time. The swimmer
(A) The component of vector (p) 0 has been swimming at a constant speed. How fast
6iˆ  3jˆ  6kˆ parallel to the does the water in the canal flow in km/hr.
 
10. If a vector r = xiˆ + yjˆ + zkˆ , makes angle ,
vector ˆi  ˆj  kˆ 3 3
(B) The component of vector 
(q) – 3 and with X-axis. Y-axis and Z-axis
n
6iˆ  3jˆ  6kˆ perpendicular
respectively, find the value of n.
to vector ˆi  ˆj  kˆ 11. Four forces are acting on a particle. One forces
(C) The component of vector of magnitude 3 N is directed upward, another is
(r) 7iˆ  2jˆ  5kˆ
directed 37º. East of North having magnitude
6iˆ  3jˆ  6kˆ perpendicular 5N, third is directed in South-West direction is of
to vector 2iˆ  ˆj  2kˆ magnitude 4 2 N. If the particle is in
(D) The component of vector (s) ˆi  ˆj  kˆ equilibrium, then the magnitude of fourth particle
6iˆ  3jˆ  6kˆ parallel to is 5  nN . Find the value of n.
vector 2iˆ  kˆ 12. Find the volume of parallelepiped (in m3) whose
edges are represented by
   
7. If a = ˆi  2jˆ  3kˆ , b = 2iˆ  ˆj  kˆ and a = (2iˆ  3jˆ  4k)ˆ m, b = (iˆ  2jˆ  k)
ˆ m and
 
c = ˆi  3jˆ  2kˆ . Match the Column I with c = (3iˆ  ˆj  2k)
ˆ m.
Column II and select the correct option from the 13. The angle between two vectors
  
given code below. a = 4iˆ  7jˆ  6kˆ and b = 3iˆ  3ˆj  c kˆ is 
10 3

Column I Column II 3c
rad. Find the value of .
(A) | (a × b) × c | 17
p. 35 3
 
(B) | a × (b × c) | q. 20 14. If c = i  (a  i)  ˆj  (a  ˆj)  kˆ  (a  k) .
(C) | a . (b × c) | r. 3 10 If c = na, find the value of n.
(D) |(a × b)(b . c) | s. 5 26

30
VECTOR AND BASIC MATH EMATICS PHYSICS-X I-IIT-JEE

EXERCISE # 4
Question Previous Year (JEE Main) 5. Two forces P and Q of magnitude 2F and 3F,

1. When forces F1, F2, F3 are acting on a particle of respectively, are at an angle with each other. If
the force Q is doubled, then their resultant also
mass m such that F2 and F3 are mutually
gets doubled. Then, the angle is:
perpendicular, then the particle remains stationary.
[JEE Online Main-2019]
If the force F1 is now removed then the acceleration (A) 30º (B) 60º
of the particle is- [AIEEE-2002] (C) 90º (D) 120º
F1 F2 F3
(A) (B)  
m m F1 6. Two vectors A and B have equal magnitudes.
(F2 – F3 ) F2  
(C) (D) The magnitude of (A  B) is 'n' times the
m m  
magnitude of (A  B). The angle between is
     
2. If A  B  B  A , then the angle between A and A and B :[JEE Online Main-2019]
B is – [AIEEE-2004]  n2 1 1  n  1
(A) sin  1   (B) cos  n  1
(A) (B)/3 (C)/2 (D)/4 2  
 n  1 

  n 2  1 1  n  1 
3. A vector A is rotated by a small angle (C) cos  1   (D) sin  n  1
2
  n  1   
radians (<< 1) to get a new vector B . In
 
that case | B  A | is- [JEE Main-2015] 7. In a plane electromagnetic wave, the directions of
 
(A) | B |  | A | (B) 0 electric field and magnetic field are represented
   2  by k̂ and 2iˆ  2ˆj , respectively. What is the unit
(C) | A |  (D) | A |  1  
 2  vector along direction of propagation of the

wave. [Sep. 2020]
1 ˆ ˆ 1 ˆ ˆ
4. In the cube of side 'a' shown in the figure, the (A) (i  j) (B) (i  2j)
vector from the central point of the face ABOD to 2 5
the central point of the face BEFO will be: 1 1 ˆ ˆ
(C) (2iˆ  ˆj) (D) ( j  k)
[JEE Online Main-2019] 5 2
z

B E 8. A force F  (iˆ  2ˆj  3k)
ˆ N acts at a point

A (4iˆ  3jˆ  k)
ˆ m. Then the magnitude of torque
H
G a
about the point (iˆ  2ˆj  k)
ˆ m will be
O F y
x N-m. The value of x is _____. [Sep. 2020]
a
a
x
D  
9. Magnitude of resultant of two vectors P and Q
1
(A) a (iˆ  k)
ˆ 1
(B) a ( ˆj  i)
ˆ 
2 2 is equal to magnitude of P. Find the angle
1 1
  
(C) a (kˆ  ˆi) (D) a (ˆj  k)
ˆ between Q and resultant of 2P and Q .
2 2
[Sep. 2020]

31
PHYSICS-X I-IIT-JEE VECTOR AND BASIC MATHEMATICS
      
10. The angle between vector (A) and (A  B) is: 13. Statement I : Two forces (P  Q) and (P  Q)
[JEE MAIN-2021] where , when act at an angle 1 to each other, the

B magnitude of their resultant is 3 (P2  Q2 ),


120°
A
 when they act at an angle 2, the magnitude of
–B
their resultant becomes 2 (P 2  Q2 ). This is

possible only when 1 < 2.


 B 
    3B  [JEE MAIN -2021]
(A) tan 1  2  (B) tan 1 
 3   2A  B 
AB 
  Statement II : In the situation given above.
 2 
1 = 60° and 2 = 90°.
 B cos   1  A 
(C) tan 1   (D) tan  0.7 B  ln the light of the above statements, choose the
 A  Bsin    
most appropriate answer from the options given
  
11. The magnitude of vectors OA, OB and OC in the below :
given figure are equal. The direction of (A) Both Statement I and Statement II are true.
  
OA  OB  OC with x-axis will be – (B) Statement I is true but Statement II is false.
[JEE MAIN -2021] (C) Statement I is false but Statement II is true.
y
C
(D) Both Statement I and Statement II are false.
A

45° 30°
60°
Question Previous Year (JEE Advanced)
  
14. Three vectors P , Q and R are shown in the
1 3  2  
1  3  1  2  figure. Let S are any point on the vector R . The
(A) tan 1   (B) tan  
1 3  2  1 3  2  
distance between the points P and S is b | R | .
 3 1 2  1  1  3  2    
(C) tan 1   (D) tan   The relation among vectors, P , Q and S is-
1 3  2  1 3  2 
[JEE Advanced-2017]
12. The resultant of these forces Y 
     b| R|
OP, OQ, OR, OS and OT is approximately P
  
_______ N.  R  QP
[Take 3  1.7, 2  1.4. Given unit vectors
P   Q
S Q
along x, y axis] [JEE MAIN -2021]
P
y 20N
T 15N O
30°
X
60° 10N Q
x' 30° x   
45° 45° (A) S  (1  b2 ) P  bQ
15N   
20N
(B) S  (1  b) P  bQ
y'
S R   
(A) 9.25iˆ  5jˆ (B) 3iˆ  15ˆj (C) S  (1  b)P  b2Q
  
(C) 1.5iˆ  15.5ˆj (D) 2.5iˆ  14.5ˆj (D) S  (b  1) P  bQ

32
VECTOR AND BASIC MATH EMATICS PHYSICS-X I-IIT-JEE
 
15. Two vectors A and B are defined as 16. A particle of mass M = 0.2 kg is initially at rest in
  the xy-plane at a point (x = –, y = –h), where  =
A   iˆ and B   (cos t ˆi  sin t ˆj), where is
10 m and h = 1m. The particle is accelerated at time
a constant and  = /6 rad s–1. t = 0 with a constant acceleration a = 10 m/s2 along
   
If | A  B|  3 | A  B | at time t =  for the first the positive x-direction. Its angular momentum and
torque with respect to the origin, in SI units, are
time, the value of , in second, is _______.  
represented by L and , respectively.
[JEE Advanced-2021]
ˆi, ˆj and kˆ are unit vectors along the positive x, y

and z-directions, respectively. If kˆ  ˆi  ˆj then


which of the following statement(s) is(are) correct ?
[JEE Advanced-2021]

(A) The particle arrives at the point (x = , y = –h)


at time t = 2s.

(B)   2kˆ when the particle passes through the
point (x = , y = –h)

(C) L  4kˆ when the particle passes through the
point (x = , y = –h)

(D)   k̂ when the particle passes through the
point (x = 0, y = –h)

33
PHYSICS-X I-IIT-JEE VECTOR AND BASIC MATHEMATICS

ANSWER KEY

EXERCISE-1

Qus. 1 2 3 4 5 6 7 8 9 10 11 12 13 14 15 16 17 18 19 20
Ans. D C D A A A C A B A A A A B D A B C C C
Qus. 21 22 23 24 25 26 27 28 29 30 31 32 33 34 35 36 37 38 39 40
Ans. B A C B A C A C C D D A C D A C C D A B
Qus. 41 42 43 44 45 46 47 48 49 50 51 52 53 54 55 56 57 58 59 60
Ans. C D B A B B B D C C B B C D B C D D C B
Qus. 61 62 63 64 65 66 67 68 69 70 71 72 73 74 75 76 77 78 79 80
Ans. A D C B A D A B C D A B A C B C B B B C
Qus. 81 82 83 84 85 86 87 88 89 90 91 92 93 94 95 96 97 98 99 100
Ans. D A C D B B B D B B B D B A C A C D B C
Qus. 101 102
Ans. B C

EXERCISE-2

Qus. 1 2 3 4 5 6 7 8 9 10 11 12 13 14 15
Ans. A,B A,B C,D A,B,D A,B,D A,B,C A,C A,B A,B,C A,C A,B,D A,C C,D A,C,D A,B,C
Qus. 16 17 18 19 20 21 22 23 24 25
Ans. D B A C A C D B B D

EXERCISE-3

1. (A)(r); (B)(s); (C)(p); (D)(q) 2. (A)(r) ;(B)(q),(s) ; (C)(p) ; (s)(q),(r)


3. (A)(q); (B)(r); (C)(p); (D)(s) 4. (A)(q); (B)(p); (C)(r),(s); (D)(t)
5. (A)(q); (B)(r); (C)(s); (D)(p) 6. (A) q, s ;(B) r ; (C) p; (D) p
7. (A) s ; (B) r ; (C) q; (D) p 8. (A)q ;(B). r ; (C) p; (D) s
9.(6) 10.(4) 11.(2) 12.(7) 13.(3) 14.(2)

EXERCISE-4

1.(A) 2.(A) 3.(C) 4.(B) 5.(D) 6.(C) 7.(A)


8.(195) 9. (90º) 10. (B) 11. (D) 12. (A) 13. (A) 14.(B)
15.(2.00) 16. (ABC)

34
UNIT AND DIMENSION PHYSICS-X I-IIT-JEE

CHAPTER

UNIT& DIMENSION
1. PHYSICAL QUANTITIES (b) Derived quantities:
 The quantities by means of which we describe the The quantities which are derived with the help of
laws of physics are called physical quantities. A fundamental quantities is called derived quantities as
physical quantity is completely specified if it has Distance Length
Speed = 
(a) Magnitude only Time Time
Ratio Here we know that length and time are the
Refractive index, dielectric constant fundamental quantities.
(b) Magnitude and unit
Scalar Example-1
Mass, charge, current 1
The Bernoulli's equation is given by P + dv2 + dgh
(c) Magnitude, unit and direction 2
Vector = constant. The quantity dv2/2 has the same units as
Displacement, torque. that of –
 Physical quantity = Magnitude × unit (A) Force (B) Impulse
Quantities: (C) Strain (D) Pressure
These are of two types – Solution.(D)
(a) Fundamental quantities Pressure (only quantities with identical dimensions can
(b) Derived quantities be added or subtracted)

(a) Fundamental quantities:


Example-2
The quantities which do not depend upon other
Which of the following sets cannot enter into the list
physical quantities are called fundamental
of fundamental quantities in any system of units?
quantities and all other quantities may be expressed
(a) Length, mass and velocity
in terms of the fundamental quantity.
(b) Length, time and velocity
There are of seven fundamental quantities in SI
(c) Mass, time and velocity
system-
(d) Length, time and mass
(i) Mass
Solution.(B)
(ii) Length
The group of fundamental quantities are those quantities
(iii) Time
(iv) Temperature which do not depend upon other physical quantities in
(v) Electric current the group. But in set (b) we can predict the relation
(vi) Luminous intensity between given quantities as length = velocity × time
(vii) Amount of substance Hence set (b)cannot enter in to the list of fundamental
These quantities are also called base quantities.
quantities. Hence correct answer is (b).

35
PHYSICS-X I-IIT-JEE UNIT AND DIMENSION

2. UNITS (iii) F.P.S. – (Foot – Pound – Second) system

 That fixed and definite quantity which we take as


our standard of reference and by which we (iv) S.I. – (System – international) system
measure other quantities of same kind, is defined Following table will show the difference between
unit. There are of two types. all the systems.
(a) Fundamental Units
(b) Derived Units
Quantity C.G.S. M.K.S. F.P.S. S.I.(Symb.)
(a) Fundamental Units: The units which are Mass gm kg pound kg
independent and which are not be derived from Length cm m Foot metre
other units, are defined as fundamental units. e.g.
Time second sec Sec second
The unit of mass, length, and time.
There are seven fundamental units. Temperature kelvin kelvin kelvin (K)
(i) Unit of mass Electric ampere Ampere(A)
(ii) Unit of length
Current
(iii) Unit of time
Luminous candela (cd)
(iv) Unit of temperature
(v) Unit of electric current Intensity
(vi) Unit of luminous intensity Amount of mole(mol)
(vii) Unit of amount of substance substance

(b) Standard Units: The fixed and definite real value


of any physical quantity is defined as standard unit. Some definitions of fundamental units in S.I. system :
(a) Metre: One metre is that distance which
 Properties of Units:
accommodates 1, 650, 763.73 waves in vacuum of
The unit of a physical quantity is inversely
1
orange red light emitted by Kr86.
proportional to its numerical value i.e., u 
n
where u and n are the units of physical quantity and (b) Kilogram:
its numerical value respectively. Relation between
(i) Kilogram is the mass of platinum iridium cylinder
unit and its numerical value n1 u1 = n2 u2
of diameter equal to its height which is preserved in

 Selection Criteria of Units: a vault at international Bureau of weights and


(i) Selected unit must be universal, of proper size measures at Severs near Paris.
and magnitude (ii) 1 kilogram is the mass of 1 lit. of water at 4ºC
(ii) Unit must be not affected by temperature,
pressure and time.
(iii) 1 kilogram is the mass of 5 × 1025 atoms
(iii) Easily definable and reproducible.
of C12
 System Of Units Used: (c) Second: One second is that amount of time in
These are of Four types – which a cesium-33 atom makes 9, 192, 631, 770
(i) C.G.S – (Centimeter – Gram – Second) system.
vibration in a cesium watch.
(ii) M.K.S. – (Metre – Kilogram – Second) system

36
UNIT AND DIMENSION PHYSICS-X I-IIT-JEE

(d) Ampere : One amp is that electric current which Example-4


when passed through two parallel conductors of A cheep wrist watch loses time at the rate of 8.5
infinite length and of negligible cross section placed at
second a day. How much time will the watch be off
unit distance apart produce a force of 2 × 10–7 N/m.
(e) Kelvin : at the end of a month ?
th (A) 4.25 min/month (B) 5 min/month
 1 
(i) 1 Kelvin is equal to the   part of the (C) 3.25 min/month (D) 4.10 min/month
 100  Solution.(A)
difference of melting point of ice and boiling
Time delay= 8.5 s/day
point of water.
= 8.5 × 30 s/ (30 day)
1
(ii) 1 Kelvin is equal to th part of the = 255 s/month
273.16
thermodynamic temperature of triple point of = 4.25 min/month.
water (273.16 K)
Example-5
(f) Candela : It is the luminous intensity in a direction
at right angles to surface of a black body at a If the units of force, energy and velocity in new
temperature equal to melting point of platinum system be 10N, 5J and 0.5 ms–1 respectively, find the
under pressure of 101, 325 N/m2. units of mass, length and time in that system.
(g) Mole : Solution.
(i) Mole is that amount of substance contains as Let M1, L1 and T1 be the units of mass, length and time
many elementary entities as there are oxygen
in SI and M2, L2 and T2 the corresponding units in new
atoms (8O16) in 0.016 kg of oxygen (old def.).
system.
(ii) It is that amount of substance which contains as
many elementary entities as there are carbon The dimensional formula for force is (M1L1T–2)
atoms 6C12 in 0.012 kg of carbon. Hence the conversion formula for force becomes
1 1 2
 Supplementary Unit : M  L  T 
n2 = n1  1   1   1 
(a) Radian : 1 radian is the angle subtended by arc  M2   L2   T2 
of length equal to the radius, at the centre of the Here n1 = 10 N, n2 = 1, Substituting we get
circle. 2 2
M   L1   T1 
(b) steradian : it is defined as the solid angle 1 = 10  1      ......(i)
subtended at the centre of the sphere by an arc of its  M2   L2   T2 
surface equal to the square of radius of the sphere. The dimensional formula for work is (M1 L2 T–2)
A 1 2 2
Solid angle  = M  L  T 
R2 n2 = n1  1   1   1 
WhenA = R2  M 2   L 2   T2 
= 1 steradian n1 = 5 J, n2 = 1
Substituting values we get
Example-3 1 2 2
M  L  T 
The acceleration due to gravity is 9.80 m/s2. What is 1=5  1  1  1 .......(ii)
its value in ft/s2
?  M 2   L 2   T2 
(A) 30.15 ft/s2 (B) 32.14 ft/s2 Similarly dimensional formula for velocity is(M0 L1 T–1).
(C) 28.23 ft/s2 (D) None of these Hence, conversion formula for velocity is
Solution.(B) 0 1 1
M  L  T 
Because 1 m = 3.28 ft, therefore n2= n1  1   1   1 
 M 2   L 2   T2 
9.80 m/s2 = 9.80 × 3.28 ft/s2= 32.14 ft/s2

37
PHYSICS-X I-IIT-JEE UNIT AND DIMENSION

Here n1 = 0.5 ms–1 , n2 = 1, (c) Similar dimension can be added or subtracted but it
Substituting values we get does not change the dimensions.
1 1
L   T1  (d) For a physical equation to be correct dimensionally
1 = 0.5  1    ........(iii)
 L2   T2  the dimension of all terms on two sides of the
equation must be same. This is known as the
1  L1 
Dividing (B) by (A), 1 =   , principle of homogeneity of dimensions.
2  L2 
L1 1 (e) Logarithmic functions as log x, ex is the dimension
L2 =  m = 0.5 m
2 2 less quantity.
L  (f) Powers are dimension less.
Substituting value of  1  in (iii), we get
 L2 
(g) If we put the value of any physical quantity in any
1
T  formula it seems unbalanced but reality is that it is
1 = 0.5 × 2  1  ,
 T2  balanced formula. Only appearance is unbalanced
T1 as :
= 1, T2 = 1s
T2 a
Sn = u + (2n – 1)
L   T1  2
Substituting value of  1  and   in (A) (h) The dimensions of two physical quantities may be
 L2   T2 
same but the quantities need not be similar.
M 
1 = 10  1  × 2 × 1, (i) Remember the following dimensional formula-
 M2 
Force = [M1L1T–2]
M 
1= 20  1  , Energy = [M1L2T–2]
 M2 
M2 = 20 M1 as M1 = 1kg, M2 = 20 kg.  Uses of Dimension :
Hence units of mass, length and time are 20 kg, The uses of dimension are as given below.
0.5 m and 1 sec respectively i. Homogeneity of dimensions in equation.
ii. Conversion of units
3. DIMENSIONS iii. Deducing relation among the physical
 Dimensions of a physical quantity are the powers to
quantities.
which the fundamental units of mass, length, time
etc. must be raised in order to represent that i. Homogeneity of Dimensions in Equation :
physical quantity. The dimensions of all the terms in an equation must
Dimensional formula = [Ma Lb Tc Qd] where a, b, c,
be identical. This simple principle is called the
d are the dimensions of M, L, T, Q respectively.
principle of homogeneity of dimensions. This is the

very useful method whether an equation may be
 Some Points About Dimensions:
(a) The dimensions of a physical quantity do not correct or not. If the dimensions of all the terms are
depend upon system of units to represent that not same the equation must be wrong. Let us check
physical quantity. the equation.

1 2
(b) Pure numbers and pure ratio do not have any x = ut + at
2
dimensions. i.e. these are dimension less, e.g.
[x] = L
refractive index, relative density, relative
[ut] = velocity × time
permeability, cos , , strain etc.

38
UNIT AND DIMENSION PHYSICS-X I-IIT-JEE

length iii. D e d uc i ng R e lat i o n a mo ng t he p hy s i c a l


= × time = L
time Quantities:Sometimes dimensions can be used to
deduce a relation between the physical quantities. If
1 2 2 2 one knows the quantities on which a particular
 2 at  = [at ] = acceleration × (time)
  physical quantity depends and if one is given that
this dependence is of product type. Method of
velocity length / time
= × (time)2= × time2 = L dimension may be helpful to derive the relation.
time time
Thus the equation is correct as far as the dimensions Example-7
Derive the expression for the time period of a simple
are concerned. The equation x = ut + at2 is also pendulum which depends on the length, mass, and
dimensionally correct although this is an incorrect gravitational acceleration.
equation. So a dimensionally correct equation need Solution.
t la mb gct = kla mb gc
not be physically correct but a dimensionally wrong
where k is a dimensionless constant and a, b and c are
equation must be wrong. exponents which we want to evaluate. Taking the
dimensions of both sides,
ii. Conversion of Units : T = La Mb (LT–2)c = La + c Mb T–2c
When we choose to work with a different set of Since the dimensions on both side must be identical we
have
units for the base quantities, the units of all the
a+c=0
derived quantities must be changed. Dimensions b = 0 and–2c = 1
can be useful in finding the conversion factor for 1
Giving a = ,b=0
2
the unit of a derived physical quantity from one
1
and c = –
system to other. 2

Putting this values in the equation t = k
Example-6 g
Convert the 1 pascal in to C.G.S. system  Limitations of the dimensional method
(A)10 CGS (B) 12 CGS (a) First of all we have to know the quantities on which
a particular physical quantity depends.
(C) 9 CGS (D) 11 CGS
(b) Method works only if the dependence is of the
Solution.(A) 1 2
F product type (Not applicable for x = ut + at )
P= 2
A
[F] MLT 2 (c) Numerical constants having no dimensions cannot
Thus, [P] =  2
= ML–1T–2 be deduce by the method of dimensions.
[A] L
So, 1 pascal = (1kg) (1m)–1 (1s)–2and (d) Method works only if there are as many equations
1 C.G.S. pressure = (1g) (1 cm)–1 (1s)–2 available as there are unknowns.
1pascal
Thus Example-8
1 CGS pressure
1 2
A gas bubble from an explosion under water
 1kg   1m   1s  oscillates with a period T proportional to pa db Ec
=    
 1gm   1cm   1s  where p is the static pressure, d is the density of
water and E is the total energy of explosion. Find the
= (103) (102)–1 = 10 values of a, b and c.
or 1 pascal = 10 CGS pressure 6 1 1
(A)a = ,b= ,c=–
One can work out the conversion factor for any derived 5 2 3
quantity if the dimensional formula of the derived 5 1 1
(B)a = ,b= ,c=
quantity is known. 6 2 3

39
PHYSICS-X I-IIT-JEE UNIT AND DIMENSION

1 5 1  Application of dimensional analysis:


(C)a = ,b= ,c=
2 6 3 1. IN MECHANICS
(D)None of these (a) Write the definition or formula for the physical
Solution.(B) quantity.
As T  pa db Ec = k pa db Ec (a) Replace M, L and T by the fundamental units of the
k is dimensions on both sides required system to get the unit of physical quantity
M0L0T1 = (M1L–1T–2)a (M1L–3)b (M1 L2 T–2)c [a] Gravitational constant G:
=Ma + b + c L–a – 3b + 2cT – 2a – 2c I. Approach
Applying principle of homogeneity of dimensions From Newton's law of gravitation we have
5 1 1 m1 m 2 Fr 2
 a= ,b= ,c= F=G G =
6 2 3 r 2
m1m2
Example-9 [MLT 2 ][L2 ]
[G] =
The velocity v of water waves depends upon their [M] [M]
wavelength , density of water  and acceleration m3 Nm 2
due to gravity, 'g'. Deduce by the method of So its SI units is or
kgs2 kg 2
dimensions the relation between these quantities.
Solution. II. Approach
Let the velocity v of water waves be given by From the relation between G and g we have
2
v ab gc g=
Gm
G=
gR
2
v = kab gc ...... (A), R m
where k is dimensionless constant of proportionality Substituting the dimensions of all physical
and a, b, c are powers to be determined using principle [LT 2 ][L2 ]
quantities[G] =
of homogeneity of dimensions. [M]
Taking dimensions on both sides of equation (A), we
m3 Nm 2
have So its SI units is or
M0 L1 T–1 = [M0 L1 T0]a [M1 L–3 T0]b kgs2 kg 2
[M0 L1 T–2]c = Mb La – 3b + cT–2c [b] Plank constant h :
Comparing dimensions of mass, length and time, we get I. Approach
1 1 According to constant h :
b = 0, c = ,a=
2 2 E
E = h  h =
1/2 0
v = k  g1/2, , Thus v   g 
Substituting the dimensions of known physical
Example-10 quantities :
If force F, length , current I and time T are taken as [ML2 T 2 ]
[h] = 1
= M1L2T–1
T
fundamental base dimensions, then the dimension of
II. Approach
permittivity 0 are –
De Broglie
(A) F L2 T2 I–2 (B) F–1 L2 T2 I2 h
(C) F–1 L–2 T2 I2 (D) F2 L2 T2 I2 =
mv
Solution.(C) h = mv
1 q1 q 2
Since F = Substituting the dimensions of known physical
4  0 r 2 quantities :
[q1  q 2 ] (IT) (IT) [h] = [L] [M] [LT –1]
we have [0] = 2
 = F–1 L–2 T2 I2
[F][r ] FL2 [h] = [ML2 T –1]
The correct answer is (c) [h] = [ML2 T –1]

40
UNIT AND DIMENSION PHYSICS-X I-IIT-JEE

III. Approach 2. IN HEAT:


Bohr's II Postulate [a] Temperature : In heat it is assumed to be a
nh fundamental quantity with dimensions [] and
mvr =
2 unit Kelvin [K]. [How ever, K = C°].
2 [b] Heat : It is energy so it dimensions are
h= × mvr [ML2T–2] and SI Units Joule (J). Practical unit
n
Substituting the dimensions of known physical of heat is calorie (cal.) and
1 calorie = 4.18 joule
quantities :
[c] Coefficient of Linear Expansion a :
[h] = [mvr] L
as 2 and n are dimensionless. It is defined as =
L
So SI unit of plank's constant is kg m2/s. [L]
Which can also be written as i.e.[] =
[L][]
(kg m2/s2) × s. But as kg m2/s2 is Joule so unit
i.e.[] = [–1]
of h is Joule × sec. i.e. J-s so its unit is (C°)–1 or K–1
[c] Coefficient of Viscosity  [d] Specific Heat C
As= mC 
I. Approach

F SoC =
According to Newton's law= m 
 dv 
A  [ML2 T 2 ]
 dy  i.e.[C] =
[M][]
Substituting the dimensional formulae of all [C] = [L2 T–2 –1]
other known physical quantities. So its S unit be J/kg-K while practical unit
[MLT2 ] cal/gm-C°
[] = [e] Latent Heat L
[L2 ][LT2 / L]
By definitionQ = ML
II. Approach i.e. [L] = [ML2 T–2] / [M]
Stoke's lawF = 6rv   [L] = [L2 T–2]
F So its S unit be J/kg while practical unit
=
6 rv cal/gm
Substituting the dimensional formulae of all [f] Coefficient of Thermal conductivity K
other known physical quantities. According to law of thermal conductivity
dQ d
[MLT2 ] = KA
[] = [] = [ML–1 T–1] dt dx
[L][LT1 ]
[ML2 T 2 / T]
III. Approach Or[K] = 
[L2 ][ / L]
Poiseuille's formula
   [K] = [MLT–3–1]
dV pr 4 Its SI unit is W/mK while practical unit

dt 8 is cal/s – cm-C°.
pr 4 [g] Mechanical Equivalent of Heat J :
  = According to I law of thermodynamics
 dV 
8   W = JH
 dt  W
  J= 
Substituting the dimensional formulae of all H
other known physical quantities. [ML2T2 ]
1 2 4     [J] =
[ML T ][L ] [ML2T2 ]
[] =
[L][L3T] i.e.[J] = [M0 L0 T0]
So SI or MKS unit of coefficient of viscosity is i.e. J has no dimensions. Its practical unit is
kg/m-s or (g/cm-s called poise in C.G.S. system) J/cal and has value 4.18J/cal.

41
PHYSICS-X I-IIT-JEE UNIT AND DIMENSION
[h] Boltzmann constant K : According to kinetic (b) Charge Q
theory of gases, energy of a gas molecule is Q
AsI =
given by t
3 So[Q] = [I] [t] [Q] = [At]
E= KT The SI unit of charge is A × s and is called coulomb (c).
2
[ML2 T 2 ] Note :
i .e.[K] =
[] (i) In MKSQ system charge is assumed to be
fundamental quantity with dimension [Q] and unit
i.e.[K] = [ML2T–2 –1]
coulomb. So in this system current will be derived
J J with dimension [QT–1] and units coulomb / s which
So its SI unit is and value 1.38 × 10–23 .
K K is ampere.
[i] Gas constant R: (ii) In CGS system there are two units of charge namely
According to gas equation for perfect gas esu of charge frankline (Fr) and emu of charge. It is
PV = RT found that
1 coulomb = 3 × 109 esu of charge
[ML1T 2 ] [L3 ]
i.e.[R] = 1
[ ][] =   emu of charge.
 10 
i.e.[R] = [ML2 T–2–1 –1] (c) Electric potential V :
So its SI unit is J/mol-K. While practical unit is W
It is defined as V =
cal/mol-K. It is a universal constant with value q
8.31 J/mol-K or 2 cal/mol-K.
[ML2 T 2 ]
So [V] =
[AT]
[j] Vander Waal’s constants a and b :
i.e.[V] = [ML2 T–3 A–1]
Vander Waal's equation So SI unit of potential is J/C and is called
 a  volt (V)
 P  2  (V – b) = RT ….(a)
 V 
(d) Electric intensity E:
Vander waal’s equation for  mol is – It is defined as
  2a '  E=
F
 P  2  [V – b' ] = RT ....(b)
q
 V 
compare eqn (a) and (b) [MLT 2 ]
So [E] = 
[AT]
2 a' = aand b' = b
 [E] = [MLT–3 A–1]
[a]
[a'] = 2  So SI unit of electric intensity is
[ ] Newton Nm

[b] Coulomb cm
And[b'] = [a'] = [mL5 T–2–2] J 
[ ] [N mJ] J  C V
  V
   
and [b'] = [L3–1] cm m

J  m3 m3 (e) Capacitance C
Unit of a ' and b' are and
mol 2 mol It is defined as
respectively. q = CV
q q2 W
i.e.C = = [as V = ]
3. In Electricity : V W q
(a) Current I : While dealing electricity we assume [AT]2
 [C] = = [M–1 L–2 T4 A2]
current to be a fundamental quantity and represent [ML2T2 ]
it by [A] with unit ampere (a) and its unit coulomb / volt is called farad.

42
UNIT AND DIMENSION PHYSICS-X I-IIT-JEE

Note : [ML2 T 2 / AT][T]


as W = qV joule / coulomb is volt  V i.e.[L] =
[A]
q = CV coulomb / volt is farad  F
= [ML2 T–2 A–2]
(f) Permittivity of free space 0:
volt  s
According to coulomb's law and its unit  ohm × s is called
1 q1 q 2 amp
F= 
4  0 r 2 Henry (H).
[q]2 [A2T 2 ] (k) Magnetic flux :
  [0] = 
[F][r]2 [MLT2 ][L2 ] According to faraday's law of electro-magnetic
 [0] = [M–1 L–3 T4 A2] induction
d
Coul Coul 2 E.M.F.=
and its unit is  [as N – m = J]  dt
N  m2 J  m W
Coul  Joule  So [] = [EMF] [T]=   [T] 
   as  volt   q
volt  m  coul 
 2 –2
[] = [ML T / AT] [T]
F  coul 
   as volt  farad  = [ML2 T–2 A–1]
m   and its unit will be volt× s known as Weber (Wb.)
(g) Dielectric constant K or Relative Permittivityr:
As K r = (/0)so it has no units anddimensions. (l) Magnetic Induction B :
(h) Resistance R: As force on a current element in a magnetic
According to ohm's law V = IR  field is given by F = Bi sin 
V W W [F] [MLT 2 ]
 R= = [as V = ]  [B] =  = [MT–2 A–1]and unit
I qI q [i][] [A][L]
[ML2 T 2 ] N J CV volt  s
[R] = = [ML2 T–3 A–2]   
[AT][A] Am A  m2 A  m2 m2 
And its unit volt / ampere is called ohm (). weber
  and is called tesla (T).
m2
(i) Resistivity or specific Resistance  :
 Note:
AsR = 2
r As  = BA cos 
r 2 R [ ] [ML2 T 2 A 1 ]
Or  =  so [B] = =
 [s] [L2 ]
[R][L2 ] Weber
 [] =  [RL]  [ML3 T–3 A–2] [MT–2 A–1] and unit
[L] m2
And its unit is ohm-m or ohm-cm. (m) Magnetic Intensity H :
(j) Coefficient of self induction L : For Biot-savart law:
According to definition of  0 Id  sin 
dB = B = 0 H 
Ldi [W / q][T] 4 r2
L EMF= Or[L] =
Id
dt [i]   [H] =  2  = [AL–1] and unit A/m.
r 

43
PHYSICS-X I-IIT-JEE UNIT AND DIMENSION

Physical Quantity Dimensions Units Relation


SI CGS
2 –2 –1 18
Mag. Flux  [ML T A ] Weber Maxwell 1wb = 10 Mx
–2 –1 2 4
Mag. Induction B [MT A ] Weber/m or tesla Gauss 1T = 10 G
–1
Mag. Intensity H [AL ] Ampere/m Orested A –3
1 = 4 × 10 Oe
m
A –3
1 = 4 × 10 Oe
m
3
= 4 × 10 G
= 4 × 10–7 t = 0 T

(n) Magnetic Dipole moment M: Solution.(C)


AsM = Nis b
[LT–1] = a [T] +
 [M] = [AL2] C[T]  M 0 L1T 1
and so unit A – m2. a = L1 T–2
b = L2 T–1
c = L1 T–2
(o) Permeability of free space 0: Hence correct answer is (C).
Force per unit length between two parallel current Note:
carrying wires is given by: It is difficult to remember dimensions of all the
dF  0 I1I 2 quantities so if we want to find out dimensions of
  any quantity we should use the formula in which
dL 4  r
whatever the quantities we are using, that should be
F
 [0] =  2   go towards fundamental quantities.
I  Example-13
 MLT 2  Find out the dimension of L (Inductance)-
 [0] =  2  (A) M1 L2 T–2 A–2 (B) M1 L3 T–2 A–2
 A  (C) M1 L2 T–3 A–2 (D) M1 L2 T–2 A–1
= [ML T–2 A–2] Solution.(A)
and its units N 2 A
We know that L =
N J Volt  s ohm  s H 
 2   
A2 A m Am m m But if we do not remember the dimension of 
Example-11  then it will be difficult for us to determine the
dimension of L.
a, b are two different physical quantities with Therefore we will use
different dimensions which one of the following is 1 2 2U
correct ? U= LI  L = 2
2 I
(A) a + b (B) a – b (C) a/b (D) ea/b (WhereU: energy of inductor)
Solution.(C)
 U = M1 L2 T–2
Example-12
[M1L2 T2 ]
b [Ma Lb Tc Ad] =
v = at + Find out the value of a, b, c- [A2 ]
ct  v
(A) L2 T–1, L2 T–1, L1 T–2  [Ma Lb Tc Ad] = [M1 L2 T–2 A–2]
(B) L1 T–2, L1 T–1, L1 T–2
 a = 1, b = 2, c = – 2, d = – 2.
(C) L1 T–2, L2 T–1, L1 T–2
(D) L1 T–2, L2 T–1, L1 T–3 Hence correct answer is (A)

44
UNIT AND DIMENSION PHYSICS-X I-IIT-JEE

Note: Example-17
Similarly find out the dimension of R & C Evaluate 24.36 + 0.0623 + 256.2
Work
FromP = I2R = Power = (A) 280.7 (B) 245.5
Time
(C) 275.2 (D) 266.3
1
and U = CV2 = Energy. Solution.(A)
2
24.36
Example-14 0.0623
To convert the physical quantity of one unit to 256.2
physical quantity of another unit. Now the first column where a doubtful digit occurs is
Solution. the one just next to the decimal point (256.2). All digits
1 force of M.K.S. = x force C.G.S. right to this column must be dropped after proper
 1 [M1 L1 T–2] M.K.S rounding. The table is rewritten and added below
= x [M1 L1 T–2] C.G.S
24.4
 (kg)1 (m)1 (sec)2 = x (gm)1 (cm)1 (sec)–2
0.1
 (1000)1 (gm) (100)1 (cm) (sec)–2
256.2
= x (gm)1 (cm)1 (sec)–2
x = 105 ——–
So 1 force of M.K.S. = 105 force C.G.S. 280.7 The sum is 280.7

4. ORDER OF MAGNITUDES 5. FRACTIONAL & PERCENTAGE ERRORS


(A) Normally decimal is used after first digit using
 If x is the error in measurement x, then
powers of ten,
Example: 3750 m will be written as 3.750 × 103m x
Fractional error =
x
(B) The order of a physical quantity is expressed in x
Percentage error = × 100
power of 10 and is taken to be 1 if  (10)1/2 = 3.16 x
and 10 if > 3.16 Percentage error in experimental measurement
Example: speed of light = 3 × 108, order = 108 experimental value ~ S tandard value
=
Mass of electron = 9.1 × 10–31, order = 10–30 S tandard value
Propagation Of Error (Addition And
(C) Significant digits : In a multiplication or division
of two or more quantities, the number of significant Subtraction) :
digits in the answer is equal to the number of Let error in x is ± x, and error in y is ± y, then
significant digits in the quantity which has the the error in x + y or x – y is ± (x + y). The errors
minimum number of significant digit. add.
Example : 12.0/7.0 will have two significant digits
only.
 Multiplication And Division :
(D) The insignificant digits are dropped from the result Let errors in x, y, z are respectively ± x, ± y and
if they appear after the decimal point. They are ± z. Then error in a quantity f (defined as)
replaced by zeroes if they appear to the left of two
xa yb
decimal point. The least significant digit is rounded f= is obtained from the relation
according to the rules given below. zc
Z f x y z
=|a| +|b| +|c| .
Thus, we write. f x y z
25.2  1374 The fraction errors (with proper multiples of
= 1040.
33.3 exponents) add. The error in f is ± f.

45
PHYSICS-X I-IIT-JEE UNIT AND DIMENSION

 Important Points: (i) Vernier Callipers


(A) When two quantities are added or subtracted the A vernier callipers provides with an auxiliary (or
absolute error in the find is the sum of the absolute vernier) scale in addition to the main scale. The
error in the quantities. vernier scale can slide along the main scale. The
vernier scale is so graduated (or marked) that the
(B) When two quantities any multiplied or divided, the
length of total number of divisions on it is smaller
fractional error in the result is the sum of the
by length of one division on main scale.
fractional error in the quantities to be multiplied or
The least count of vernier scale is calculated by
to be divided.
using the following formula
(C) If the same quantity x is multiplied together n times Least count of vernier scale (or vernier constant)
(i.e. xn), then the fractional error in xn is n times the value of 1 main scale division
=
fractional error in x, Total number of division on vernier scale n
Or
x
i.e. ± n Least count (vernier constant) = 1 M.S.D.
x (Main scale division) – 1 V.S.D.(vernier scale
Example-18 division).
In an experiment to determine acceleration due to
Example-19
gravity by simple pendulum, a student commit
If N division of vernier coincides with (N – 1)
positive error in the measurement of length and 3%
division of main scale. Given one main scale division
negative error in the measurement of time period. is equal to ‘a unit’, find the least count of the
The percentage error in the value of g will be- vernier.
(A) 7% (B) 10% Solution:
(C) 4% (D) 3% Vernier constant = 1MSD – 1VSD
N 1 a
Solution: (A) =  1   MSD = ,
 N  N

We know T = k Generally, the value of 1 main scale division on vernier
g
calipers is 0.1 cm and there are 10 divisions on the
 vernier scale, i.e., x = 0.1 cm and n = 10.
 T2 = k'   0.1 cm
g  Least count of vernier calipers = = 0.01 cm.
10
 
g = k'
T2  Zero error of vernier calipers:
g  2 T If the zero marking of main scale and vernier
× 100 = × 100 + × 100
g  T calipers does not coincide, necessary correction has
= 1% + 2 × 3% = 7% to be made for this error which is known as zero
error of the instrument. If the zero error of the
6. EXPERIMENTS BASED ON VERNIER vernier scale is to the right of the zero of the main
CALIPERS & SCREW GAUGE scale the zero error is said to be positive & the
 A meter scale can measure accurately up to one– correction will be negative otherwise vice versa.
tenth part of one cm. Its least measurement 0.1 cm,
(ii) Screw gauge:
is called least count of scale. There is limitation of
meter scale that the meter scale cannot measure the Least count
value less than 0.1 cm. For greater accuracy Pitch
=
measurement we have devices such as, Total number of division n on the circular scale

46
UNIT AND DIMENSION PHYSICS-X I-IIT-JEE

 Zero error of screw gauge: Example-21


In a perfect instrument the zeros of the main scale What will be the measurement of following screw
gauge position?
and circular scale coincides with each other, in this
condition screw gauge has zero-error, otherwise the 0 5 25
20
instrument is said to have zero-error which is equal 15
10
to the cap reading with the gap closed. This error is
Solution:
positive when zero line or reference line of the cap Reading = Main scale reading + Number of circular
lies above the line of graduation and corresponding scale division (or screw gauge reading) least count
corrections will be just opposite otherwise vice- 5.5mm + 16 0.01mm = 5.66 mm
versa.

Example-20
What will be the measurement of following screw
gauge position?

0 55
0
45
40
Solution:
Reading = Main scale reading + Number of circular
scale division (or screw gauge reading)least count
3mm + 45 0.01mm = 3.45 mm

47
PHYSICS -X I-IIT-JEE UNIT AND DIMENSION

EXERCISE # 1
Based On Units
1. Which is the correct unit for measuring nuclear 8. To determine the Young's modulus of a wire, the
F L
radii ? formula is Y  : ; where L = length, A
A L
(A)Micron (B) millimetre
area of cross-section of the wire, L  change in
(C)Angstrom (D) Fermi
length of the wire when stretched with a force F .
2. Which of the following is not a unit of time ? The conversion factor to change it from CGS to
MKS system is
(A) microsecond (B) leap year (A)1 (B)10 (C) 0.1 (D) 0.01
(C) lunar month (D) light year
9. One yard in SI units is equal
3. The magnitude of any physical quantity (A)1.9144 metre (B) 0.9144 metre
(A) Depends on the method of measurement (C)0.09144 kilometre (D) 1.0936 kilometre
(B) Does not depend on the method of 10. Match List-I with List-II and select the correct
measurement answer using the codes given below the lists
(C) Is more in SI system than in CGS system List-I List-II
I. Joule A.Henry  Amp/sec
(D) Directly proportional to the fundamental II. Watt B.Farad  Volt
units of mass, length and time III. Volt C.Coulomb  Volt
IV. Coulomb D.Oersted  cm
4. One second is equal to E.Amp  Gauss
F. Amp 2  Ohm
(A)1650763.73 time periods of Kr clock
Codes:
(B) 652189.63 time periods of Kr clock (A) I  A, II  F, III  E, IV  D
(C)1650763.73 time periods of Cs clock (B) I  C, II  F, III  A, IV  B
(D) 9192631770 time periods of Cs clock (C) I  C, II  F, III  A, IV  E
(D) I  B, II  F, III  A, IV  C
5. Density of wood is 0.5gm / cc in the CGS system
11. If x  at  bt 2 , where x is the distance travelled
of units. The corresponding value in MKS units is by the body in kilometers while t is the time in
(A)500 (B)5 (C) 0.5 (D) 5000 seconds, then the units of b are
(A) km/s (B) kms (C) km/s2 (D) kms2
6. The velocity of a particle depends upon as
a 
v  a  bt  ct 2 ; if the velocity is in m / sec , the 12. The equation  P   (v  b) constant. The
unit of a will be  v2 
units of a are
(A) m / sec (B) m / sec 2
(A) Dyne  cm5 (B) Dyne  cm 4
2 3
(C) m / sec (B) m / sec
(C) Dyne/cm3 (D) Dyne / cm2

7. If u 1 and u 2 are the units selected in two 13. Match List-I with List-II and select the correct
answer by using the codes given below the lists
systems of measurement and n 1 and n 2 their
List-I
numerical values, then (A)Distance between earth and stars
(A) n1 u1  n 2 u 2 (B) n1 u1  n 2 u 2  0 (B)Inter-atomic distance in a solid
(C)Size of the nucleus
(C) n1 n 2  u1u 2 (D) (n1  u1 )  (n 2  u 2 ) (D) Wavelength of infrared laser

48
UNIT AND DIMENSION PHYSICS-X I-IIT-JEE

List-II 19. There are two different quantities A and B having


1. Microns different dimensions. Then which of the
2. Angstroms
following operation is dimensionally correct ?
3. Light years
4. Fermi (A) A + B (B) A – B (C) A/B (D) eA/B
5. Kilometres
Codes 20. A wave is represented by
a b c d y = a sin (At – Bx + C)
(A) 5 4 2 1 where A, B, C are constants and t is in seconds &
(B) 3 2 4 1
(C) 5 2 4 3 x is in metre. The Dimensions of A, B, C are-
(D) 3 4 1 2 (A) T–1, L, M0L0T0 (B) T–1, L–1, M0L0T0
(C) T, L, M (D) T–1, L–1, M–1
14. In C.G.S. system the magnitude of the force is
100 dynes. In another system where the 21. A dimensionless quantity -
fundamental physical quantities are kilogram, (A) never has a unit (B) always has a unit
metre and minute, the magnitude of the force is (C) may have a unit (D) does not exist
(A)0.036 (B)0.36 (C) 3.6 (D) 36
p
15. A physical quantity is measured and its value is 22. If v = , then the dimensions of  are (p is

found to be nu where n  numerical value and
pressure, is density and v is speed of sound has
u  unit. Then which of the following relations
their usual dimension) -
is true
(A) M0L0T0 (B) M0L0T–1
(A) n  u 2 (B) n  u 1
(C) M L T0 0 (D) M0L1T0
1
(C) n  u (D) n 
u 23. If energy (E), velocity (V) and force (F), be taken
as fundamental quantities, then what are the
16. How many wavelength of Kr 86 are there in one dimensions of mass -
metre (A) EV2 (B) EV–2 (C) FV–1 (D) FV–2
(A)1553164.13 (B)1650763.73 1 2
24. The formula S = ut – at where S is the
(C) 652189.63 (D) 2348123.73 3
distance travelled, u is the initial velocity, a is the
Based On Dimensions acceleration and t is the time is -
(A)Dimensionally correct only
17. A quantity X is defined as the ratio of the angular (B)Dimensionally incorrect only
to linear momentum of an object. Then the (C)Dimensionally and numerically correct
dimensions of X are - (D)Dimensionally and numerically wrong
(A) M0L1T0 (B) M1L1T1
(C) M1L2T–2 (D) M0L–1T0 25. If L and R are respectively the inductance and
L
resistance, then the dimensions of will be
18. The dimensional formula of angular velocity R

 v (A) M 0 L0 T 1
    is- (B) M 0 LT 0
 r
(A) M0 L0 T–1 (B) M L T–1 (C) M 0 L0T
(C) M0 L0 T1 (D) M L0 T–2 (D) Cannot be represented in terms of M, L &T

49
PHYSICS-X I-IIT-JEE UNIT AND DIMENSION

26. Which pair has the same dimensions? 32. A spherical body of mass m and radius r is
(A) Work and power allowed to fall in a medium of viscosity  . The
(B) Density and relative density time in which the velocity of the body increases
(C) Momentum and impulse from zero to 0.63 times the terminal velocity (v)
(D) Stress and strain is called time constant ( ) . Dimensionally  can
be represented by
27. If C and R represent capacitance and resistance
mr 2  6 mr 
respectively, then the dimensions of RC are (A) (B)  2 
6  g 
(A) M0 L0 T2 (B) M 0 L0T m
(C) (D) None of the above
(C) ML1 (D) None of the above 6rv

28. Dimensions of one or more pairs are same. 33. The frequency of vibration f of a mass m
Identify the pairs suspended from a spring of spring constant K is
(A) Torque and work
given by a relation of this type f  Cm x K y ;
(B) Angular momentum and work
where C is a dimensionless quantity. The value
(C) Energy and Young's modulus
of x and y are
(D) None of these
1 1 1 1
(A) x  ,y  (B) x   , y  
29. The equation of state of some gases can be 2 2 2 2
a 1 1 1 1
expressed as  P  
 (V  b)  RT . Here P is (C) x  , y   (D) x   , y 
 V2  2 2 2 2

the pressure, V is the volume, T is the absolute 34. The quantities A and B are related by the
temperature and a, b, R are constants. The relation, m  A / B , where m is the linear
dimensions of 'a ' are density and A is the force. The dimensions of
B are of
(A) ML5 T 2 (B) ML1T 2 (A) Pressure (B) Work
(C) M 0 L3T 0 (D) M0 L6 T0 (C) Latent heat (D) None of the above
35. The velocity of water waves v may depend upon
30. If V denotes the potential difference across the their wavelength  , the density of water  and
plates of a capacitor of capacitance C , the the acceleration due to gravity g . The method of
dimensions of CV 2 are dimensions gives the relation between these
quantities as
(A) Not expressible in MLT
(A) v 2 rg (B) v2  g
(B) MLT 2
(C) v2  g (D) v 2  g 1 3
(C) M 2 LT 1
36. The equation of a wave is given by
(D) ML2 T 2
x 
Y  A sin    k 
31. If L denotes the inductance of an inductor v 
where  is the angular velocity and v is the
through which a current i is flowing, the
linear velocity. The dimension of k is
dimensions of Li 2 are
(A) LT (B) T (C) T 1 (D) T 2
(A) ML2 T 2
37. If C and L denote capacitance and inductance
(B) Not expressible in MLT
respectively, then the dimensions of LC are
(C) MLT 2
(A) M0 L0 T0 (B) M0 L0 T2
2 2 2
(D) M L T (C) M 2 L0T 2 (D) MLT 2

50
UNIT AND DIMENSION PHYSICS-X I-IIT-JEE

L 43. 0 and 0 denote the permeability and


38. The dimensions of "time constant" during
R permittivity of free space, the dimensions of
growth and decay of current in all inductive
circuit is same as that of 0 0 are
(A) Constant (B) Resistance (A) LT1 (B) L2T2
(C) Current (D) Time
1 3 2 2
(C) M L Q T (D) M 1L3I 2 T 2
39. The period of a body under SHM i.e. presented
by T  P a D bSc ; where P is pressure, D is 44. The dimensions of physical quantity X in the
density and S is surface tension. The value of X
equation Force  is given by
a, b and c are Density
3 1 (A) M1L4 T 2 (B) M 2 L2 T 1
(A)  , ,1 (B) 1,  2,3
2 2
1 3 1 1 (C) M 2 L2T 2 (D) M1L2 T 1
(C) ,  ,  (D) 1, 2,
2 2 2 3 45. The Martians use force (F) , acceleration (A)
40. The velocity of a freely falling body changes as and time (T) as their fundamental physical
p q
g h where g is acceleration due to gravity and quantities. The dimensions of length on Martians
h is the height. The values of p and q are system are
1 1 1 (A) FT2 (B) F1T2
(A) 1, (B) ,
2 2 2
(C) F1A2T 1 (D) AT 2
1
(C) ,1 (D) 1,1
2
46. If P represents radiation pressure, c represents
41. A small steel ball of radius r is allowed to fall speed of light and Q represents radiation energy
under gravity through a column of a viscous striking a unit area per second, then non-zero
liquid of coefficient of viscosity  . After some
integers x,y and z such that PxQycz is
time the velocity of the ball attains a constant
dimensionless, are
value known as terminal velocity vT . The (A) x  1, y  1, z  1
terminal velocity depends on (i) the mass of the
(B) x  1, y  1, z  1
ball m , (ii)  , (iii) r and (iv) acceleration due
(C) x  1, y  1, z  1
to gravity g . Which of the following relations is
(D) x  1, y  1, z  1
dimensionally correct
mg r 47. If R and L represent respectively resistance
(A) v T  (B) v T 
r mg and self inductance, which of the following
mgr combinations has the dimensions of frequency
(C) vT  rmg (D) v T 
 R L R L
(A) (B) (C) (D)
L R L R
 0 LV
42. The quantity X  :  0 is the permittivity 48. If velocity v , acceleration A and force F are
t
chosen as fundamental quantities, then the
of free space, L is length, V is potential
dimensional formula of angular momentum in
difference and t is time. The dimensions of X
terms of v, A and F would be
are same as that of
(A) FA1v (B) Fv3 A 2
(A) Resistance (B) Charge
(C) Voltage (D) Current (C) Fv2 A 1 (D) F2 v2 A 1

51
PHYSICS-X I-IIT-JEE UNIT AND DIMENSION
49. The dimensions of Planck's constant and angular 55. A physical quantity x depends on quantities y
momentum are respectively and z as follows: x  Ay  B tan Cz , where
(A) ML2 T 1 and MLT 1
A, B and C are constants. Which of the
(B) ML2 T 1 and ML2 T 1
following do not have the same dimensions
(C) MLT 1 and ML2 T 1
1 2 2
(A) x and B (B) C and z 1
(D) MLT and ML T
(C) y and B / A (D) x and A
50. In a certain system of units, 1 unit of time is 5
sec, 1 unit of mass is 20 kg and unit of length is 56. If L, C and R denote the inductance,
10 m. In this system, one unit of power will
capacitance and resistance respectively, the
correspond to -
1 dimensional formula for C2 LR is
(A) 16 watts (B) watts
16 (A) [ML2 T 1I0 ] (B) [M0 L0 T 3 I0 ]
(C) 25 watts (D) None of these
(C) [M1L2T6I2 ] (D) [M0 L0 T 2 I0 ]
51. The force F on a sphere of radius 'a ' moving in
57. If the velocity of light (c) , gravitational constant
a medium with velocity 'v' is given by
F  6av . The dimensions of  are (G) and Planck's constant (h) are chosen as

(A) ML1T 1 (B) MT 1 fundamental units, then the dimensions of mass


in new system is
(C) MLT 2 (D) ML3
(A) c1/2 G1/2 h1/2 (B) c1/2 G1/2 h 1/ 2
52. A force F is given by F  at  bt 2 , where t is (C) c1/2 G 1/2 h1/ 2 (D) c 1/2 G1/2 h1/ 2
time. What are the dimensions of a and b
58. According to Newton, the viscous force acting
(A) MLT 3 and ML2 T 4
between liquid layers of area A and velocity
(B) MLT 3 and MLT 4 v
gradient v / z is given by F  A where
(C) MLT 1 and MLT0 z
(D) MLT 4 and MLT1  is constant called coefficient of viscosity. The
dimension of  are
53. If the speed of light (c) , acceleration due to
gravity (g) and pressure (p) are taken as the (A) [ML2T 2 ] (B) [ML1T1 ]
fundamental quantities, then the dimension of (C) [ML2 T 2 ] (D) [M 0 L0T 0 ]
gravitational constant is
59. Frequency is the function of density () , length
(A) c2g 0p2 (B) c0g 2p1
(a) and surface tension (T) . Then its value is
(C) cg3p2 (D) c1g0 p1
(A) k1/2 a 3/2 / T (B) k3/2 a 3/2 / T
54. If the time period (T) of vibration of a liquid
(C) k1/2a 3/2 / T3/4 (D) k1/2 a1/2 / T 3/2
drop depends on surface tension (S) , radius (r)
of the drop and density () of the liquid, then the 60. In the relation y  a cos(t  kx) , the dimensional
expression of T is formula for k is
(A) T  k r / S 3 1/2 3
(B) T  k  r / S (A) [M0 L1T 1 ] (B) [M 0 LT 1 ]

(C) T  k r3 / S1/2 (D) None of these (C) [M 0 L1T 0 ] (D) [M 0 LT]

52
UNIT AND DIMENSION PHYSICS-X I-IIT-JEE
61. Position of a body with acceleration 'a' is given 67. The mean time period of second's pendulum is
by x  Ka m t n , here t is time. Find dimension of 2.00s and mean absolute error in the time period
m and n. is 0.05s. To express maximum estimate of error,
the time period should be written as
(A) m 1, n 1 (B) m  1, n  2
(A) (2.00  0.01) s (B) (2.00 +0.025) s
(C) m  2, n  1 (D) m  2, n  2 (C) (2.00  0.05) s (D) (2.00  0.10) s

62. In a system of units if force (F), acceleration (A) 68. A body travels uniformly a distance of
and time (T) are taken as fundamental units then (13.8 0.2) m in a time (4.0  0.3) s. The velocity
the dimensional formula of energy is of the body within error limits is
(A) FA2 T (B) FAT 2 (A) (3.45  0.2) ms-1 (B) (3.45  0.3) ms-1
(C) F2 AT (D) FAT (C) (3.45  0.4) ms-1 (D) (3.45  0.5) ms-1

69. The decimal equivalent of 1/20 upto three


Based On Errors
significant figures is
63. The volume of one sphere is 1.76 c.c. The (A) 0.0500 (B) 0.05000
volume of 25 such spheres (according to the idea (C) 0.0050 (D) 5.0 × 10-2
of significant figures) is -
(A) 44.00 cc (B) 44.0 c.c 70. The radius of a sphere is (5.3  0.1) cm. The
percentage error in its volume is
(C) 44 c.c (D) 0.44 × 102 c.c.
0.1 0.1
(A)  100 (B) 3   100
64. A calorie is a unit of heat and equals 4.2 J. 5.3 5.3
Suppose we employ a system of units in which of 0.1  100 0.1
(C) (D) 3   100
the unit of mass is  kg, the unit of length is  3.53 5.3
metre and the unit of time is  second. In this 71. A thin copper wire of length lmetre increases in
new system, 1 calorie =  length by 2% when heated through 10ºC. What is
(A)–1–22 (B) 4.2 2–2 the percentage increase in area when a square
(C)2–2 (D) 4.2 –1–22 copper sheet of length lmetre is heated through
10ºC
65. The percentage errors in the measurement of
(A) 4% (B) 8%
mass and speed are 2% and 3% respectively. (C) 16% (D) None of the above
How much will be the maximum error in the
estimation of the kinetic energy obtained by 72. In the context of accuracy of measurement and
measuring mass and speed significant figures in expressing results of
(A) 11% (B) 8% experiment, which of the following is/are correct
(C) 5% (D) 1% (i) Out of the two measurements 50.14 cm and
0.00025 ampere, the first one has greater
66. The random error in the arithmetic mean of 100 accuracy
observations is x; then random error in the (ii) If one travels 478 km by rail and 397 m. by
arithmetic mean of 400 observations would be road, the total distance travelled is 478 km.
1 (A) Only (i) is correct
(A) 4x (B) x
4 (B)Only (ii) is correct
1 (C) Both are correct
(C) 2x (D) x (D) None of them is correct.
2

53
PHYSICS-X I-IIT-JEE UNIT AND DIMENSION

73. A physical parameter a can be determined by 79. According to Joule's law of heating, heat
measuring the parameters b, c, d and e using the produced H  I 2 Rt, where I is current, R is
relation a = b c / d  e . If the maximum errors resistance and t is time. If the errors in the
in the measurement of b, c, d and e are b 1 %, c1 measurement of I, R and t are 3%, 4% and 6%
respectively then error in the measurement of H
%, d 1 % and e1%, then the maximum error in the
is
value of a determined by the experiment is (A)  17% (B)  16% (C)  19% (D)  25%
(A) ( b1  c1  d1  e1 )%
(B) ( b1  c1  d1  e1 )% 80. If there is a positive error of 50% in the
measurement of velocity of a body, then the error
(C) ( b1 c1  d1  e1 )%
in the measurement of kinetic energy is
(D) ( b1 c1  d1  e1 )% (A) 25% (B) 50%
(C) 100% (D) 125%
74. The relative density of material of a body is
found by weighing it first in air and then in water. 1
A3 B 2
If the weight in air is (5.00 0.05 ) Newton and 81. A physical quantity P is given by P= 3
.
weight in water is (4.00  0.05) Newton. Then the C 4
D2
relative density along with the maximum The quantity which brings in the maximum
permissible percentage error is percentage error in P is
(A) 5.0  11% (B) 5.0  1% (A)A (B)B (C)C (D)D
(C) 5.0  6% (D) 1.25  5%

V 82. The number of significant figures in all the given


75. The resistance R = where V= 100  5 volts numbers 25.12, 2009, 4.156 and 1.217  10 4 is
i
and i = 10  0.2 amperes. What is the total error (A) 1 (B) 2 (C) 3 (D) 4
in R
83. If the length of rod A is 3.25  0.01 cm and that
5
(A) 5% (B) 7% (C) 5.2% (D) % of B is 4.19  0.01 cm then the rod B is longer
2 than rod A by
76. The period of oscillation of a simple pendulum in (A)0.94  0.00 cm (B)0.94  0.01 cm
the experiment is recorded as 2.63 s, 2.56 s, 2.42
(C) 0.94  0.02 cm (D)0.94  0.005 cm
s, 2.71 s and 2.80 s respectively. The average
absolute error is 84. A physical quantity is given by X  Ma Lb T c .
(A) 0.1s (B) 0.11s (C) 0.01s (D) 1.0s The percentage error in measurement of M, L and
77. The length of a cylinder is measured with a meter T are ,  and  respectively. Then maximum
rod having least count 0.1 cm. Its diameter is percentage error in the quantity X is
measured with vernier calipers having least count (A) a  b  c (B) a  b  c
0.01 cm. Given that length is 5.0 cm. and radius is a b c
2.0 cm. The percentage error in the calculated (C)   (D) None of these
  
value of the volume will be
(A) 1% (B) 2% (C) 3% (D) 4% 85. A physical quantity A is related to four
a 2 b3
78. In an experiment, the following observations observable a, b, c and d as follows, A  ,
c d
were recorded:L = 2.820 m, M = 3.00 kg,
l = 0.087 cm, Diameter D= 0.041 cm Taking the percentage errors of measurement in a, b, c
4MgL and d are 1%,3%,2% and 2% respectively. What
g = 9.81 m/s2 using the formula,Y= , the
D2l is the percentage error in the quantity A
maximum permissible error in Y is (A)12% (B)7%
(A) 7.96% (B) 4.56% (C) 6.50% (D) 8.42% (C) 5% (D) 14%

54
UNIT AND DIMENSION PHYSICS-X I-IIT-JEE

EXERCISE # 2
One or More than One correct Answer
Question
Type Questions
8. Choose the correct statement(s)
1. The dimensions of the quantities in one (or more) (A) A dimensionally correct equation must be
of the following pairs are the same. Identify the correct
pair (s). (B) A dimensionally correct equation may be
(A) Torque and work
correct
(B) Angular momentum and work
(C) A dimensionally incorrect equation must be
(C) Energy and Young’s modulus
incorrect
(D) Light year and wavelength
(D) A dimensionally incorrect equation my be
2. The pairs of physical quantities that have the correct
same dimensions is (are)
(A) Reynolds number and coefficient of friction 9. Which of the following pairs have the same
(B) Curie and frequency of a light wave dimensions ?
(A) h/e and magnetic flux
(C) Latent heat and gravitational potential
(B) h/e and electric flux
(D) Planck’s constant and torque (C) Electric flux and q/0
3. The SI unit of the inductance, the henry can by (D) Electric flux and µ0I
written as 10. The values of measurement of a physical quantity
(A) weber/ampere (B) volt-second/ampere in five trails were found to be 1.51, 1.53, 1.53,
2
(C) joule/k(ampere) (D) ohm-second 1.52 and 1.54. Then
4. Let [0] denote the dimensional formula of the (A) Average absolute error is 0.01
permittivity of the vacuum and [µ0] that of the (B) Relative error is 0.01
permeability of the vacuum. If M = mass, L = (C) Percentage error is 0.01 %
length, T = time and I = electric current. (D) Percentage error is 1 %
(A) [0] = [M–1L–3T2] (B) [0] = [M–1L–3T4I2]
11. If S and V are one main scale and one Vernier
(C) [0] = [MLT–2I–2] (D) [0] = [ML2T–1I]
scale and n – 1 divisions on the main scale and
5. L, C and R represent the physical quantities equivalent to n divisions of the Vernier, then
inductance, capacitance and resistance (A) The least count is S/n
respectively. The combination which have the (B) The vernier constant is S/n
dimensions of frequency are (C) The same vernier constant can be used for
1 R 1 C circular Verniers also
(A) (B) (C) (D) (D) The same Vernier constant cannot be used for
RC L LC L
circular Verniers
6. Which of the following pairs have different
E 1
dimensions ? 12. Consider three quantities : x = ,y= ,
(A) Frequency and angular velocity b µ00
(B) Tension and surface tension 1
and z = . Here, I is the length of a wire, C is
(C) Density and energy density CR
(D) Linear momentum and angular momentum the a resistance. All other symbols have usual
meanings. Then
7. Pressure is dimensionally
(A) x and y have the same dimensions
(A) Force per unit area (B) x and z have the same dimensions
(B) Energy per unit volume (C) y and z have the same dimensions
(C) Momentum per unit area per second (D) None of the above three pairs have the same
(D) Momentum per unit volume dimensions

55
PHYSICS-X I-IIT-JEE UNIT AND DIMENSION

13. The velocity 'v' of a particle is given by Question Passage Based Questions
v = a + bt + ct2 , then
(A) Dimensions of a is same as v Passage # 1 (Q.No.18 to 20)
(B) Dimensions of b is same as ct A gas bubble, from an explosion under water, oscillates
(C) Dimensions of c is same as v/t2 with a period T proportional to pa db Ec, where p is the
(D) Dimensions of b2 is same as ac pressure, d is the density of water and E is the total
energy of explosion -
m
14. If y = (where m is the mass, µ0 is magnetic 18. The value of a is -
00
5 1 1
permeability of free space and 0 is electrical (A) – (B) (C) – (D) 1
permittivity of free space) then the dimensional 6 2 2
formula of y is same as that of the-
(A) Work 19. The value of b is -
(B) Angular momentum 5 1 1
(A) – (B) (C) – (D) 1
(C) Linear momentum × velocity 6 2 2
(D) Kinetic energy × mass
 t 
20. The value of c is -
15. In the expression Q = Q0  1  e   , Q is charge, t 5 1 1
  (A) – (B) (C) (D) 1
 
6 2 3
is time and  is some constant then which of the
following option(s) is / are correct?
t Passage # 2 (Q.No.21 to 23)
(A) is dimensionless Let us consider a particle P where is moving straight on

(B) the dimensions of Q is [Mº Lº AT] the X-axis. We also know that the rate of change of its

t dx
(D) e  is dimensionless position is given by ; where x is its separation from
dt
(D) Q0 is dimensionsless
dx
the origin and t is time. This term is called the
16. Which of the following options is/are correct ? dt
dV velocity of particle (v). Further the second derivation of
(A) = [M0L3T–1] where V is volume, t is time
dt x, w.r.t. time is called acceleration (A) or rate of change
dm dx 2 dv
(B) = [ML0T–1] where m is mass, t is time of velocity and represented by or . If the
dt dt 2
dt
d
(C) = [M0LT–1] where  is length, t is time acceleration of this particle is found to depend upon
dt
time as follows
dv
(D) = [M0LT–1] where v is speed, t is time Ct
dt f = At + Bt2 + then.
D  t2
17. A book with many printing errors contains four
different expressions for the displacement y of a 21. The dimensions of A are -
particle executing simple harmonic motion –
(A) LT–2 (B) LT–3 (C) LT3 (D) L2T3
Find the wrong formulae on dimensional ground.
(where A is length, v is speed and  is angular
22. The dimensions of B are -
frequency)
(A) y = A sin (2 t/T) (A) LT–4 (B) L2T–3 (C) LT4 (D) LT–2
(B) y = A sin vt
(C) y = (A/T) sin (t/A) 23. The dimensions of C are -
(D) y = (A/ 2 ) (sin t + cos t) (A) L2T–2 (B) LT–2 (C) LT–1 (D) T2

56
UNIT AND DIMENSION PHYSICS-X I-IIT-JEE

Passage # 3 (Q.No.24&25) 27. The volumes of two bodies are measured to be


Dimensional methods provide three major advantage in V1 = (10.2 ± 0.02) cm3 and V2 = (6.4 ± 0.01) cm3.
verification, derivation and to change the system of Then sum of volumes with error limits is -
units. Any emperical formula that is derived based on (A) (16.6 ± 0.03) cm3 (B) (16.6 ± 0.06) cm3
this method has to be verified and proportionality (C) (16.6 ± 0.05) cm3 (D) (3.8 ± 0.03) cm3
constants found by experimental means. The presence
or absence of certain factors (numerical constants of 1, Passage # 5 (Q.No.28& 29)
2,  etc) cannot be identified by this method. So every The relative density of the material of a body is the ratio
dimensionally correct relation cannot be taken as of its weight in air and the loss of its weight in water by
perfectly correct. using a spring balance, the weight of the body in air is
24. Energy (E) of a oscillating particle is dependent measured to be 5.00 ± 0.05 N. The weight of the body
on mass M, frequency 'f' and amplitude 'A' of in water is measured to be 4.00 ± 0.05 N.
oscillation. Then energy 'E' is proportional to -
Mf 28. The maximum possible percentage error in
(A) 2 (B) Mf2A2 relative density is -
A
(C) Mf2 A–2 (D)MfA–2 (A) 11% (B) 10% (C) 9% (D) 7%

25. Which of the following relations is dimensionally 29. In the above case the relative density is -
incorrect? (A) 1.25 ± 9% (B)1.25 ± 11%
42  (C)5.00 ± 11% (D)1 ± 10%
(A) g = where g is acceleration due to
T2
gravity,  is length, T is time period Passage # 6 (Q.No.30 to 34)
The van der Waal’s equation of state for some gases can
2k be expressed as
(B) v = ; m  mass
m a 

k  kinetic energy  P  2  (V  b) = RT
 V 
v  speed where P is the pressure, V is the molar volume, and T is
F G the absolute temperature of the given sample of gas and
(C) 2 = 2 ; F  force, m  mass
m r a, b and R are constant
Ggravitationalconstant
r  distance 30. The dimensions of a are
(D) v2 = u2 + 2as2; v  final velocity (A) ML5T–2 (B) ML–1T–2
u  initial velocity (C) L3 (D) L0
a  acceleration
s  displacement 31. The dimensions of constant b are
(A) ML5T–2 (B) ML–1T–2
Passage # 4 (Q.No.26& 27) (C) L3 (D) L6
Let x = a ± b
Further, let a is the absolute error in the measurement 32. Which of the following does not have the same
of a, b the absolute error in the measurement of b and dimensional formula as that for RT?
x is the absolute error in the measurement of x. a ab
(A) PV (B) Pb (C) 2 (D) 2
Therefore, the maximum absolute error in x is, V V
x = ± (a + b)
26. Which of the following is dimensionless ? 33. The dimensional representation of ab/RT is
a (A) ML5T–2 (B) M0L3T0
(A)a (B) (C)x (D) none –1 –2
(C) ML T (D) None of these
a

57
PHYSICS-X I-IIT-JEE UNIT AND DIMENSION
34. In the above problem, the dimensional formula Passage # 8(Q.No.38 to 40)
for RT is same as that of The accuracy of measurement also lies in the way the
(A) Energy (B) Force result is expressed. The number of digits to which a
(C) Specific heat (D) Latent heat value is to be expressed is one digit more than number
of sure numbers. Rules do exist to deal with number of
Passage # 7(Q.No.35 to 37) digits after an operation is carried out on the given
Dimensional methods provide three major advantages in values. The error can be minimized by many trials and
verification, derivation, and changing the system of using the correct methods and instruments.
units. Any empirical formula that is derived based on
this method has to be verified and proportional 38. If the length and breadth are measured as 4.234
constants found by experimental means. The presence and 1.05 m, the area of the rectangle is
or absence of certain factors-non-means. So every (A) 4.4457 m2 (B) 4.45 m2
dimensionally correct relation cannot be taken as (C) 4.446 m2 (D) 0.4446 m2
perfectly correct.
39. The order of magnitude of 147 is
35. If  kilogram,  meter, and  second are the (A) 1 (B) 2
fundamental units. (C) 3 (D) 4
(A)–12  (B)–12
 (C) 2–1  (D) 4.2–1–22 40. The number of significant figures can reduce in
(A) Addition (B) Subtraction
36. The time period of oscillation of a drop depends (C) Multiplication (D) Division
on surface tension , density of the liquid , and
radius r. The relation is-
r 2 r2 r 3 
(A) (B) (C) (D) 3
   r

37. The energy of an SHM is dependent on mass m,


frequency f, and amplitude A of oscillation. The
relation is-
(A) Mf/A2 (B) MfA–2
(C) Mf2A–2 (D) Mf2A2

58
UNIT AND DIMENSION PHYSICS-X I-IIT-JEE

EXERCISE # 3
Question Column Match Type Questions 5. Using significant figures, match the following

1. Match the Columns. Column I Column II


Column-I Column-II (A) 0.12345 (p) 5
(A) Radiation energy (p) Joule/m2 (B) 0.12100 cm (q) 4
(B) Surface tension (q) ML2T–2
(C) 47.23  2.3 (r) 1
(C) Torque (r) ML–1T–1
8
(D) Coefficient of (s) MT–2 (D) 3 × 10 (s) 2
Viscosity
6. Match the two columns
2. If R is resistance, L is inductance, C is capacitance,
Column I Column II
H is latent heat, and s is specific heat, then match
the quantity given in Column I with the (A)Boltzmann constant (p)[ML2T–1]
dimensions given in Column II. (B)Coefficient of viscosity (q)[ML–1T–1]
Column I Column II
(C)Planck constant (r)[MLT–3K–1]
(A) LC (p) L2T–2
(D)Thermal conductivity (s) [ML2T–2K–1]
(B) LR (q) L2T–2K–1
(C) H (r) T2
7. Suppose two students are trying to make a new
(D) s (s) M2L4T–5A–4
measurement system so that they can use it like a
3. There are four Vernier scales, whose
code measurement system and others do not
specifications are given in Column I and II the
least count is given in Column II. Match the understand it. Instead of taking 1 kg, 1m and 1
Columns I and Ii with correct specification and sec as basic unit they took unit of mass as a kg,
corresponding least count (s = value of main the unit of length as b m and unit of time as 
scale division, n = number of marks on Vernier.
second. The called power in new system as
Assume (n – 1) main scale divisions are equal to
n Vernier division. ACME then match the two columns.
Column I Column II Column I Column II
(A) s = 1 mm, n = 10 (p) 0.05 mm (A) 1 N in new (p)–1–22
(B) s = 0.5 mm, n = 10 (q) 0.01 mm
system
(C) s = 0.5 mm, n = 20 (r) 0.1 mm
(D) s = 1mm, n = 100 (s) 0.025 mm
(B)1 J in new system (q)–1–12
(C) 1 Pascal (SI (r)–12
4. Match the columns.
unitof pressure)
Column I Column II
(A) Backlash error (p) Always subtracted innew system
(B) Zero error (q) Least count = (D)ACME in Watt (s)223
1 MSD = 1 VSD
(C) Vernier calipers (r) May be negative 8. Some physical quantities are given in column I and
or positive some SI units in which these quantities may be
(D) Error in screw (s) One to loose expressed are given in column (II) Match the
gage fittings
physical quantities in column (I)

59
PHYSICS-X I-IIT-JEE UNIT AND DIMENSION

Column I Column II Question Numerical Questions


(A) Gravitational field intensity joule
(p) 11. Force applied by water jet from a pipe depends
GM e mol  kelvin
I= ; where upon (i) velocity of water (ii) density of water
R e2
(iii) cross-sectional area of pipe. How many times
G = gravitational constant force will be increased if velocity of a water is
Me = mass of the earth increased 2 times?
Re = radius of the earth
12. A student measures diameter of a sphere using
(B) Electric field intensity, newton
(q) vernier calliper having least count 0.1 mm and
E kg
E= Where F = force due reports diameter equal to 0.025307 meter.
Q
Numbers of significant figure in diameter will be-
to charge
3 2
Q = electric charge 13. A quantity x is defined as x = a  b . Value of
V newton cd
(C) R = ; where V = potential (r)
I coulomb a, b, c and d are reported as a = 3 ± 0.001, b = 5 ±
difference across a resistance 0.0013, c = 6 ± 0.24 and d = 10 ± 0.4. Percentage
I = current in the resistance error in x will be -
(D) Universal gas constant volt
(s) 14. The lengths of sides of cuboid are a, 2a and 3a. If
PV ampere
R= the relative percentage error in the measurement
nR of a is 1%, then what is the relative percentage
9. Match the physical quantities given in Column (I) error in the measurement of volume of cube.
with dimensions expressed in terms of mass (M),
15. The length of a cylinder is measured with a metre
length (L), time (T), and change (Q) given in
rod having least count 0.1 cm. Its diameter is
Column (II) 
measured with vernier calipers having least count
Column I Column II 0.01 cm. Given that length is 5.0 cm and radius is
(A)Angular momentum (p)[ML2T–2] 2.0 cm. The percentage error in the calculated
(B) Latent heat (q) [ML2Q–2] value of the volume will be -
(C) Torque (r) [ML2T–1]
16. A 2m wide truck is moving with a speed of
(D) Capacitance (s) [ML3T–1Q–2]
5 5 m/s along a straight horizontal road. A man
(E) Inductance (t) [M–1L–2T2Q2]
starts crossing the road with a uniform speed v
(F) Resistivity (u) [L2T–2]
when the truck is 4m away from him. The
10. Column I gives three physical quantities. Select minimum value of v (in m/s) to cross the truck
the appropriate units for the choices given in safely is-
Column (II) Some of the physical quantities may  t2 
17. If equation dt = a sin  a 2  1 , find
2 –1 


have more than one choice.  3at  2t 2  
Column I Column II the value of x.
(A)Capacitance (p) ohm–second
18. If unit of mass becomes 2 times, the unit of
(B) Inductance (q) coulomb2–joule–1
length becomes 4 times and the unit of time
(C) Magnetic induction (r) coulomb (volt)–1
becomes 4 times in the unit of Plank’s constant.
(s) newton(ampere metre)–1 Due to this unit of Planck’s constant become n
(t) volt–second (ampere)–1 times. Find the value of n.

60
UNIT AND DIMENSION PHYSICS-X I-IIT-JEE
19. A stone is lying at rest in river. The minimum 23. Acceleration due to gravity on the surface of the
mass of stone, m = kpvxg–3 is needed for GM
earth is g = . The gravitational constant G is
remaining at rest. Here, k = constant having no R2
unit, g = acceleration due to gravity, v = river exactly know. But percentage error in
flow velocity, r = density of water. Find the measurement of the mass of earth M and radius
value of x. of the earth R are 1 % and 2 % respectively. The
maximum percentage error is measurement of
20. If the unit of velocity is run, the unit of time is acceleration due to gravity on the surface of the
second and unit of force is strength in a earth in n %. Find the value of n.
hypothetical system of unit. In this system of
unit, the unit of mass is 24. During measurement and kinetic energy T, the
y percentage error in measurement of mass of
(strength)x(second) y(run)z. Find the value of .
x particle and momentum of particle are 2% and
3%, respectively. The percentage error in
21. A student determines a dimensionless quantity, B measurement of kinetic energy is n%. Find the
en value of n.
= . Find the value of n. (Here, e = electric
20 hc
25. If x = 0.72 + 0.8 + 3.87 – 1.089, then find number
change, 0 = electric permittivity of vacuum, h =
of significant digits in the value of x.
Planck’s constant and c = speed of light).

26. The length of a wire is 2.17 cm and radius is 0.46


22. If volume is written as, V = Kgxcyhz. Here, K is
cm. Find number of significant digits in the
dimensionless constant and g, c, h are
value of volume of wire.
gravitational constant, speed of light and Planck’s
x
constant, respectively. Find the value of .
z

61
PHYSICS-X I-IIT-JEE UNIT AND DIMENSION

EXERCISE # 4
Question Previous Year (JEE Mains)
8. The dimension of magnetic field in M, L, T and C
1. The pairs having same dimensional formula –
[AIEEE-2002] (Coulomb) is given as [AIEEE-2008]
(A) Angular momentum, torque (A) MT2C–2 (B) MT–1C–1
(B) Torque, work (C) MT–2C–1 (D) MLT–1C–1
(C) Plank's constant, boltzman's constant
(D) Gas constant, pressure 9. Resistance of a given wire is obtained by
measuring the current flowing in it and the
2. Thephysical quantities not having same
voltage difference applied across it. If the
dimensions are – [AIEEE-2003]
(A) Momentum and Planck’s constant percentage errors in the measurement of the
(B) Stress and Young’s modulus current and the voltage difference are
(C) Speed and (00)–1/2 3% each, then error in the value of resistance of
(D) Torque and work the wire is- [AIEEE-2012]
(A) zero (B) 1 %
1
3. Dimensions of , where symbols have their (C) 3 % (D) 6 %
00
usual meaning, are – [AIEEE-2003]
10. Let [0] denote the dimensional formula of the
(A) [ L–2 T2 ] (B) [ L2 T–2 ]
(C) [ L T–1 ] (D) [ L–1 T] permittivity of vacuum. If M = mass,
L = length, T = time and A = electric current,
4. Which one of the following represents the correct then - [JEE Main -2013]
dimensions of the coefficient of viscosity – (A) [0] = [M–1 L2 T–1 A–2]
[AIEEE-2004] (B) [0] = [M–1 L2 T–1 A]
(A) ML–1T–2 (B) MLT–1
(C) ML–1T–1 (D) ML–2T–2 (C) [0] = [M–1 L–3 T2 A]
(D) [0] = [M–1 L–3 T4 A2]
5. Out of the following pair, which one does NOT
have identical dimensions is [AIEEE-2005] 11. The dimensions of angular momentum, latent
(A) Angular momentum and Planck's constant heat and capacitance are, respectively -
(B) Impulse and momentum [JEE Main Online -2013]
(C)Moment of inertia and moment of a force (A) ML T A , L T , M–1 L–2 T2
2 1 2 2 –2

(D)Work and torque (B) ML2 T–1, L2T2, M–1 L–2 T4A2
(C) ML2T–1, L2T–2, ML2TA2
6. Which of the following units denotes the
dimensions ML2 /Q2, where Q denotes the (D) ML2T–1, L2T–2, M–1L–2T4A2
electric charge – [AIEEE 2006] 12. The current voltage relation of diode is given by
(A) H/m2 (B) Weber (Wb) I = (e1000V/T – 1) mA, where the applied voltage V is
(C) Wb/ m2 (D) Henry (H) in volts and the temperature T is in degree Kelvin. If
a student makes an error measuring
7. A body of mass m = 3.513 kg is moving along ± 0.01 V while measuring the current of 5 mA at
the x-axis with a speed of 5.00 ms–1. The
300 K, what will be the error in the value of current
magnitude of its momentum is recorded as
[AIEEE-2008] in mA ? [JEE Main -2014]
(A)17.565 kg ms –1
(B)17.56 kg ms–1 (A) 0.02 mA (B) 0.5 mA
(C)17.57 kg ms–1 (D)17.6 kg ms–1 (C) 0.05 mA (D) 0.2 mA

62
UNIT AND DIMENSION PHYSICS-X I-IIT-JEE
13. In terms of resistance R and time T, the 18. A, B, C and D are four different physical
 quantities having different dimensions. None of
dimension of ratio of the permeability  and them is dimensionless. But we know that the

equation AD = C ln(BD) holds true. Then which
permittivity  is : [JEE Main Online -2014]
of the combination is not a meaningful quantity ?
(A) [RT–2] (B) [R2T–1]
[JEE-Main On line-2016]
(C) [R2] (D) [R2T2]
A
(i) A2 – B2C2 (ii) –C
14. The period of oscillation of a simple pendulum is T B
L (A  C) C AD 2
(iv) (iii) –
= 2 . Measured value of L is 20.0 cm known to D BD C
g
(A)i,ii (B)ii,iii
1 mm accuracy and time for100 oscillations of the (C)iii,iv (D)i,iv
pendulum is found to be 90 s using a wrist watch of
1 s resolution. The accuracy in the determination of 19. The following observations were taken for
g is - [JEE Main - 2015] determining surface tension T of water by
(A) 2 % (B) 3 % capillary method:
Diameter of capillary, D = 1.25 × 10–2 m
(C) 1 % (D) 5 %
rise of water, h = 1.45 × 10–2 m.
15. If electron charge e, electron mass m, speed of Using g = 9.80 m/s2 and the simplified relation T
light in vacuum c and Planck's constant h are rhg
= × 103 N/m, the possible error in surface
taken as fundamental quantities, the permeability 2
tension is closest to – [JEE-Main 2017]
of vacuum 0 can be expressed in units of :
(A) 0.15% (B) 1.5% (C) 2.4% (D) 10%
[JEE Main Online - 2015]
h   mc 2  20. Time (T), velocity (c) and angular momentum (h)
(A)  2 
(B)  2 
 ce   he are chosen as fundamental quantities instead of
 
mass, length and time. In terms of these, the
h  hc 
(C)  2 
(D)  2 
dimensions of mass would be-
 me   me  [JEE-Main On line-2017]
(A) [M] = [T C h] (B) [M] = [T C–2 h]
–1 –2
16. A student measures the time period of
(C)[M] = [T–1 C–2 h–1] (D) [M] = [T–1 C2 h]
100 oscillations of a simple pendulum four times.
The data set is 90 s, 91 s, 95 s and 92 s. If the 21. A physical quantity P is described by the relation
minimum division in the measuring clock is 1 s, P = a1/2 b2 c3 d–4If the relative errors in the
then the reported mean time should be: measurement of a, b, c and d respectively, are
2%, 1%, 3% and 5%, then the relative error in P
[JEE-Main 2016]
will be- [JEE-Main On line-2017]
(A) 92 ± 2 s (B) 92 ± 5.0 s
(A) 12% (B) 8% (C) 25% (D) 32%
(C) 92 ± 1.8 s (D) 92 ± 3 s
22. The density of a material in the shape of a cube is
17. In the following 'I' refers to current and other
determined by measuring three sides of the cube
symbols have their usual meaning. Choose the and its mass. If the relative errors in measuring
option that corresponds to the dimensions of the mass and length are respectively 1.5 % and
electrical conductivity : 1%, the maximum error in determining the
[JEE-Main On line-2016] density is - [JEE Main - 2018]
–3 –3 2
(A) ML T I (B) M–1 L3 T3 I (A) 2.5 % (B) 3.5 %
(C) M–1 L–3 T3 I2 (D) M–1 L–3 T3 I (C) 4.5 % (D) 6 %

63
PHYSICS-X I-IIT-JEE UNIT AND DIMENSION
23. The relative error in the determination of the 28. The diameter and height of a cylinder are
surface area of a sphere is . Then the relative measured by a meter scale to be 12.6 ± 0.1 cm
error in the determination of its volume is and 34.2 ± 0.1 cm, respectively. What
[JEE-Main Online-2018]
will be the value of its volume in appropriate
2 1
(A)  (B)  significant figures ?[JEE-Main Online-2019]
3 3
3 (A) 4260 ± 80 cm3
(C)  (D)
2 (B) 4300 ± 80 cm3
(C) 4264.4 ± 81.0 cm3
24. The characteristic distance at which quantum
gravitational effects are significant, the Planck (D) 4264 ± 81 cm3
length, can be determined from a suitable
combination of the fundamental physical 29. The force of interaction between two atoms is
constants G, h and c. Which of the following
 x2 
correctly gives the Planck length ? given by : F = exp  
 kt 
, where x is the
[JEE-Main Online-2018]  
1
2  Gh  2 distance, k is the Boltzmann constant and T is
(A) G hc (B)  3 
 c  temperature and  and  are two constants. The
1
(C) G 2 h 2c (D) Gh2c3 dimension of  is : [JEE-Main Online-2019]
(A) M2L2T–2 (B) M2LT–4
25. The percentage errors in quantities P,Q,R and S
(C) M0L2T–4 (D) MLT–2
are 0.5 %, 1%, 3% and 1.5 % respectively in the
3 2
measurement of a physical quantity A = P Q . 30. If speed (V), acceleration (A) and force (F) are
RS
considered as fundamental units, the dimension
The maximum percentage error in the value of A
of Young's modulus will be :
will be- [JEE-Main Online-2018]
(A) 8.5 % (B) 6.0 % [JEE-Main Online-2019]
(C) 7.5 % (D) 6.5 % (A) V–2A2F2 (B) V–4A2F
(C) V–4A–2F (D) V–2A2F–2
26. Expression for time in terms of G (universal
gravitational constant), h (Planck constant) and c
(speed of light) is proportional to : 31. Let , r, C and V represent inductance, resistance,
[JEE-Main Online-2019]
capacitance and voltage, respectively. The
5
Gh hc 
(A) (B) dimension of in SI units will be:
3 G
c rCV
[JEE-Main Online-2019]
c3 Gh
(C) (D)
5 (A) [LTA] (B) [LA–2]
Gh c
(C) [A–1] (D) [LT2]
27. The density of a material in SI units is 128 kg m–
3. In certain units in which the unit of length is 25
0
cm and the unit of mass is 50 g, the numerical 32. In SI units, the dimesions of is –
0
value of density of the material is :
[JEE-Main Online-2019] [JEE Mains Online-2019]
(A) 410 (B) 640 (A) A–1TML3 (B) A2T3M–1L–2
(C) 16 (D) 40 (C) AT2M–1L–1 (D) AT–3ML3/2

64
UNIT AND DIMENSION PHYSICS-X I-IIT-JEE
33. If surface tension (S), Moment of inertia (I) and 38. If momentum (P), area (a) and time (T) are taken
Planck's constant (h), were to be taken as the to be the fundamental quantities then the
fundamental units, the dimensional formula for dimensional formula for energy is :
linear momentum would be : [JEE Mains 2020]
–1 –2
[JEE Mains Online-2019] (A) [PA T ] (B) [PA1/2T–1]
(C) [P2AT–2] (D) [P1/2AT–1]
(A) S3/2 I1/2h0 (B) S1/2I1/2h0
(C) S1/2I1/2h–1 (D) S1/2I3/2h–1 39. Using screw gauge of pitch 0.1 cm and 50
divisions on its circular scale, the thickness of an
34. The area of a square is 5.29 cm2. The area of 7 object is measured. It should correctly be
such squares taking into account the significant recorded as : [JEE Mains 2020]
figures is :[JEE Mains Online-2019] (A) 2.123 cm (B) 2.125 cm
(A) 37 cm2 (B) 37.0 cm2 (C) 2.121 cm (D) 2.124 cm
(C) 37.03 cm2 (D) 37.030 cm2 40. Amount of solar energy received on the earth's
surface per unit area per unit time is defined a
35. In the formula X = 5YZ2, X and Z have
solar constant. Dimension of solar constant is:
dimensions of capacitance and magnetic field,
[JEE Mains 2020]
respectively. What are the dimensions of Y in SI 2 –2
(A) ML T (B) MLT–2
units ?[JEE Mains Online-2019] 2 0 –1
(C) M L T (D) ML0T–3
(A) [M–2 L–2 T6 A3] (B) [M–1 L–2 T4 A2]
(C) [M–3 L–2 T8 A4] (D) [M–2 L0 T–4 A–2] 41. Dimensional formula for thermal conductivity is
(here K denotes the temperature)
36. Which of the following combinations has the [JEE Mains 2020]
–3
dimension of electrical resistance (0 is the (A) MLT K (B) MLT–2K
(C) MLT–2K–2 (D) MLT–3K–1
permittivity of vacuum and µ0 is the permeability
of vacuum) ?[JEE Mains Online-2019] 42. A quantity x is given by (IFv2/WL4) in terms of
moment of inertia I, force F, velocity v, work W
(A)  0 / 0 (B) µ0 / 0 and Length L. The dimensional formula for x is
same as that of : [JEE Mains 2020]
(C)  0 / 0 (D)0 / µ0
(A) Planck's constant
37. The least count of the main scale of a vernier (B) Force constant
callipers is 1 mm. Its vernier scale is divided into (C) Energy density
10 divisions and coincide with 9 divisions of the (D) Coefficient of viscosity
main scale. When jaws are touching each other,
43. A screw gauge has 50 divisions on its circular
the 7th division of vernier scale coincides with a
division of main scale and the zero of vernier scale. The circular scale is 4 units ahead of the
scale is lying right side of the zero of main scale. pitch scale marking, prior to use. Upon one
When this vernier is used to measure length of a complete rotation of the circular scale, a
cylinder the zero of the vernier scale between 3.1 displacement of 0.5 mm is noticed on the pitch
cm and 3.2 cm and 4th VSD coincides with a scale. The nature of zero error involved, and the
main scale division. The length of the cylinder is: least count of the screw gauge, are respectively
[JEE Mains 2020]
[JEE Mains 2020]
(VSD is vernier scale division)
(A) 3.21 cm (B) 2.99 cm (A) Negative, 2 µm (B) Positive, 10 µm
(C) 3.2 cm (D) 3.07 cm (C) Positive, 0.1 µm (D) Positive, 0.1 mm

65
PHYSICS-X I-IIT-JEE UNIT AND DIMENSION

44. The density of a solid metal sphere is determined 49. Match list-I with list-II:
by measuring its mass and its diameter. The List-I List-II
maximum error in the density of the sphere is (a) Magnetic Induction (i) ML2T–2A–1
 x  (b) Magnetic Flux (ii) M0L–1A
  % . If the relative errors in measuring the
 100  (c) Magnetic Permeability (iii) MT–2A–1
mass and the diameter are 6.0% and 1.5%
(d) Magnetization (iv) MLT–2A–2
respectively, the value of x is –
Choose the most appropriate answer from the
[JEE Mains 2020]
options given below: [JEE Mains 2021]
45. A student measuring the diameter of a pencil of (A) (a)-(ii), (b)-(i), (c)-(iv), (d)-(iii)
circular cross-section with the help of a vernier (B) (a)-(ii), (b)-(iv), (c)-(i), (d)-(iii)
scale records the following four readings 5.50 (C) (a)-(iii), (b)-(ii), (c)-(iv), (d)-(i)
mm, 5.55 mm, 5.45 mm; 5.65 mm. The average (D) (a)-(iii), (b)-(i), (c)-(iv), (d)-(ii)
of these four readings is 5.5375 mm and the
50. In a Screw Gauge, fifth division of the circular
standard deviation of the data is 0.07395 mm.
scale coincides with the reference line when the
The average diameter of the pencil should
ratchet is closed. There are 50 divisions on the
therefore be recorded as : [JEE Mains 2020]
circular scale, and the main scale moves by 0.5
(A) (5.5375 ± 0.0739) mm
mm on a complete rotation. For a particular
(B) (5.538 ± 0.074) mm
observation the reading on the main scale is 5
(C) (5.54 ± 0.07) mm
mm and the 20th division of the circular scale
(D) (5.5375 ± 0.0740) mm
coincides with reference line. Calculate the true
46. A battery of unknown emf connected to a reading. [JEE Mains 2021]
potentiometer has balancing length 560 cm. If a (A) 5.20 mm (B) 5.00 mm
resistor of resistance 10 is connected in parallel (C) 5.25 mm (D) 5.15 mm
with the cell the balancing length change by 60
51. If E, L, M and G denote the quantities as energy,
cm. If the internal resistance of the cell is (n/10)
angular momentum, mass and constant of
, the value of 'n' is [JEE Mains 2020]
gravitation respectively, then the dimensions of P
47. The dimension of stopping potential V0 in in the formula P = EL2M–5G–2 are:
photoelectric effect in units of Planck's constant [JEE Mains 2021]
'h', speed of light 'c' and Gravitational constant 'G' (A) [M–1L–1T2] (B) [M1L1T–2]
and ampere A is : [JEE Mains 2020] (C) [M0L1T0] (D) [M0L0T0]
2 3/2 1/3 –1
(A) h G c A
(B) h–2/3 c–1/3 G4/3 A–1 52. Which of the following is not a dimensionless
(C) h1/3 G2/3 c1/3 A–1 quantity? [JEE Mains 2021]
(D) h0c5G–1A–1 (A) Quality factor
(B) Power factor
(C) Permeability of free space (µ0)
hc5
48. A quantity f is given by f = , where c is (D) Relative magnetic permeability (µr)
G
speed of light, G universal gravitational constant 53. If E and H represents the intensity of electric field
and h is the Planck's constant. Dimension of f is and magnetising field the unit of E/H will be
that of : [JEE Mains 2020] respectively, then [JEE Mains 2021]
(A) Momentum (B) Area (A) joule (B) ohm
(C) Energy (D) Volume (C) newton (D) mho

66
UNIT AND DIMENSION PHYSICS-X I-IIT-JEE
54. Match List-I with List-II. 59. Which of the following equations is dimensionally
[JEE Mains 2021] incorrect? [JEE Mains 2021]
List-I List-II Where t = time, h = height, s = surface tension, =
(a) RH(Rydberg constant) (i) kg m–1s–1 angle,
(b) h (Planck’s constant) (ii) kg m2s–1  = density, a, r = radius, g = acceleration due to
(c) µB(Magnetic field (iii) m–1 gravity,
v = volume, p = pressure, W = work done,  =
energy density)
torque
(d) (coefficient of viscocity) (iv) kg m–1s–2 e = permittivity,  = electric field, J = current
Choose the most appropriate answer from the density, L = length
options given below:
(A) (a)-(iii), (b)-(ii), (c)-(iv), (d)-(i) 2s cos 
(A) h  (B) W = 
(B) (a)-(iv), (b)-(ii), (c)-(i), (d)-(iii) prg
(C) (a)-(ii), (b)-(iii), (c)-(iv), (d)-(i) pa 2 E
(D) (a)-(iii), (b)-(ii), (c)-(i), (d)-(iv) (C) v  (D) J  
8L t
55. If force (F), length (L) and time (T) are taken as the 60. A student determined Young's Modulus of elasticity
fundamental quantities. Then what will be MgL3
dimension of density: [JEE Mains 2021] using the formula Y  . The value of g is
4bd3
–3
(A) [FL T ] 2 (B) [FL–3T3] taken to be 9.8 m/s2, without any significant error,
(C) [FL–4T2] (D) [FL–5T2] his observation are as following.
[JEE Mains 2021]
56. If velocity [V], time [T] and force [F] are chosen as
Least count of the
the base quantities, the dimensions of the mass will Physical Observed
Equipment used for
be: [JEE Mains 2021] Quantity Value
measurement
–1
(A) [FT V ] –1 (B) [FVT–1] Mass (M) 1g 2kg
(C) [FT2V] (D) [FTV–1] Length of bar (L) 1mm 1m
57. The diameter of a spherical bob is measured using a Breadth of bar (b) 0.1mm 4cm
vernier callipers. 9 divisions of the main scale, in Thickness of bar (d) 0.01mm 0.4cm
Depression () 0.01mm 5mm
the vernier callipers, are equal to 10 divisions of
vernier scale. One main scale division is 1 mm. The Then the fractional error in the measurement of Y is :
(A) 0.155 (B) 0.0083 (C) 0.0155 (D) 0.083
main scale reading is 10 mm and 8th division of
vernier scale was found to coincide exactly with one Question Previous Year (JEE Advanced)
of the main scale division. If the given vernier
1
callipers has positive zero error of 0.04 cm, then the 61. The dimension of   0E2 (0 : permittivity of
2
radius of the bob is ___ × 10–2 cm.
free space, E electric field) is– [IIT-2000]
[JEE Mains 2021] (A) MLT–1 (B) ML2T–2
(C) ML T–1 –2 (D) ML2T–1
58. Match List-I with List-II.
List-I List-II V
62. A quantity X is given by 0 L where 0 is
(a) Torque (i) MLT–1 t
(b) Impulse (ii) MT–2 permittivity of free space, L is length, V is a
(c) Tension (iii) ML2T–2 potential difference and t is a time interval. The
dimensional formula for x is same as that of-
(d) Surface tension (iv) MLT–2
[IIT-2001]
Choose the most appropriate answer from the otpion (A)Resistance (B) charge
given below: [JEE Mains 2021] (C)Voltage (D) current
(A) (a)-(iii), (b)-(iv), (c)-(i), (d)-(ii)
63. A cube has a side 1.2 × 10–2m. Its volume will be
(B) (a)-(i), (b)-(iii), (c)-(iv), (d)-(ii)
recorded as - [IIT-2003]
(C) (a)-(iii), (b)-(i), (c)-(iv), (d)-(ii)
(A) 1.728 × 10–6 m3 (B) 1.72 × 10–6m3
(D) (a)-(ii), (b)-(i), (c)-(iv), (d)-(iii) (C) 1.7 × 10–6 m3 (D) .72 × 10–6 m3

67
PHYSICS-X I-IIT-JEE UNIT AND DIMENSION
64. A wire is of mass (0.3 ± .003) gm. The radius is 69. Estimate the wavelength (in nm) at which plasma
(0.5 ± 0.005) cm and length is (6 ± .06) cm. The reflection will occur for metal having the density
maximum percentage error in density is –
[IIT-2004] of electrons N  4 × 1027 m–3. Take 0 = 10–11
(A) 3% (B) 4% (C) 8% (D) 16% and m  10–30, where these quantities are in
proper SI unit-
  z 
65. P= exp     Temperature (A) 800 (B) 600
  K B  (C) 300 (D)200
P  Pressure KB Boltzmann constant
z  Distance
Dimension of  is - [IIT-2004] 70. Students I, II and III perform an experiment for
(A) M0 L0 T0 (B) M–1L1T2 measuring the acceleration due to gravity (g)
(C) M0L2T0 (D) ML–1T–2 using pendulum. They use different lengths of the
66. Which of the following physical quantities do not pendulum and / or record time for different
have the same dimensions ? [IIT-2005] number of oscillations. The observations are
(A) Pressure, Young’s modulus, stress
(B) Electromotive force, voltage, potential shown in the table.
(C) Heat, Work, Energy Least count for length = 0.1 cm
(D) Electric dipole, electric field, flux Least count for time = 0.1 s
67. Which of the following physical quantities do not Length Total time
have the same dimensions ? [IIT-2005] Number of Time
of the for (n)
(A) Pressure, Young’s modulus, stress Student oscillations period
(B) Electromotive force, voltage, potential Pendulum oscillations
(n) (s)
(C) Heat, Work, Energy (cm) (s)
(D) Electric dipole, electric field, flux
I 64.0 8 128.0 16.0
Passage for Q.No.68&69
A dense collection of equal number of electrons and II 64.0 4 64.0 16.0
positive ions is called neutral plasma. Certain solids III 20.0 4 36.0 9.0
containing fixed positive ions surrounded by free If EI, EII and EIII are the percentage error in g, i.e.,
electrons can be treated as neutral plasma. Let 'N' be
the number density of free electrons, each of mass 'm'.  g 
 100  for students I, II and III, respectively.
When the electrons are subjected to an electric field,  g 
they are displaced relatively away from the heavy [IIT-JEE 2008]
positive ions. If the electric field becomes zero, the
electrons begin to oscillate about the positive ions with (A) EI = 0 (B) EI is minimum
a natural angular frequency 'p', which is called the (C) EI = EII (D) EII is maximum
plasma frequency. To sustain the oscillations, a time
varying electric field needs to be applied that has an 71. A student uses a simple pendulum 1 m length to
angular frequency , where a part of the energy is determine g, the acceleration due to gravity. He
absorbed and a part of it is reflected. As  approaches
uses a stop watch the least count of 1 sec for this
p, all the free electrons are set to resonance together
and all the energy is reflected. This is the explanation records 40 seconds for 20 oscillations. For this
of high reflectivity of metals. [IIT-2011] observation, which of the following statement(s)
is (are) true ? [IIT-2010]
68. Taking the electronic charge as 'e' and the (A) Error T in measuring T, the time period, is
permittivity as '0', use dimensional analysis to 0.05 seconds
determine the correct expression for p. (B) Error T in measuring T, the time period, is 1
Ne mε0 second
(A) (B) (C) Percentage error in the determination of g is
mε0 Ne
5%
Ne2 mε0 (D) Percentage error in the determination of g is
(C) (D)
mε0 Ne2 2.5 %

68
UNIT AND DIMENSION PHYSICS -X I-IIT-JEE
72. To find the distance d over which a signal can be correspond to 11 main scale divisions. The
seen clearly in foggy conditions, a railways readings of the two calipers are shown in the
engineer uses dimensional analysis and assumes figure. The measured values (in cm) by calipers
(C2)and (C2), respectively, are
that the distance depends on the mass density  of
[JEE Advance -2016]
the fog, intensity (power/area) S of the light from
2 3 4
the signal and its frequency ƒ. The engineer finds
that d is proportional to S1/n. The value of n is. C1
[IIT-2014]
0 5 100
73. Planck's constant h, speed of light c and
2 3 4
gravitational constant G are used to form a unit of
length L and a unit of mass M. Then the correct
C2
option(s) is (are) [JEE Advance -2015]
(A) M  c (B) M  G 0 5 100
(A) 2.85 and 2.82 (B) 2.87 and 2.83
(C) L  h (D) L  G (C)2.87 and 2.86 (D) 2.87 and 2.87
74. The energy of a system as a function of time t is
78. In an experiment to determine the acceleration
given as E(t) = A2 exp(–t), where  = 0.2 s–1. The due to gravity g, the formula used for the time
measurement of A has an error of 1.25 %. If the ( )
error in the measurement of time is 1.50 %, the period of periodic motion is= 2 . The
percentage error in the value of E(t) at t = 5 s is values of R and r are measured to be (60±1)mm
[JEE Advance -2015] and (10±1)mm, respectively. In five successive
75. In terms of potential difference V, electric current measurements, the time period is found to be
I, permittivity 0, permeability µ0, and speed of 0.52s, 0.56s, 0.57s, 0.54s and 0.59s. The least
count of the watch used for the measurement of
light c, the dimensionally correct equation (s) is time period is 0.01s. Which of the following
(are) [JEE Advance -2015] statement(s) is (are) true?
(A) µ0I2 = 0V2 (B)0I = µ0V [JEE Advance -2016]
(C) I = 0cV (D) µ0cI = 0V (A) The error in the measurement of r is 10%
(B) The error in the measurement of T is3.75%
(C) The error in the measurement of T is 2%
76. A length-scale () depends on the permittivity () (D) The error in the determined value of g is11%
of a dielectric material, Boltzmann constant (kB), 79. Consider an expanding sphere of instantaneous
the absolute temperature (T), the number per unit radius R whose total mass remains constant. The
volume (n) of certain charged particles, and the expansion is such that the instantaneous density 
charge (q) carried by each of the particles. Which remains uniform throughout the volume. The rate
of the following expressions (s) for  is (are)  1 d 
dimensionally correct?[JEE Advance -2016] of fractional change in density   is
  dt 
 nq 2   k T  constant. The velocity  of any point on the
(A) =   (B) =  B2 
 k B T   nq  surface of the expanding sphere is proportional to
[JEE Advance -2017]
 q2   q2  1
(C) =  2 /3 (D) =  1/ 3 (C) R 2/3 (D) R 3
  n k T    n k T  (A) R (B)
R
 B   B 
80. The relation between [E] and [B] is
77. There are two Venire calipers both of which have [JEE Advance -2018]
1 cm divided into 10 equal divisions on the main
scale. The Venire scale of one of the calipers (C1) (A) [E] = [B] [L] [T]
has 10 equal divisions that correspond to 9main (B) [E] = [B] [L]–1 [T]
scale divisions. The Venire scale of the other (C) [E] = [B] [L] [T]–1
caliper (C2) has 10 equal divisions that (D) [E] = [B] [L]–1 [T]–1

69
PHYSICS-X I-IIT-JEE UNIT AND DIMENSION

81. The relation between [0] and [0] is 84. Let us consider a system of units in which mass
[JEE Advance -2018] and angular momentum are dimensionless. If
2 –2
(A) [0] = [0] [L] [T] length has dimension of L, which of the
(B) [0] = [0] [L]–2 [T]2 following statement(s) is/are correct ?
(C) [0] = [0]–1 [L]2 [T]–2 [JEE Advance -2019]
(D) [0] = [0] –1 [L]–2 [T]2 (A) The dimension of force is L–3
(B) The dimension of Power is L–5
PARAGRAPH (Q.No.82 & 84) (C) The dimension of energy is L–2
If the measurement errors in all the independent (D) The dimension of linear momentum is L–1
quantities are know, then it is possible to determine the
error in any dependent quantity. This is done by the use 85. Sometimes it is convenient to construct a system
of series expansion and truncating the expansion at the of units so that all quantities can be expressed in
first power expansion and truncating the expansion at terms of only one physical quantity. In one such
the first power of the error. For example consider the system, dimensions of different quantities are
relation z = x/y. If the errors in x, y and are x, y and given in terms of a quantity X as follows:
z, respectively, then
[position] = [X]; [speed] = [X];
z  z 
x  x x  x   y 
 1 [acceleration] = [Xp];
1  
y  y y  x   y  [Linear momentum] = [Xq];
1
[Force] = [Xr]. Then - [JEE Advance 2020]
 y  (A) + p = 2 (B) p + q – r = 
The series expansion for 1   , to fist power in
 y  (C) p – q + r =  (D) p + q + r = 
Dy/y, is 1 ( y / y) . The relative error in independent 86. The smallest division on the main scale of a
variables are always added. So ther error in z will be Vernier calipers is 0.1 cm. Ten divisions of the
 x y  Vernier scale correspond to nine divisions of the
z  z   . main scale. The figure below on the left shows
 x y 
the reading of this calipers with no gap between
The above derivation makes the assumption that x/x its two jaws. The figure on the right shows the
<<1, y/y <<1. Therefore, the higher power of these reading with a solid sphere held between the
quantities are neglected. [JEE Advance -2018] jaws. The correct diameter of the sphere is [JEE
Advance -2021]
1 a
Consider the ratio r =   0 main scale 1 3 main scale 4
82.  to be determined
1 a 
by measuring a dimensionless quantity a. If the
error in the measurement of a is a (a/a << 1),
then what is the error r in determining r?
0 Vernier scale 10 0 Vernier scale 10
[JEE Advance -2018]
a 2 a (A) 3.07 cm (B) 3.11 cm
(A) (B)
(1  a ) 2 (1  a ) 2 (C) 3.15 cm (D) 3.17 cm
 
(C)
2 a
(D)
2a  a   EB
(1  a 2 ) (1  a 2 ) 87. A physical quantity S is defined as S  ,
0
where is electric field, is magnetic field and µ0 is
83. In an experiment the initial number of radioactive
the permeability of free space. The dimensions of
nuclei is 3000. It is found that 1000  40 nuclei 
decayed in the first 1.0 s. For |x| << 1, 1n (1 + x) S are the same as the dimensions of which of the
= x up to first power in x. The error in the following quantity (ies) ? [JEE Advance -2021]
determination of the decay constant , in s-1 , is Energy Force
[JEE Advance -2018] (A) (B)
charge×current Length×Time
(A) 0.04 (B) 0.03
(C) 0.02 (D) 0.01 Energy Power
(C) (D)
Volume Area

70
UNIT AND DIMENSION PHYSICS-X I-IIT-JEE

ANSWER KEY
EXERCISE-1

Qus. 1 2 3 4 5 6 7 8 9 10 11 12 13 14 15 16 17 18 19 20
Ans. D D B D A A A A B B C B B C D B A A C B
Qus. 21 22 23 24 25 26 27 28 29 30 31 32 33 34 35 36 37 38 39 40
Ans. C A B A C C B A A D A D D C C B B D A B
Qus. 41 42 43 44 45 46 47 48 49 50 51 52 53 54 55 56 57 58 59 60
Ans. A D B C D B A B B A A B B A D B C B A C
Qus. 61 62 63 64 65 66 67 68 69 70 71 72 73 74 75 76 77 78 79 80
Ans. B B B D B B C B A B A C D A B B C C B D
Qus. 81 82 83 84 85
Ans. C D C A D
EXERCISE-2

Qus. 1 2 3 4 5 6 7 8 9 10 11 12 13 14 15
Ans. A,D A,B,C,D A,B,C,D B,C A,B,C B,C,DA,B,C B,D A,C A,C,D A,B,C A,B,C A,B,C,D A,C A,B,C
Qus. 16 17 18 19 20 21 22 23 24 25 26 27 28 29 30
Ans. A,B,C B,C A B C B A C B C B A A C A
Qus. 31 32 33 34 35 36 37 38 39 40
Ans. C C D A D C D B B B

EXERCISE-3
1. (A)(q), (B)(p),(s)(C)(q), (D)(r) 2. (A)(r) ; (B)(s), (C)(p), (D)(q)
3. (A)(r), (B)(p), (C)(s), (D)(q) 4. (A)(s), (B)(p), (r), (C)(q), (D)(r), (s)
5. (A)(p), (B)(p), (C)(s), (D)(r) 6. (A)(s), (q)(B)(q)(C)(p), (D)(r)
7. (A)(q), (B)(p), (C)(r), (D)(s) 8. (A)(q), (B)(r), (C)(s), (D)(p)
9. (A)(r), (B)(u), (C)(p), (D)(t)(E). (q)(F)(s)
10. (A)(p),(q), (B)(p),(t)(C)(s)
11. (D) 12. (D) 13. (D) 14. (C) 15. (C) 16. (5) 17. (0) 18. (8)
19. (6) 20. (A) 21. (B) 22. (A) 23. (5) 24. (7) 25. (B) 26. (B)

EXERCISE-4
1. (B) 2. (A) 3. (B) 4. (C) 5. (C) 6. (D) 7. (D) 8. (B) 9. (D)
10. (D) 11. (D) 12. (D) 13. (C) 14. (B) 15. (A) 16. (A) 17. (C) 18. (C)
19. (C) 20. (A) 21. (D) 22. (C) 23. (C) 24. (B) 25. (D) 26. (D) 27. (D)
28. (A) 29. (B) 30. (B) 31. (C) 32. (B) 33. (B) 34. (C) 35. (C) 36. (C)
37. (D) 38. (B) 39. (D) 40. (D) 41. (D) 42. (C) 43. (B) 44. 1050 45. (C)
46. (12) 47. (D) 48. (C) 49. (D) 50. (D) 51. (D) 52. (C) 53. (B) 54. (A)
55. (C) 56. (D) 57. (52) 58. (C) 59. (C) 60. (C) 61. (C) 62. (A,D) 63. (C)
64. (B) 65. (C) 66. (A) 67. (A) 68. (C) 69. (B) 70. (B) 71. (A,C) 72. (C)
73. (A,C,D) 74. (D) 75. (A,C) 76. (B,D) 77. (B) 78. (A,B,D) 79. (A) 80. (C)
81. (D) 82. (B) 83. (C) 84. (A,B,C) 85. (A,B) 86. (C) 87. (B,D)

71
MOTION I N ONE DIMEN SIO N PHYSICS-X I-IIT-JEE

CHAPTER

MOTION IN ONE DIMENSION


1. DISTANCE 3. COMPARATIVE STUDY OF DISTANCE AND
Distance is the actual length of the path. It is the DISPLACEMENT
characteristic property of any path i.e. path is
Distance is the actual path travelled by a moving
always associated when we consider distance
between two positions. Distance between A and B body, while displacement is the change in the
while moving through path position.
(1) May or may not be equal to the distance A S
between A and B while moving through path r
ri
(2). B
(1) rf
O

B In the above figure distance travelled is S, while


A
 
(2) displacement is r  rf  ri
(i) It is a scalar quantity
0 1 0 1. Regarding distance and displacement it is worth
(ii) Dimension : [M L T ]
(iii) Unit : In C.G.S. centimeter (cm), In M.K.S. (m) noting that:
(a) Distance is scalar, while displacement is vector
2. DISPLACEMENT both having same dimensions [L] and same SI unit
Displacement of a particle is a position vector of its metre.
final position w.r.t. initial position. (b) The magnitude of displacement is equal to
Displacement =
minimum possible distance so,
AB = (x2 – x1) î + (y2 – y1) ĵ + (z2 – z1) k̂ Distance  |Displacement|
It is the characteristic property of any point i.e. (c) For motion between two points displacement is
depends only on final and initial positions.
single valued, while distance depends on actual
To Determine The Position Vector :
A (x1, y1, z1) path and so can have many values.
(d) For a moving particle distance can never decreases
 with time while displacement can. Decrease in
rA
B (x2, y2, z2) displacement means body is moving towards the
 initial position.
rB
O (e) For moving particle distance can never be negative
Position vector of A w.r.t. O  OA or zero, while displacement can be. (Zero
 displacement means that body after motion has
 rA  x1î  y1ˆj  z1k̂
come back to initial position)
Position Vector of B w.r.t. O = OB
 Distance > 0 but |Displacement| > = or < 0
 rB  x 2 î  y 2 ĵ  z 2 k̂ (f) In general magnitude of displacement is not equal
Position vector of B w.r.t. A; to distance. However it can be so if the motion is
AB  (x 2  x1 )î  ( y 2  y1 ) ĵ  (z 2  z1 )k̂ along a straight line without change in direction.

72
PHYSICS-X I-IIT-JEE MOTION I N ONE DIMEN SIO N

Comparative study of distance & displacement : Example-1


Distance = r , Which of the following graph(s) is / are not possible ?
|Displacement| = 2r
|Displacement| = s

Distance

Distance
(a) (b)
0 Time 0 Time

Displacement

Displacement
A r B

(c) (d)

0 t1 Time 0 Time
Solution.
h
The (a) graph shows that with increase in time distance
first increases and then decreases. However, distance
can never decrease with time so this graph is not
A B physically possible. The graph (c) shows that at certain
instant of time (t1) body is present at two positions. Also
Distance = 2h, Displacement = 0
it shows that time first increases then decreases. These
A conditions are not possible physically.
4. SPEED
S It is the distance covered by the particle in one
second.
(i) It is a scalar quantity
B
(ii) Unit : In M.K.S. Meter/Second or km/sec. In C.G.S.
Distance = s, |Displacement| = s cm/sec
0 1 –1
Note : Distance and Displacement, while moving in (iii) Dimension : [M L T ]
a circle from A to B and then from B to A.  Types of speed :
(a) Instantaneous speed
(b) Average speed
A R R B (c) Uniform speed
(d) Non-uniform speed
(a) Instantaneous Speed :
It is the speed of a particle at particular instant.
Half Cycle Full Cycle
Distance S dS
R 2R Instantaneous speed = lim  
Displacement 2R 0 t  0 t dt
Direction 1. A  B, Total distance
When particle (b) Average speed =
Total time
moves from
A to B (c) Uniform speed : If during the entire motion speed
2. B  A, of the body remains same, the body is said to have
When particle
uniform speed.
moves from
B to A (d) Non-uniform speed : If speed changes, the body
is said to have non-uniform speed.

73
MOTION I N ONE DIMEN SIO N PHYSICS-X I-IIT-JEE

5. VELOCITY This is the velocity at time t. Initially t = 0,


It is defined as rate of change of displacement.  v = – 5 m/s
(i) It is a vector quantity Hence correct answer is (b).
(ii) Its direction is same as that of displacement 
Note: Hence speed will be | v| = 5 m/s.
(iii) Unit and dimension: Same as that of speed
 Types of Velocity: Example-3
(a) Instantaneous velocity
(b) Average velocity The displacement of a particle moving in
(c) Uniform velocity one-dimensional direction under a force at time t is
(d) Non-uniform velocity given by t = x + 3, where x is in m and t in sec. The
(a) Instantaneous velocity : It is defined as the displacement of the particle, when its velocity is zero,
velocity at some particular instant. will be-
 
r dr (a) 0 (b) 3m (c) –3m (d) 2m
Instantaneous velocity  lim 
t  0  t dt Solution.
(b) Average velocity:
Given t = x + 3
Total displacement 2
Average velocity =  x = t – 6t + 9 
Total time
dx
(c) Uniform velocity : A particle is said to have  = 2t– 6 
uniform velocity, if magnitudes as well as dt
direction of its velocity remains same and this dx
 Instantaneous velocity, v = = 2t – 6
is possible only when the particles moves in dt
same straight line reversing its direction. when v = 0, 2t – 6 = 0  t = 3 sec. Thus at t=3
(d) Non-uniform velocity: A particle is said to 2
sec, x = (t – 6t + 9) = 0
have non-uniform velocity, if either of Hence correct answer is (a).
magnitude or direction of velocity changes
(or both changes).
Note:
(a) Velocity is a vector while speed is a scalar having
6. COMPARATIVE STUDY OF INSTANTANEOUS –1
same units (m/s) and dimension [LT ]
SPEED AND INSTANTANEOUS VELOCITY
(b) If during motion velocity remains constant
Instantaneous velocity or simply velocity is defined
as rate of change of particle's position with time throughout a given interval of time, the motion is
  
 r d r  said to be uniform and for uniform motion, v=
v  lim  where the position r of a particle 
t 0 t dt constant = vav
 
at any instant changes by  r in a small time t However converse may or may not be true i.e. If v
The magnitude of velocity is called speed i.e. speed 
 = vav , the motion may or may not be uniform.
= | velocity| i.e. v = | v|
Note: In straight line motion there is no change in
 (c) If velocity is constant, speed (= | velocity |) will also
direction so vand v both have same meaning.
be constant. However converse may or may not be
true i.e. if speed = constant, velocity may or may not
Example-2
be constant as velocity has a direction in addition to
A particle moves along the x-axis in such a way that its
magnitude which may or may not change. e.g. in
x-coordinates varies with time as x = 2 – 5t + 6t2. What 
case of uniform rectilinear motion. v = constant and
will be its initial velocity? 
(a) 5 m/s (b) –5 m/s (c) 2 m/s (d) –2 m/s so speed | v| = constant while in case of uniform

Solution. circular motion, v = constant but v  constant due
2 to change in direction.
Here displacement, x = 2 – 5t + 6t
The velocity at any instant t is given by (d) Velocity can be positive or negative, as it is a vector
dx d 2 but speed can never be negative as it is the
v=  (2 – 5t + 6t ) 
dt dt magnitude of velocity
 v = – 5 + 12 t 
i.e. v = | v|

74
PHYSICS-X I-IIT-JEE MOTION I N ONE DIMEN SIO N
(e) If displacement is given as a function of time, the v
time derivative of displacement will give velocity
and modulus of velocity gives speed.
II
e.g. s = A0 – A1t + A2t ,
2
0 t2 t3
t
t1
ds I III
v= = – A1 + 2A2t. So, initially (t = 0), velocity
dt
= – A1, while speed = |–A1| = A1
 Special Note: e.g. From the adjoining v-t graph. The distance
ds s s travelled by body in time t3 = Area I + Area II +
It is common misconception, that = , Here
dt t t Area III and the displacement of body = Area II –
A0 Area III – Area I
= – A1 + A2t Which is totally different from
t
ds Example-4
the above value of .
dt Can a body have uniform speed but non-uniform
ds velocity?
(f) As by definition, v = , the slope of displacement
dt (a) Yes
versus time graph gives velocity. (b) No
(c) Depend on direction
(d) Unpredictable
Displacement

Solution:
Yes, hence correct answer is (a).
e.g. Speed of a particle in circular path is constant but
 due to change in direction its velocity changes.
O t Time
Example-5
ds
i.e. v = = tan  = slope of s-t curve State whether the following graph can be seen in nature
dt
or not. Explain.
ds
(g) As, v =  ds = vdt
dt
From figure vdt = dA. so, dA = ds 
Speed

  s = dA =
  vdt t2 0 t3 Time
velocity

(a) Yes
v (b) No
(c) Sometime
(d) At a particular instant
O time Solution.
t
Area under velocity versus time graph with proper This graph shows that speed is negative for some
algebraic sign gives displacement while without interval of time (t2 to t3). Since speed can never be
sign gives distance. negative, so this graph is physically not possible.
Hence correct answer is (b).

75
MOTION I N ONE DIMEN SIO N PHYSICS-X I-IIT-JEE

Example-6 Example-8
Out of the following graph(s), which is / are not From the adjoining graph, the distance traversed by
possible ? particle in 4 sec, is-
v(m/s)

Velocity
Time

(a) (b) 20

0 0 10
Position Time
0 1 2 3 4
Time (sec)
Velocity

(a) 60 m (b) 25 m
(c) (d)
Velocity

(c) 55 m (d) 30 m
0 t4 t5
Time Solution.
0 Time The given graph can be drawn as shown in figure
Distance travelled = Area under v-t graph
Solution.
The graph (a) shows that on increasing position (x), time first (m/s) A B
increases, then decreases, which is impossible. The v 20
III C
graph (c) shows that at a given instant of time (t4) 10 G' D
I II IV V
particle has two velocities. Also it shows that at time H G F E
 0 1 2 3 4
(t5) the acceleration is infinite (= slope of v/t curve). Time (sec)
Since both these conditions cannot be achieved = Area I + Area II + Area III + Area IV + Area V
practically, then these graphs are not possible. = (1/2) (OH × AH) + HG × AH
Hence correct answer is (a) and (c).
+ 1/2 (G'C × BG')+ (GF × GG') + (EF × CF)
Example-7 = (1/2) (1 × 20) + (1 × 20) + 1/2 (1 × 10) + (1 × 10)
From the adjoining displacement-time graph for two + (1 × 10) = 10 + 20 + 5 + 10 + 10 = 55 m
particles A & B the ratio of velocities vA : vB will be- Hence correct answer is (c).
Y
B 7. COMPARATIVE STUDY OF AVERAGE SPEED
Displacement

AND AVERAGE VELOCITY


A
The average speed of a particle for a given interval
60º
of time is defined as the ratio of distance travelled

30º to the time taken, while average velocity is defined


X
time as the ratio of displacement to time taken.
(a) 1 : 2 (b) 1 : 3 Distance traveled
Average speed =
(c) 3 : 1 (d) 1 : 3 Time taken
Solution. s
i.e. vav =
The line having greater slope has greater velocity, hence t
the line making an angle 60º with time axis has greater Displacement
Average velocity =
v A tan 30º 1 / 3 1 Time taken
velocity. Now,   
v B tan 60º 3 3 
 r
i.e v av 
Hence correct answer is (d). t

76
PHYSICS-X I-IIT-JEE MOTION I N ONE DIMEN SIO N

Note: (i) If a particle travels at speeds v1, v2 etc. for intervals


(a) Average speed is a scalar, while average velocity is t1 , t2 etc. respectively, then
a vector both having same unit s v t  v t  ............ vi t i
(m/s) vav = = 11 2 2 
t t1  t 2  ............. t i
0 –1
(b) Both have dimension [M LT ] If t1 = t2 = ............. = tn = t
(c) For a given time interval average velocity is single v1  v 2  ......... 1
then vav =   vi
valued, while average speed can have many values n n
depending on path followed. i.e. average speed is arithmetic mean of individual
(d) If after motion body comes back to its initial speeds.
  

position vav = 0 [as  r = 0], but vav > 0 and finite  Special Note:
(as s > 0) If a particle moves for two equal time-intervals
(e) For a moving body average speed can never be v1  v 2
vav =
negative or zero (unless t  ), while average 2
velocity can be i.e.
 Example-9
vav > 0 while vav > = or < 0
A car travels first half distance between two places with
(f) In general average speed is not equal to magnitude a speed of 40 km/h and the rest half distance with a

of average velocity (as s  | r |). However it can speed of 60 km/h. The average speed of the car will be-
be so if the motion is along a straight line without (a) 100 km/hr (b) 50 km/hr

change in direction (as s = | r |). (c) 48 km/hr (d) 200 km/hr
Solution.
(g) If a graph is plotted between distance (or Let the total distance travelled be x.
displacement) and time, the slope of chord during a Time taken to travel first half distance
given time interval gives average speed (or x/2 x
velocity) t1 = = hr
40 80
Time taken to travel the rest half distance
B
Displacement

s2 x/2 x
t2 = = hr

s 60 120
s1 A
Total distance
t  Average speed =
Total time
t1 t2 x
Time = = 48 km/hr
( x / 80)  ( x / 120)
s Hence correct answer is (c).
vav = = tan  = slope of chord
t
Example-10
(h) If a particle travels distances L1, L2, L3 etc at speeds
v1, v2, v3 etc. respectively, then A table has its minute hand 4.0 cm long. The average
s L1  L 2  .........  L n L i velocity of the tip of the minute hand between 6.00 a.m.
vav = =   to 6.30 a.m. and 6.00 a.m. to 6.30 p.m. will respectively
t L1 L 2 Ln L
  .......... .   i be- (in cm/s)
v1 v 2 vn vi –3 –4
(a) 4.4 × 10 , 1.8 × 10
 If L1 = L2 ––––––– = Ln = L –4 –3
(b) 1.8 × 10 , 4.4 × 10
1 1 1 1  1 1 –3
(c) 8 × 10 , 4.4 × 10
–3
then =    ...... = 
vav n  v1 v2  n vi –3
(d) 4.4 × 10 , 8 × 10
–4

 Special Note: Solution.


If a particle moves a distance at speed v1 and comes At 6.00 a.m. the tip of the minute hand is at
0
back with speed v2, 12 marks and at 6.30 a.m. or 6.30 p.m. it is 180 away.
2v v  Thus the straight line distance between the initial and
Then vav = 1 2 while vav = 0 final positions of the tip is equal to the diameter of the
v1  v2 clock.
[as displacement = 0] Displacement = 2 R = 2 × 4 cm = 8 cm
Time taken from 6 a. m. to 6.30 a.m. is 30 minutes

77
MOTION I N ONE DIMEN SIO N PHYSICS-X I-IIT-JEE

= 1800 s. The average velocity is When only When only When both the
Displacement 8.0 cm –3 direction magnitude direction changes
vav=  = 4.4×10 cm/s
time 1800 s changes and magnitude
change
Again time taken from 6 am to 6.30 p.m.
To change the In this case, net In this case, net
= 12 hrs + 30 minutes = 45000 s
direction net force or net force or net
Displaceme nt 8 –4
acceleration or acceleration acceleration has
 vav =  = 1.8 × 10 cm/s
time 45000 net force should be two components.
Hence correct answer is (a). should be parallel or anti- One component is
perpendicular parallel to the parallel or anti-
Example-11 to direction of direction of parallel to velocity
The average speed and average velocity during one velocity velocity. and another one is
complete cycle of radius R will respectively be- Example: (straight line perpendicular to
(T is the time to take one complete revolution) Uniform motion) velocity
R 2R R circular Example: Example :
(a) ,0 (b) , motion When ball is Projectile motion
T T T
2R 2R
thrown up under
(c) ,0 (d) 0, gravity
T T
Solution.
 Types of acceleration:
2R
Average speed vav = and (a) Instantaneous acceleration:
T It is defined as the acceleration of a body at some

average velocity vav = 0/T = 0 particular instant.
Hence correct answer is (c). Instantaneous acceleration
 
v dv
= lim 
Example-12 t 0 t dt
A boy covers a distance AB of 2 km with speed of 2.5
(b) Average acceleration:
km/h, while going from A to B and comes back from B   
 v v 2  v1
to A with speed 0.5 km/hr, his average speed will be- a av  
t t 2  t1
(a) 1.5 km/hr (b) 0.83 km/hr
(c) 1.2 km/hr (d) 3 km/hr
Solution. (c) Uniform acceleration:
A body is said to have uniform acceleration if
As boy goes from A to B and then comes back from B
magnitude and direction of the acceleration remains
to A hence his average speed
constant during particle motion.
2v1v 2 2  2.5  0.5
vav =  Note:
v1  v 2 2.5  0.5
If a particle is moving with uniform acceleration,
2. 5 this does not necessarily imply that particle is
= = 0.8 km/hr
3 moving in straight line.
Hence correct answer is (b). Example : Parabolic motion
8. ACCELERATION (d) Non-uniform acceleration:
It is defined as the rate of change of velocity. A body is said to have non-uniform acceleration, if
(i) It is a vector quantity. magnitude or direction or both change during
(ii) Its direction is same as that of change in velocity motion.
and not of the velocity (That is why acceleration in
circular motion is towards the centre) Note: 
(iii) There are three ways possible in which change in (i) Acceleration is a vector with dimensions
–2 2
velocity may occur [LT ] and SI units (m/s )

78
PHYSICS-X I-IIT-JEE MOTION I N ONE DIMEN SIO N

(ii) If acceleration is zero, velocity will be constant Solution.


and motion will be uniform. 2
x = a0 – a1t + a2t 
(iii) However if acceleration is constant then
dx
acceleration is uniform but motion is non-  = – a1 + 2a2t 
dt
uniform and if acceleration is not constant then
both motion and acceleration are non-uniform. d 2x
 = 2a2
 dt 2
(iv) If a force F acts on a particle of mass m then
  Hence correct answer is (c).
by Newton's II law a = F/m
(v) As by definition Example-14

 ds If the displacement of a particle is proportional to the
v
dt square of time, then-
  
 dv d  d s  d 2 s (a) Velocity is inversely proportional to t
so, a     2
dt dt  dt  dt (b) Velocity is proportional to t

i.e. if s is given as a function of time, second (c) Velocity is proportional to t
time derivative of displacement gives (d) Acceleration is constant
acceleration.
Solution.
(vi) If velocity is given as function of position then
Given that s  t2  s = kt2, where k is constant
by chain rule
dv dv dx ds
a  .   velocity v = = 2kt, velocity varies with time
dt dx dt dt
dv dx dv
   av [ as  v] acceleration a = = 2k = constant.
dx dt dt
 Hence acceleration of particle is constant
 dv
(vii) As acceleration a  the slope of velocity - Hence correct answer are (b) & (d).
dt
time graph gives acceleration i.e.
 Example-15
 dv
a = tan  2
dt The displacement is given by x = 2t + t + 5, the
 
da acceleration at t = 5 sec will be-
(viii) The slope of a -t curve, i.e. is a measure of 2 2
dt (a) 8 m/s (b) 12 m/s
rate of non-uniformity of acceleration. 2 2
(c) 15 m/s (d) 4 m/s
However we do not define this physical
quantity as it is not involved in basic laws or Solution.
equation of motion. 2 dx
Given, x = 2t + t +5  =4t+1
dt
(ix) Acceleration can be positive or negative.
Positive acceleration means velocity is d 2x  d 2x  2
 =4   = 4 m/s
increasing with time while negative dt 2  dt 2 
 t  5sec
acceleration called retardation means velocity
is decreasing with time. Hence correct answer is (d).
Example-13 Example-16
The displacement x of a particle along a straight line at The position x of a particle varies with time (t) as x =
2 3
time t is given by x = a0 – a1t + a2t2. The acceleration of at – bt . The acceleration of the particle will be equal
the particle is- to zero at time –
(a) a0 (b) a1 2a a a
(a) (b) (c) (d) 0
(c) 2a2 (d) a2 3b b 3b

79
MOTION I N ONE DIMEN SIO N PHYSICS-X I-IIT-JEE

Solution.
2 3 Example-19
Given that x = at – bt 
dx 2 A car accelerates from rest at a constant rate  for
 Velocity v = = 2at – 3bt and
dt sometime after which it decelerates at constant rate  to
d  dx  come to rest. If the total time elapsed is t sec. The
acceleration a =   
dt  dt  maximum velocity of car will be-
2a a
0 = 2a – 6bt  t =  vmax
6b 3b A
Hence correct answer is (c).
v
Example-17
In the above example, the average acceleration of the tan–1  tan–1  B
0 t2
particle in the interval t = 1 to t = 3 sec will be- t1
t
(a) 12 a – 2b (b) 2b – 12 a
(c) 2a – 12b (d) 12b – 2a   
(a) (b) t
Solution. t (   ) 
In the light of above example, we have  
(c) t (d)
dx 2   (   )
= 2at – 3bt
dt Solution.
Now velocity at t = 1 sec, If the car accelerates for time t1 and decelerates for time
 dx  t2, then according to given problems
v1 =   = 2a – 3b and
 dt  t 1 t = t1 + t2 …. (1)
 dx  If vmax is the maximum velocity of the car, then from v/t
that at t = 3 sec, v2 =   = 6a – 27 b
 dt  t 3 curve, we have
v 2  v1 vmax v
Thus average acceleration aav = = ,  = max
t 2  t1 t1 t2
6a  27b  2a  3b 4a  24b [as slope of v/t curve gives acceleration.]
=  = 2a – 12 b
3 1 2
 1 1 t t 
Hence correct answer is (c). so    = 1 2  vmax = t
  v max (   )
Example-18
[ t = t1 + t2]
The velocity v of a moving particle varies with
displacement as x = v  1 , the acceleration of the Hence correct answer is (c).
particle at x = 5 unit will be- Special Note : In the above example the total distance
(a) 6 unit (b) 24 unit travelled by car, s = 1/2 (area  OAB)
(c) 240 unit (d) 25 unit 
s = (1/2) (vmax) t = (1/2) t [as area of v/t
Solution.  
2 2 graph gives total distance covered]
x = v  1  x = v + 1  v = x –1
dv Example-20
 = 2x. Now acceleration
dx The retardation of a moving particle, if the relation
dv 2 2
a=v (x – 1). 2x between time and position is t = Ax + Bx (where A and
dx
This is the acceleration at position x. Now at B are constant) will be-
2 –3 –3
x = 5 unit, a = (5 –1) (2 × 5) = 240 unit (a) 2A (Ax + B) (b) 2A (2Ax + B)
–3 –3
Hence correct answer is (c). (c) A/2 (Ax + B) (d) A/2 [2Ax + B]

80
PHYSICS-X I-IIT-JEE MOTION I N ONE DIMEN SIO N
  
Solution.
2
(c) From (i) and (ii) s  u t + (1/2) a t2 ....(iii)
As t = Ax + Bx  dt/dx = 2Ax + B Again from (i) and (iii)
–1   
 v = (2Ax + B) … (1) s  v t  (1 / 2 ) a t 2
[as dx/dt = v], Now by chain rule
[Here negative sign does not indicate that
dv dv dx dv
a  . v  retardation is occurring]
dt dx dt dx   
–1 d –1 (d) From (i) and (ii) v2 = u2 + 2 a.s .... (iv)
 a = (2Ax + B) (2Ax + B)  th
dx s n = displacement of particle in n second
–3  
= – 2 A (2Ax + B) = s n – sn 1
So retardation = – a = 2A (2Ax + B)
–3   2 
= { u(n) + (1/2) a n } – { u (n –1)
Hence correct answer is (c).  2
+ (1/2) a (n – 1) }
  
Example-21 s n = u + 1/2 a (2n – 1)
It is possible to be accelerating if you are travelling at [This equation is dimensionally non balanced
constant speed? Is it possible to round a curve, with zero because we have substituted value of t = 1s and
acceleration? With constant acceleration? With variable second is neglected that's why it seems to be
acceleration? unbalanced]
(a) No, yes, no, no (b) Yes, no, yes, yes Equations (i), (iii) and (iv) one called 'equations of
(c) Yes, no, no, no (d) No, no, yes, yes motion' and are very useful in solving the problems
Solution. of motion along a straight line with constant
If speed is constant, velocity may change due to change acceleration.
in direction and as acceleration is rate of change of
Note :
velocity so acceleration may not be zero when speed is
(i) These equations can be applied only and only when
constant. Actually in uniform circular motion, speed =
acceleration is constant. In case of circular motion
constant but acceleration  0.
For motion on a curve we at least have to change the or simple harmonic motion as acceleration is not
direction of motion, so we will require a force and constant (due to change in direction or magnitude)
hence, acceleration i.e. it is not possible to round a so these equation cannot be applied.
  
curve with zero acceleration. However, in rounding a (ii) v= u + at
curve acceleration may be constant or variable. In case   2
 and s = ut + (1/2) a t
of projectile motion acceleration is constant (= g ) while are vector equation, while
in case of circular motion acceleration  constant, either        
v . v  u . u  2a . s  v 2  u 2  2 a . s
due to change in direction or both change in direction
and magnitude. is a scalar equation
Hence correct answer is (b). (iii) If the velocity and acceleration are collinear, we
conventionally take the direction of motion to be
9. MOTION WITH UNIFORM ACCELERATION positive, so equation of motions becomes
  2 2
v = u + at, s = ut + (1/2) at , v = u + 2as
2
Let u = Initial velocity (at t = 0), v = Velocity of
the particle after time t If the velocity and acceleration are anti-parallel
  then,
a = Acceleration (uniform), s = Displacement of the
particle during time 't' v = u – at
  2 2 2
s = ut – (1/2) at = v = u – 2as
 vu
(a) Acceleration, a [Because of uniform
t
acceleration, this acceleration is instantaneous as Example-22
well average acceleration]. From above equation
   A particle starts with an initial velocity 2.5 m/s along
v  u  at ....(i) the positive x-direction and it accelerates uniformly at
 2
(b) Displacement s = Average velocity × time, the rate 0.50 m/s . Time taken to reach the velocity 7.5
 
 uv m/s will be-
s t .... (ii)
2 (a) 5 s (b) 2 s
[This is very useful equation, when acceleration is (c) 10 s (d) 15 s
not given]

81
MOTION I N ONE DIMEN SIO N PHYSICS-X I-IIT-JEE

Solution. Solution.
We have v = u + at or 7.5 = 2.5 + 0.50 t  Let the passenger catch the bus after time t. From
2
 t = 10 s "s = ut + (1/2) at ", the distance travelled by the bus
Hence correct answer is (c) 2
s1 = 0 + (1/2) at … (1)
Example-23 and the distance travelled by the passenger
A particle starts with an initial velocity 2.5 m/s along the s2 = ut + 0 … (2)
positive x-direction and it accelerates uniformly at the rate [Note : acceleration of passenger = 0]
Now the passenger will catch the bus if,
0.50 m/s2. The distance travelled by the particle in first two
d + s1 = s2 … (3)
seconds will be-
In the light of eq. (1) & (2), eq. (3) gives
(a) 4m (b) 5m
(c) 1m (d) 6 m 2 u  u 2  2ad
(1/2) at – ut + d = 0  t =
Solution. a
2
We have, s = ut + (1/2) at
2 So the passenger will catch the bus if t is real i.e. u 
= (2.5) (2) + (1/2) (0.50) (2) = 6m
2 2ad  u  2 ad

Since the particle does not return back, it is also the So, the minimum speed of passenger for catching the
distance travelled. bus is 2 ad
Hence correct answer is (d) Hence correct answer is (c)
Example-24
Example-26
A car accelerates from rest at a constant rate  for
A body moving with uniform acceleration describes 4 m
sometime after which it decelerates at constant rate  to
in 3rd second and 12 m in the 5th second. The distance
come to rest. If the total time elapsed is t sec. What will
described in next three second is-
be the total distance traveled ?
(a) 100 m (b) 80 m (c) 60 m (d) 20 m
 t 2  2 Solution.
(a) (b) t
   Let u is the initial velocity and a is the acceleration then
 2  t 2 2 Sn = u + (1/2) a(2n – 1)   
(c) (1/2) t (d) 2t
(   )   S3= u + (1/2) a(3 × 2 –1)
Solution. 5
2  4=u+ a … (i)
From " s = ut + (1/2) at ", 2
2 2
we have s1 = (1/2) t1 , s2 = vt2 – (1/2) t2 similarly for 5th second
 t 2 S5 = u+(1/2) a (2×5–1)
Total distance = s1 + s2 = (1/2)
   12 = u + (9/2) a … (ii)
2
t From (i) & (ii) u = – 6 m/s and a = 4 m/s ,
[Putting value of s1 and s2 and v = ]
  so, distance travelled in 5 sec,
2
Hence correct answer is (c). From "s = ut + 1/2 at " ,
2
Example-25 s = – 6 × 5 + (1/2) × 4 × 5 = 20 m
A passenger is standing ’d’ m away from a bus. The bus Similarly distance travelled in 8 sec
2
begins to move with constant acceleration a. To catch = –6 × 8 + (1/2) 4 × 8 = 80 m
the bus, the passenger runs at a constant speed v So distance travelled in next 3 sec
towards the bus. The minimum speed of the passenger = 80 – 20 = 60 m
so that he may catch the bus will be- Hence correct answer is (c)
(a) 2ad (b) ad (c) 2 ad (d) ad

82
PHYSICS-X I-IIT-JEE MOTION I N ONE DIMEN SIO N

Example-27 1. Body Falling Freely Under Gravity:


Taking initial position as origin and direction of
A particle starts with an initial velocity 2.5 m/s along the motion (i.e. downward direction) as positive, here
positive x-direction and it accelerates uniformly at the rate we have u = 0 (as body starts from rest)
0.50 m/s2. The distance covered in reaching the velocity a=+g
7.5 m/s will be- (as acceleration is in the direction of motion)
So, if the body acquires velocity v after falling a
(a) 25 m (b) 50 m (c) 75 m (d) 100 m
distance h in time t, equations of motion viz
Solution. v = u + at
2 2
We have, v = u + 2a x 1 2
2 2 s = ut +   at
or (7.5) = (2.5) + 2(0.50) x  x = 50 m 2
2 2
Hence correct answer is (b) and v = u + 2as reduces to
v = gt ....(1)
Example-28 1 2
h =   gt ....(2)
A particle starts moving from position of rest under a 2
constant acceleration. If it travels a distance x in t sec. 2
v = 2 gh ....(3)
The distance it will travel in next t sec will be- These equations can be used to solve most of the
(a) 2x (b) 3x (c) 4x (d) 5x problems of freely falling as
Solution. t is given h is given v is given
The velocity of particle after time t will be From eq. (1) & From eq. (2) From eq. (3) &
(2) &(3) (1)
v = u + at = 0 + at = at
v = gt 2h v
Now for next t sec, it will be the initial velocity, t= t=
2 g g
From " s = u't + (1/2) at ", we have 
1 v= v2
 x' = (at) t + (1/2) at
2
[Here u' = at] and h = gt2 2gh
h=
2
2 2g
x' = 3/2 at …(1)
s v a
This is the distance travelled in next t sec
Also given that particle travels x distance in tan = g g
2 
t sec. so again using "s = ut + (1/2) at "
t (B) t
1 2 (A) (C) t
We have, x = at …(2)
2
From (1) & (2) , we have, x' = 3x Note :
Hence correct answer is (b) (a) If the body is dropped from a height H, as in time t,
it has fallen a distance h from its initial position, the
10. MOTION UNDER GRAVITY height of the body from the ground will be h' = H –
2
Ideal Motion: h, with h = 1/2 gt .
2 2
The most important example of motion in a straight (b) As h = (1/2) gt i.e. h  t , distance fallen in time t,
line with constant acceleration is motion under 2t, 3t etc. will be in the ratio of
2 2 2
gravity. In case of motion under gravity unless 1 : 2 : 3 : ––––––– i.e. square of integers.
th
stated it is taken for granted that. (c) The distance fallen in n sec.,
2 2
(i) The acceleration is constant, i.e. hn – hn –1 = (1/2) g(n) – (1/2) g(n –1)
  = (1/2) g(2n –1)
a = g = 9.8 m/s2 So distance fallen in Ist, IInd, IIIrd sec will be in the
and directed vertically downwards. ratio 1 : 3 : 5 i.e. odd integers only.
(ii) The motion is in vacuum i.e. viscous force or thrust
of the medium has no effect on the motion. 2. Body is projected vertically up:
Taking initial position as origin and direction of
Now in the light of above assumptions, there are
motion (i.e. vertically up) as positive, here we have
two possibilities. v = 0 [as at the highest point, velocity = 0], a = – g

83
MOTION I N ONE DIMEN SIO N PHYSICS-X I-IIT-JEE

[as acceleration is downwards while motion (c) As from case (b) time taken to reach a height h,
upwards].
tU = 2h / g
So, if the body is projected with velocity u and
reaches the highest point at a distance h above the And from case (a) time taken to fall down through a
ground in time t, the equations of motion viz distance h,
v = u + at
1 2 tD = 2h / g
s = ut +   at
2 so tU = tD = 2 h / g
2 2
and v = u + 2as reduces to
So in case of motion under gravity time taken to go
0 = u – gt
h = ut – 1/2 gt
2 up is equal to the time taken to fall down through
2 the same distance.
and 0 = u – 2gh
or u = gt ....(1) (d) If a body projected vertically up reaches a height h
2
h = 1/2 gt ....(2) then from case (b), u = 2gh and if a body falls
2
( u = gt), u = 2 gh ....(3)
freely through a height h from case (a), v =
These equations can be used to solve most of the
problems of bodies projected vertically up as, if 2gh
So in case of motion under gravity the speed with
If is given If h is given is u given
which a body is projected up is equal to the speed
From eq. (1) & (2) From eq. (2) & From eq. (3) &
(3) (1) with which it comes back to the point of projection
u = gt t = 2h / g t = u/g
1 2 u= 2hg h = u2/2g Example-29
h= gt
2 From the top of a building a ball is dropped, while
u a
s u2/2g +
another is thrown horizontally at the same time. Which
+
O f ball will strike the ground first ?
g
u/g 2u/g (a) The ball projected horizontally
O –
u/g – (b) The ball projected vertically
(c)
(a) (b) (c) Both at the same time
(d) It depends upon mass of the balls
 Discussion:
From cases (1) and (2) it is clear that: Solution.
(a) In case of motion under gravity for a given body, Both the balls will reach the ground simultaneously as
mass, acceleration and mechanical energy remains horizontal velocity does not effect the vertical motion, t1
constant while speed, velocity, momentum, kinetic 2
energy and potential energy changes. = t2 = ( 2 h / g ) [from "h = 1/2 gt ", as u = 0]
(b) The motion is independent to the mass of the body Note:
as in any equation of motion mass is not involved. However for the ball dropped vertically,
This is why a heavy and light body when released v1 = (2gh ) , while for the ball projected horizontally :
from same height reaches the ground
Horizontal velocity (vH)2 = u and
simultaneously and with same velocity.
Vertical velocity (vv)2 = (2gh ) ,
i.e. t= 2h / g
So that v2 = (u 2  2gh)
and v = 2gh
i.e. on hitting the ground speed of horizontally projected
However, momentum, kinetic energy or potential
ball will be more than the ball dropped vertically
energy depends on the mass of the body
Hence correct answer is (c)
(all  mass)

84
PHYSICS-X I-IIT-JEE MOTION I N ONE DIMEN SIO N

Example-30 Example-32
A body is released from a height and falls freely Statement given below is true or false ? Give reason in
towards the earth. Exactly 1 sec later another body is brief. "Two balls of different masses are thrown
released. What is the distance between the two bodies 2 sec vertically upwards with the same speed. They reach
2
after the release of the second body? If g = 9.8 m/s .
through the point of projection in their downward
(a) 2.45m (b) 24.5m
(c) 4.9m (d) 9.8m motion with the same speed".
Solution: (a) True
According to given problem 2nd body falls for 2 s so that (b) False
2
h2 = (1/2) g (2) … (1) (c) Depend upon conditions
While Ist has fallen for 2 + 1 = 3 s, (d) None of these
2
so h1 = (1/2) (3) g … (2) Solution.
 Separation between two bodies 2 s after the release
The statement is true as motion under gravity is
of IInd body
2 2
d = h1 – h2 = (1/2) g(3 –2 ) = 4.9 × 5 = 24.5 m Hence independent of mass of the body and as body comes
correct answer is (b) back to the point of projection with same speed, so
v1 = u1 and v2 = u2, Here u1 = u2 = u (given)
Example-31 so, v1 = v2 = u
If a body travels half its total path in the last second of Hence correct answer is (a)
its fall from rest. The time and height of its fall, will
2
respectively be- (g = 9.8 m/s ) 11. MOTION WITH VARIABLE ACCELERATION
(a) 0.59 s, 57 m (b) 3.41 s, 57 m
(c) 5.9 s, 5.7 m (d) 5.9 s, 34.1 m There are only two equations in this type of motion.
dx
(a) v =
Solution. dt
If the body falls a height h in time t, from 2nd equation dv d 2 x
of motion we have (b) a = = 2
2 dt dt
h = 1/2 gt ….(1)
[u = 0 as body starts from rest]
Now the distance fallen in (t – 1) s will be Example-33
2
h = 1/2 g(t – 1) …(2) The displacement of particle is zero at t = 0 and at t = t
So from eq. (1) & (2) distance fallen in the last second it is x. It starts moving in the x direction with velocity,
2 2
h – h' = (1/2) gt – (1/2) g (t – 1) ,
which varies as v = k x , where k is constant. The
h – h' = (1/2) g (2t – 1)
But according to given problem as velocity-
(h – h') = h/2 (a) Varies with time
i.e. (1/2) h = (1/2)g (2t –1) (b) Independent to time
2
or (1/2) gt = g(2t –1) (c) Inversely proportional to time
2
[as from eq. (1) h = (1/2) gt ] (d) Nothing can be said
2
or t – 4t + 2 = 0 Solution.
or t = [4 ± (42  4  2) ] /2 dx
v=k x  =k x 
or t = 2 ± 2 or t = 0.59 or 3.41s dt
0.59 s is physically unacceptable as it gives the total time t dx x 1/ 2
taken by the body to reach ground is lesser than one sec   =  kdt  = kt + c
x 1/ 2
while according to the given problem time of motion must
be greater than 1 s. Given that, at t = 0, x = 0 c=0
So t = 3.41 s & 1/2
Now, 2x = kt  x = (1/2) kt,
2
h = (1/2) × (9.8) × (3.41) = 57 m Now, v = k (1/2 kt) = k t/2
2
Hence correct answer is (b)
Thus velocity varies with time. Hence correct answer is (a)

85
MOTION I N ONE DIMEN SIO N PHYSICS-X I-IIT-JEE

Example-35 3t 2
2 Hence, v = ,
The acceleration of a particle is given as a = 3x . At 2
t = 0, v = 0, x = 0. The velocity at t = 2 sec will be- 3  22
(a) 0.05 m/s (b) 0.5 m/s Velocity at t = 2 sec is = 6 m/s
(c) 5 m/s (d) 50 m/s 2
dx 3t 2 3 2
Also,  
Solution. dt 3  dx  2  t dt 
2 dv 2
a = 3x  v = 3x 3 t3
dx x= + c'
 vdv = 3x dx 
2 2 3

v2 x2 t3
 =3 +c at t = 0, x = 0  c' = 0,  x = ,
2 3 2
At t = 0, v = 0, x = 0 23
Now displacement at t = 2 sec is =4m
v2 3 2
c = 0 Now, =x
2 Hence correct answer is (a)
2 3 3/2
v = 2x  v = 2 x … (1)
dx 3/2 Note:
 = 2x
dt Prohibit the use of definite integral to avoid
Remember, when a is function of x. blunders as constant may change from the given
vdv initial conditions.
Use a =
dx 12. VARIOUS GRAPHS RELATED TO MOTION
dv 3/2
When a is function of t, a = dx = 2 x dt A. Displacement-Time Graph :
dt
(a) For a stationary body
dx
 3 / 2 = 2 t + c', at t = 0, x = 0, v = 0  y
x
 c' = 0 Displacement
2 2
Now = 2 t  4 = 2xt
x
2
x= 2 …(2)
t x
3/ 2 0 Time
 2
From (1) and (2) v = 2 2  , (b) For a body moving with constant velocity
t  y
at t = 2 sec v = 1/2 m/sec
Hence correct answer is (b)
Displacement

Example-36
The acceleration of a particle is given by a = 3t and at
t = 0, v = 0, x = 0. The velocity and displacement at x
t = 2 sec will be- 0 Time
(a) 6 m/s, 4 m (b) 4 m/s, 6 m
(c) 3 m/s, 2 m (d) 2 m/s, 3 m (c) For a body moving with non-uniform velocity
Solution. y
dv
a = 3t  = 3t   dv =  3tdt 
Displacement

dt
3t 2
v= +c
2
Substituting the initial conditions,
At t = 0, v = 0 and x = 0 x
0 Time
c=0

86
PHYSICS-X I-IIT-JEE MOTION I N ONE DIMEN SIO N

(d) For a body with accelerated motion (a) For the body having constant velocity or zero
y acceleration

Displacement
y

Velocity
x
0 Time
x
0 Time
(e) For a body with decelerated motion
(b) When the body is moving with constant
y
retardation and its initial velocity is not zero
y
Displacement

Velocity
x
0 Time
x
(f) For a body which returns towards the point of 0 Time
reference (c) When body moves with non-uniform
y acceleration and its initial velocity is zero.
y
Displacement

Velocity
 > 90º
0 x

Time
x
0 Time
(g) For a body whose velocity constantly changes
y (d) When the body is accelerated and its initial
velocity is zero
Displacement

y
Velocity

x
0
Time
(h) For a body whose velocity changes after certain x
0 Time
interval of time
(e) When the body is decelerated.
y
y
Displacement

Q
P R
Velocity

x
0 S
Time x
0
B. Velocity-Time Graph: Time

87
MOTION I N ONE DIMEN SIO N PHYSICS-X I-IIT-JEE

C. Acceleration-Time Graph : (c) When acceleration is decreasing and is


(a) When acceleration is constant negative
y y

Acceleration

Acceleration
x x
0 0 Time
Time
(d) When initial acceleration is zero and rate of
(b) When acceleration is increasing and is positive change of acceleration is non-uniform
y y
Acceleration

Acceleration
x x
0 Time 0 Time

88
PHYSICS-X I-IIT-JEE MOTION I N ONE DIMEN SIO N

EXERCISE # 1
Based On Distance and Displacement 7. A person walks along an east west street, and a
graph of his displacement from home is shown in
1. A Body moves 6 m north. 8 m east and 10m figure. His average velocity for the whole time
vertically upwards, what is its resultant interval is –
displacement from initial position? x(m)
B
10
(A) 10 2m (B) 10 m (C) m (D) 10  2m
2 A

2. Mark the wrong statement - C


15 20
(A) Nothing is in the state of absolute rest or
t(sec)
state of absolute motion
(B) Magnitude of displacement is always equal to
the distance travelled (A) 0 (B) 23 m/s
(C) Magnitude of displacement can never be (C) 8.4 m/s (D) None of the above
greater than the distance travelled
8. A particle is confined to move along the
(D) Magnitude of displacement may be equal to x-axis between reflecting walls at x = 0 and
the distance travelled x = a. Between these two limits it moves freely at
constant velocity v. If the walls are perfectly
3. A car moving at a speed v is stopped in a certain reflecting, then its displacement time graph is -
distance when the brakes produce a deceleration
a. If the speed of the car was no, what must be the displacement
deceleration of the car to stop it in the same a
distance and in the same time? (A)

(A) n a (B) na (C) n2a (D) n3a


0 a/v 2a/v 3a/v
4. A car is moving at a certain speed. The minimum time
distance over which it can be stopped is x. If the
displacement

speed of the car is doubled, what will be the mini-


a
mum distance over which the car can be stopped (B)
during the same time?
(A) 4x (B) 2x (C) x/2 (D) x/4
0 a/v 2a/v 3a/v
5. A body covered a distance of L m along a curved time
path of a quarter circle. The ratio of distance to
displacement

displacement is-
(A) /2 2 (B) 2 2 / (C)
 (C) / 2 (D) 2 / time

6. An old man goes for morning walk on a


semicircular track of radius 40 m ; if he starts
displacement

from one end of the track and reaches to other


end, the distance covered by the man and his a
displacement will respectively be- (D)
(A) 126 m, 80 m (B) 80 m, 126 m
0 a/v 2a/v 3a/v
(C) 80 m, 252 m (D) 252 m, 80 m time

89
MOTION I N ONE DIMEN SIO N PHYSICS-X I-IIT-JEE
9. Figure shows the displacement time graph of a 12. Which of the following options is correct for the
particle moving on the x-axis - object having a straight line motion represented
by the following graph
x D
C

t B
t0 t
A
(A) The particle is continuously going in positive O
s
x direction (A) The object moves with constantly increasing
(B) The particle is at rest velocity from O to A and then it moves with
(C) The velocity increases up to a time t0,and constant velocity.
(B) Velocity of the object increases uniformly
then becomes constant (C) Average velocity is zero
(D) The particle moves at a constant velocity up (D) The graph shown is impossible
to a time t0, and then stops 13. Two boys are standing at the ends A and B of a
ground where AB  a . The boy at B starts
10. The displacement time graph for a one running in a direction perpendicular to AB with
velocity v1 . The boy at A starts running
dimensional motion of a particle is shown in
simultaneously with velocity v and catches the
figure. Then the instantaneous velocity at other boy in a time t, where t is
t = 20 sec is – 2 2 2 2 2
(A) a / v  v1 (B) a /(v  v1 )
(C) a /(v  v1) (D) a /(v  v1)
Displacement, (m) 

2
14. The velocity acquired by a body moving with
uniform acceleration is 20 meter/second in first 2
1 seconds and 40 m/sec in first 4 sec. The initial
velocity will be -
(A) 0 m/sec (B) 40 m/sec
0 (C) 20 m/sec (D) None
0 10 20 30 40 50 60
15. A body moves along the sides AB, BC and CD of
Time, sec 
a square of side 10 meter with velocity of
(A) 0.1 m/s (B) – 0.1 m/s constant magnitude 3 meter/sec. Its average
(C) – 0.05 m/s (D) 1.0 m/s velocity will be-
(A) 3 m/sec (B) 0.87 m/sec
(C) 1.33 m/sec (D) None
Based On Uniform Motion
16. A body covers half the distance with a velocity
11. A man walks on a straight road from his home to 10 m/s and remaining half with a velocity
a market 2.5 km away with a speed of 5 km/h. 15 m/s along a straight line. The average velocity
will be-
Finding the market closed, he instantly turns and
(A) 12 m/s (B) 10 m/s
walks back home with a speed of 7.5 km/h. The (C) 5 m/s (D) 12.5 m/s
average speed of the man over the interval of 17. A point travelling along a straight line traverse
one third the distance with a velocity v0. The
time 0 to 40 min. is equal to
remaining part of the distance was covered with
25 velocity v1 for half the time and with velocity v2
(A) 5 km/h (B) km/h
4 for the other half of the time. The mean velocity
(C) 30
km/h (D) 45
km/h of the point averaged over the whole time of
4 8 motion will be-

90
PHYSICS-X I-IIT-JEE MOTION I N ONE DIMEN SIO N

v 0 ( v1  v 2 ) 3v 0 ( v1  v 2 ) 23. A body is moving from rest under constant


(A) (B)
3(v1  v 2  v3 ) v1  v 2  v 3 acceleration and let S 1 be the displacement in
v0 (v1  v 2 ) 3v0 (v1  v2 ) the first (p  1) sec and S 2 be the displacement in
(C) (D)
v1  v 2  4v 3 v1  v2  4v0 the first p sec. The displacement in ( p 2  p  1) th
sec. will be
18. A particle moves along the x-axis in such a way (A) S1  S2 (B) S1S2
that its x-coordinate varies with time (C) S1  S2 (D) S1 / S2
2
according to the equation x = 2 – 5t + 6t . The
24. A body starts from the origin and moves along the
initial velocity and acceleration will respectively
X-axis such that the velocity at any instant is given
be- by (4 t 3  2t) , where t is in sec and velocity in m/s.
2 2
(A) 5 m/s, 12 m/s (B) –12 m/s, –5 m/s What is the acceleration of the particle, when it is 2
2 2
(C) 12, – 5 m/s (D) –5 m/s, 12 m/s m from the origin
(A) 28 m / s 2 (B) 22 m / s 2
19. The position of a body with respect to time is (C) 12 m / s 2 (D) 10 m / s 2
3 2
given by x = 4t – 6t + 20 t + 12. Acceleration at
25. The relation between time and distance is
t = 0 will be-
t  x 2  x , where  and  are constants. The
(A) –12 units (B) 12 units
retardation is
(C) 24 units (D) –24 units
(A) 2v 3 (B) 2v 3
(C) 2v 3 (D) 2 2 v 3
20. A body travels 200 cm in the first two seconds
and 220 cm in the next four second. The velocity 26. A point moves with uniform acceleration and
at the end of the seventh second from the start v1,v2 and v 3 denote the average velocities in the
will be- three successive intervals of time t 1 , t 2 and t3 .
(A) 10 cm/s (B) 5 cm/s Which of the following relations is correct
(C) 15 cm/s (D) 20 cm/s (A) (v1  v 2 ) : (v2  v3 )  (t1  t 2 ) : (t 2  t 3 )
(B) (v1  v2 ) : (v2  v3 )  (t1  t 2 ) : (t 2  t 3 )
21. An  particle travels along the inside of straight
(C) (v1  v2 ) : (v2  v3 )  (t1  t 2 ) : (t1  t 3 )
hollow tube, 2.0 meter long, of a particle
(D) (v1  v2 ) : (v2  v3 )  (t1  t 2 ) : (t 2  t 3 )
accelerator. Under uniform acceleration, how
long is the particle in the tube if it enters at a
27. The initial velocity of a particle is u (at t  0 )
speed of 1000 m/s and leaves at 9000 m/s -
–4 –7
and the acceleration f is given by at . Which of
(A) 4 × 10 sec (B) 2 × 10 sec the following relation is valid
–4 –7
(C) 40 × 10 sec (D) 20 × 10 sec t2
(A) v  u  at2 (B) v  u  a
2
Based On Non-uniform Motion (C) v  u  at (D) v u

22. A particle experiences a constant acceleration for


28. The velocity of a body depends on time
20 sec after starting from rest. If it travels a
according to the equation v  20  0.1t 2 . The
distance S1 in the first 10 sec and a distance S 2 body is undergoing
in the next 10 sec, then (A) Uniform acceleration
(A) S1  S2 (B) S1  S2 / 3 (B) Uniform retardation
(C) Non-uniform acceleration
(C) S1  S2 / 2 (D) S1  S2 / 4
(D) Zero acceleration

91
MOTION I N ONE DIMEN SIO N PHYSICS-X I-IIT-JEE

29. The position of a particle moving in the xy-plane 34. A truck starts from rest with an acceleration of
at any time t is given by x  (3t 2  6t) metres, 1.5 m/s2 while a car 150 m behind starts from rest
with an acceleration of 2 m/s2. How long will it take
y  (t 2  2t) metres. Select the correct statement
before both the truck and car side by side, and how
about the moving particle from the following much distance is travelled by each ?
(A) The acceleration of the particle is zero at t  0 (A) 2.45 s, 500 m (truck), 650 m (car)
second (B) 5 s, 450 m (truck), 600 m (car)
(B) The velocity of the particle is zero at t  0 (C) 24.5 s, 450 m (truck), 600 m (car)
second (D) 5.3 s, 500 m (truck), 650 m (car)
(C) The velocity of the particle is zero at t  1 35. Two car travelling towards each other on a straight
second road at velocity 10 m/sec and 12 m/sec respectively.
(D) The velocity and acceleration of the particle are When they are 150 m apart, both drivers apply their
never zero 2
brakes and each car decelerates at 2 m/sec until it
stops. How far apart will they be when they have
30. Two trains travelling on the same track are both come to a stop?
approaching each other with equal speeds of 40 (A) 8.9 m (B) 89 m
m/s. The drivers of the trains begin to decelerate (C) 809 m (D) 890 m
simultaneously when they are just 2.0 km apart.
36. The driver of a train travelling at 115 km/hour
Assuming the decelerations to be uniform and sees on the same track 100 m in front of him a
equal, the value of the deceleration to barely slow train travelling in same direction at
avoid collision should be 25 km/hr. The least retardation that must be
(A) 11.8 m/ s 2 (B) 11.0 m/ s 2 applied to the faster train to avoid a collision will
(C) 2.1 m/ s 2 (D) 0.8 m/ s 2 be-
2 2
(A) 3.125 m/s (B) 31.25 m/s
2 2
31. What is the relation between displacement, time (C) 312.5 m/s (D) 0.3125 m/s
and acceleration in case of a body having uniform
37. A car is moving with a velocity of 20 m/sec. The
acceleration?
driver sees a stationary truck at a distance of 100
1 2
(A) S  ut  ft (B) S  (u  f ) t m ahead. After some reaction time t he applies
2
the brakes, produces a retardation of
(C) S  v 2  2 fs (D) None of these 2
4 m/s . The maximum reaction time to avoid
collision will be –
32. The position of a particle moving along the
(A) 5 sec (B) 2.5 sec
x-axis at certain times is given below: (C) 4 sec (D) 10 sec
t (s) 0 1 2 3 38. An engine driver of a passenger train travelling at
x (m) –2 0 6 16 40 m/s sees a goods train, whose last
Which of the following describes the motion compartment is 250 m ahead on the same track.
correctly The goods train is travelling in the same direction
(A) Uniform, accelerated as the passenger train, with a constant speed of 20
(B) Uniform, decelerated m/s. The passenger train driver has reaction time
(C) Non-uniform, accelerated of 0.5 sec. He applies the brakes which causes the
2
(D) There is not enough data for generalization train to decelerate at the rate of 1 m/sec , while
33. A car, starting from rest, accelerates at the rate f the goods train continues with its constant speed.
Can the driver save a crash?
through a distance S, then continues at constant
(A) Yes, if the distance between the trains before
speed for time t and then decelerates at the rate f application of brakes is more than 200 m
2
(B) Yes, if the distance between the trains before
to come to rest. If the total distance traversed is application of brakes is more than 250 m
15 S, then (C) No, if the distance between the trains before
1 2 1 2 application of brakes is more than 200 m
(A) S ft (B) S ft
2 4
(D) No, if the distance between the trains before
1 2 1 2 application of brakes is more than 250 m
(C) S ft (D) S ft
72 6

92
PHYSICS-X I-IIT-JEE MOTION I N ONE DIMEN SIO N

Based On Motion Under Gravity 43. An aeroplane is moving with horizontal velocity
u at height h . The velocity of a packet dropped
39. A stone is dropped into water from a bridge from it on the earth's surface will be ( g is
44.1m above the water. Another stone is thrown
acceleration due to gravity)
vertically downward 1 sec later. Both strike the
water simultaneously. What was the initial speed (A) u 2  2 gh (B) 2 gh
of the second stone? (C) 2gh (D) u 2  2gh
(A) 12.25 m / s (B) 14.75 m / s
(C) 16.23 m / s (D) 17.15 m / s 44. A body dropped from a height h with an initial
speed zero, strikes the ground with a velocity
40. An iron ball and a wooden ball of the same radius 3 km/ h . Another body of same mass is dropped
are released from the same height in vacuum. from the same height h with an initial speed
They take the same time to reach the ground. The u  4km / h . Find the final velocity of second
reason for this is body with which it strikes the ground
(A) Acceleration due to gravity in vacuum is
(A) 3 km/h (B) 4 km/h
same irrespective of the size and mass of the
(C) 5 km/h (D) 12 km/h
body
(B) Acceleration due to gravity in vacuum 45. The time taken by a block of wood (initially at
depends upon the mass of the body rest) to slide down a smooth inclined plane 9.8 m
(C) There is no acceleration due to gravity in o
vacuum long (angle of inclination is 30 ) is
(D) In vacuum there is a resistance offered to the
motion of the body and this resistance
depends upon the mass of the body

41. A frictionless wire AB is fixed on a sphere of 30°


radius R. A very small spherical ball slips on this
1
wire. The time taken by this ball to slip from A to sec
(A) 2 (B) 2 sec
B is
A (C) 4 sec (D) 1 sec

 46. A ball is released from the top of a tower of


O height h meters. It takes T seconds to reach the
B R ground. What is the position of the ball in T/3
seconds
(A) h/9 meters from the ground
C (B) 7h/9 meters from the ground
2 gR (C) 8h/9 meters from the ground
(A)
g cos  (D) 17h/18 meters from the ground
cos 
(B) 2 gR .
g 47. When a ball is thrown up vertically with velocity
R Vo
(C) 2 , it reaches a maximum height of 'h'. If one
g wishes to triple the maximum height then the ball
gR should be thrown with velocity
(D)
g cos 3 Vo 3Vo
(A) (B)
(C) 9Vo (D) 3 / 2Vo
42. A stone is dropped from a certain height which
can reach the ground in 5 second. If the stone is 48. A ball is thrown from the ground with a velocity
stopped after 3 second of its fall and then allowed of 80 ft/sec. Then the ball will be at a height of
to fall again, then the time taken by the stone to 96 feet above the ground after time -
reach the ground for the remaining distance is (A) 2 and 3sec (B) only 3 sec
(A) 2 sec (B) 3 sec (C) only 2sec (D) 1 and 2 sec
(C) 4 sec (D) None of these

93
MOTION I N ONE DIMEN SIO N PHYSICS-X I-IIT-JEE

49. A pebble is thrown vertically upwards from 54. A ball is thrown from ground vertically upward,
bridge with an initial velocity of 4.9 m/s. It reaches the roof of a house 100 meters high. At
strikes the water after 2s. If acceleration due to the moment this ball was thrown vertically
2
gravity is 9.8 m/s . The height of the bridge & upward, another ball is dropped from rest
velocity with which the pebble strike the water vertically downward from the roof of the house.
will respectively be - At which height from the ground do the balls
(A) 4.9 m, 1.47 m/s (B) 9.8 m, 14.7 m/s pass each other and after what time?
(C) 49 m, 1.47 m/s (D) 1.47 m, 4.9m/s 100
50. A balloon going upward with a velocity of (A) t = sec, h = 25 m
1960
12 m/sec is at a height of 65 m from the earth at 100
any instant. Exactly at this instant a packet drops (B) t = sec; h = 25 m
from it. How much time will the packet take in 1690
2 200
reaching the earth? (g = 10 m/sec )
(A) 7.5 sec (B) 10 sec (C) t = , h = 75 m
1690
(C) 5 sec (D) None 100
(D) t = sec; h = 75 m
1960
51. A body is falling from a height 'h'. It takes t1 sec
to reach the ground, the time taken to reach the 55. From the foot of a tower 90 m high a stone is
half of the height will be- thrown up so as to reach the top of the tower.
t1 Two second later another stone is dropped from
2 t1
the top of the tower. When and where two stones
(A) (B) 2 meet?
t1
22
(C) 2 (D) 2t1 (A) sec later, at 83.6 m
7
22
52. A body thrown up with a velocity reaches a (B) sec later, at 86.6 m
7
maximum height of 100 m. Another body with 2. 2
double the mass thrown up with double the initial (C) sec later, at 86.6 m
7
velocity will reach a maximum height of- 2. 2
(A) 400 m (B) 200 m (D) sec later, at 83.6 m
7
(C) 100 m (D) 250 m

53. A ball dropped from the top of a building takes 56. A motor boat covers the distance between two
0.5 sec to clear the window of 4.9 m height. What spots on the river in t1 = 8 hr and t2 = 12 hr
is the height of building above the window? downstream and upstream respectively. The time
(A) 2.75 m (B) 5.0 m required for the boat to cover this distance in still
(C) 5.5 m (D) 4.9 m water will be-
(A) 6.9 hr (B) 9.6 hr
(C) 69 sec (D) 96 sec

94
PHYSICS-X I-IIT-JEE MOTION I N ONE DIMEN SIO N

EXERCISE # 2
Question Multiple Correct Answer Type Questions
4. A train accelerates from rest for time t1, at a
1. A particle moves in a straight line with an a–t
constant acceleration  for distance x. Then it
curve shown in figure. The initial displacement
decelerates to rest at constant retardation  in
and velocity are zero.
time t2 for distance y. Then -
a(m/s2)
x   t x t
4 (A) = (B) = 1 (C) = 1 (D) x = y
y   t2 y t2
2 10
0 t(s) 5. In the figure is shown the position of a particle
–2 moving on the x-axis as a function of time. Then-

(A) The time at which the particle comes to rest x


again is 10 sec 20
(B) Velocity of particle at t = 2 sec is 8 m/s
(C) Distance travelled by particle before coming 10
to rest is 29.36 m
(D) Displacement of the particle before coming to 2 4 6 8
t(s)
rest is 29.36 m
(A) the particle has come to rest for 6 times
2. A body starting from rest is moved along straight
(B) the maximum speed is at t = 6 s
line by a machine delivering constant power.
(C) the velocity remain positive for t = 0 to
Choose the correct graph -
t = 6s
(D) the average velocity for the total period
(A) K (B) K shown is negative
S t
6. Velocity of a particle moving in a curvillinear

path varies with time as v  (2t î  t 2 ˆj) m/s.
(C) v (D) S Here, t is in second. At t = 1 sec.
t t (A) acceleration of particle is 8 m/s2
(B) tangential acceleration of particle is
3. A train travels between two of its station stops 6
with the acceleration schedule shown. t is the m/s2
5
time interval during which train breaks to stop.
2
a(m/s2)
(C) radial acceleration of particle is m/s2
5
(D) None of these
2
1 7. Two particles A and B start simultaneously from
0 the same point and move in a horizontal plane. A
8 10 t(s) has an initial velocity u1 due east and acceleration
6
t
–2 a1 due north. B has an initial velocity u2 due north
and acceleration a2 due east.
(A) t = 10 sec (A) Their paths must intersect at some point
(B) Distance between two stations is 416 m (B) They must collide at some point
(C) t = 15 sec (C) They will collide only if a1u1 = a2u2
(D) Distance between two station is 510 m (D) If u1 > u2 and a1 < a2, the particles will have
the same speed at some point of time.

95
MOTION I N ONE DIMEN SIO N PHYSICS-X I-IIT-JEE

8. At the moment t = 0 a particle leaves the origin 12. Equation of a particle moving along the x axis
and moves in the positive direction of the is : x = u (t – 2) + a (t – 2)
2
x-axis. Its velocity varies with time as v = 5 m/s
(1 – t/2). (A) the initial velocity of the particle is u
Select the correct statements - (B) the acceleration of the particle is a
(A) At t = 8.8 s the particle is at the distance 10m (C) the acceleration of the particle is 2a
from the origin (D) at t = 2 particle is at origin
(B) At t = 2 s the particle is at the distance 10m
from the origin 13. The velocity-time plot for a particle moving on a
(C) At t = 4 s the velocity of the particle is straight line is shown in figure.
– 5m/s
(D) At t = 2 s, the particle is at maximum position 10
in positive direction of x-axis

9. A car accelerates from rest at a constant rate of v


10 20
2 ms–2 for some time. Then it retards (speed
0
decrease) at a constant rate of 4 ms–2 and comes
t
to rest. It remains in motion of 6 s.
(A) Its maximum speed is 8 ms–1 –10
(B) Its maximum speed is 6 ms–1 (A) the particle has constant acceleration
(C) It travelled a total distance of 24 m (B) the particle has never turned around
(D) It travelled a total distance of 18 m
(C) the particle has zero displacement
10. Which of the following graph(s) is / are not (D) the average speed in the interval 0 to 10 sec is
possible ? same as the average speed in the interval 10
sec to 20 sec
Distance

A particle having a velocity v = v0k at t = 0 is


Distance

14.
(A) (B) decelerated at the rate || =  v , where  is a
positive constant.
O
O
Time Time 2 v0
(A) The particle comes to rest at t =

(B) The particle will come to rest at infinity
Distance
Distance

(C) The distance travelled by the particle before


(C) (D)
2 v 30 / 2
coming to rest is

O t1 Time O Time
(D) The distance travelled by the particle before
11. The position of particle travelling along x-axis is 2 v 30 / 2
3 2 coming to rest is
given by xt = t – 9t + 6t where xt is in cm and t 3
is in second. Then–
(A) the body comes to rest firstly at (3 – 7)s and 15. At time t = 0, a car moving along a straight line
then at (3 + 7) s has a velocity of 16 m/s. If slows down with an
(B) the total displacement of the particle in acceleration of –0.5 m/s2, where t is in second.
travelling from the first zero of velocity to the Mark the correct statement (s).
second zero of velocity is zero (A) The direction of velocity changes at t = 8 s
(C) the total displacement of the particle in
(B) The distance travelled in 4 s is approximately
travelling from the first zero of the velocity to
the second zero of velocity is –74 cm 58.67 m
(D) the particle reverses its velocity at (C) The distance travelled by the particle in 10 s
(3 – 7) s and then at (3 + 7)s and has a is 94 m
negative velocity for (3 – 7)s < t < (3 + 7)s (D) The speed of particle at t = 10 s is 9 m/s

96
PHYSICS-X I-IIT-JEE MOTION I N ONE DIMEN SIO N

16. An object moves with constant acceleration a. 20. A particle is resting over a smooth horizontal
Which of the following expressions are also floor. At t = 0, a horizontal force starts acting on
constant? it. Magnitude of the force increases with time
d|v| dv according to law F = t, where  is a positive
(A) (B)
dt dt constant. From figure, which of the following
statements are correct ?
 v 
d  Y
2
d (v ) | v |  2
(C) (D)  1
dt dt

17. Ship A is located 4 km north and 3 km east of


ship B. Ship A has a velocity of 20 km/h towards
the south and ship B is moving at 40 km/h in a
direction 37º north of east. X and Y-axes are O X
along east and north directions, respectively-
(A) Velocity of A relative to B is (A) Curve 1 can be the plot of acceleration
against time
(–32 î – 44 ĵ ) km/h
(B) Curve 2 can be the plot of velocity against
(B) Position of A relative to B as a function of time
time is given by (C) Curve 2 can be the plot of velocity against
rAB = [(3 – 32t) î + (4 – 44t) ĵ ) km acceleration
(C) Position of A relative to B is (D) Curve 1 can be the plot of displacement
(32 î – 44 ĵ ) km/h against time
(D) Position of A relative to B as a function of 21. A train starts from rest at S = 0 and is subjected
time is given by (32t î – 44t ĵ ) km to an acceleration as shown in figure. Then,
a(m/s2)
18. Starting from rest a particle is first accelerated for
time t1 with constant acceleration a1 and then 6
stops in time t2 with constant retardation a a2. Let
v1 be the average velocity in this case and s1 the
total displacement. In the second case it is
accelerating for the same time t1 with constant
acceleration 2a1 and come to rest with constant
S(m)
retardation a2 in time t2. If v2 is the average 30
velocity in this case and s2 the total displacement,
(A) velocity at end of 10 m displacement is
then
20 m/s
(A) v2 = 2v1 (B) 2v1 < v2 < 4v1
(B) velocity of the train at S = 10 m is 10 m/s
(C) s2 = 2s1 (D) 2s1 < s2 < 4s1
(C) the maximum velocity attained by train is
19. A particle is moving along a straight line. The 180 m/s
displacement of the particle becomes zero in a (D) the maximum velocity attained by the train is
certain time (t > 0). The particle does not 15 m/s
undergo any collision. 22. For a moving particle, which of the following
(A) The acceleration of the particle may be zero options may be correct?
always Here, Vav is average velocity and vav the average
(B) The acceleration of the particle may be speed.
uniform (A) |Vav | < vav
(C) The velocity of the particle must be zero at (B) |Vav | > vav
some instant
(C) Vav = 0 but vav  0
(D) The acceleration of the particle must change
its direction (D) Vav  0 but vav = 0

97
MOTION I N ONE DIMEN SIO N PHYSICS-X I-IIT-JEE

23. Identify the correct graph representing the motion 26. The speed of a train increases at a constant rate 
of a particle along a straight line with constant from zero to v and then remains constant for an
acceleration with zero initial velocity.
interval and finally decreases to zero at a constant
v
v rate . The total distance travelled by the trains is
(A) (B) l. The time taken to complete the journey is t.
Then-
0 0 l (   )
t t (A) t =

x x l v 1 1
(B) t =    
(C) (B) v 2    

2l
(C) t is minimum when v =
0 0 (  )
t t
2l
24. A man who can swim at a velocity v relative to (D) t is minimum when v =
(  )
water wants to cross a river of width b, flowing
with a speed u.
(A) The minimum time in which he can cross the 27. A particle moves in x-y plane and at time t is at

river is
b the point (t2, t3 – 2t), then which of the following
v is/are correct?
(B) He can reach a point exactly opposite on the
(A) At t = 0, particle is moving parallel to
b
back in time t = if v > u y-axis
v  u2
2

(B) At t = 0, direction of velocity and acceleration


(C) He cannot reach the point exactly opposite on
the bank if u > v are perpendicular
(D) He cannot reach the point exactly opposite on 2
the bank if v > u (C) At t = , particle is moving parallel to axis
3

25. The figure shows the velocity (v) of particle (D) At t = 0, particle is at rest
plotted against time (t).
v 28. A car is moving with uniform acceleration along
a straight line between two stops X and Y. Its
T speed at X and Y are 2 m/s and 14 m/s. Then-
O t
2T
(A) Its speed at mid-point of XY is 10 m/s
(B) Its speed at a point A such that XA : AY
(A) The particle changes its direction f motion at = 1: 3 is 5 m/s
some point (C) The time to go from X to the mid-point of XY
(B) The acceleration of the particle remains is double of that to go from mid-point to Y
constant
(D) The distance travelled in first half of the total
(C) The displacement of the particle is zero
time is half of the distance travelled in the
(D) The initial and final speeds of the particle are
the same second half of the time

98
PHYSICS -X I-IIT-JEE MOTION I N ONE DIMEN SIO N

29. A particle is thrown in vertically in upward 31. The maximum height reached by ball, as
direction and passes three equally spaced measured from the ground would be
(A) 73.65 m (B) 116.25 m
windows of equal heights. Then
(C) 82.56 m (D) 63.25 m
3
32. Displacement of ball with respect to ground
2
during its flight would be
1 (A) 16.25 m (B) 8.76 m
(C) 20.24 m (D) 30.56 m

33. The maximum separation between the floor of


Ground elevator and the ball during its flight would be
(A) 12 m (B) 15 m
(A) The average speed of the particle while (C) 9.5 m (D) 7.5 m
passing the windows satisfy the relation
Passage # 2 (Q.No. 34 to 36)
v a1  v a 2  v a 3 A man is driving out of his driveway by backing
up. He realizes he has forgotten his lunch, so he
(B) The time taken by the particle to cross the
pulls back into the driveway. Car experts agree
windows satisfies the relation t1 < t2 < t3 that the best way to do this is to press on the
(C) The magnitude of the acceleration of the brake until the car comes to a complete stop, shift
from reverse into first gear, then accelerate
particle while crossing the windows, satisfies forward.
the relation a1 = a2  a3 The driver, however, shifts into first gear while
the car is rolling backward and pushes on the
(D) The change in the speed of the particle, while accelerator until he is going forward. This causes
crossing the windows, would satisfy the some wear on the transmission. The following
chart shows some data about his progress.
relation v1 < v2 < v3 (Negative velocity = backwards)
t(s) x(m) v(m/s)
Question Comprehension based questions 0.0 1.35 –1.8
0.5 0.60 – 1.2
Passage-1 (Q.No. 30 to 33)
1.0 0.15 – 0.6
An elevator without a ceiling is ascending up
1.5 0.00 0.0
with an acceleration of 5 m/s2. A boy on the 2.0 0.15 0.6
elevator shoots a ball in vertical upward direction 2.5 0.60 1.2
from a height of 2m above the floor of elevator. 3.0 1.35 1.8
At this instant the elevator is moving up with a 34. What is the value of his average velocity ?
velocity of 10 m/s and floor of the elevator is at a
(A) –1.8 m/s (B) 0.0 m/s
height of 50 m from the ground. The initial speed
of the ball is 15 m/s with respect to the elevator. (C) 1.2 m/s (D) 1.8 m/s
Consider the duration for which the ball strikes
the floor of elevator in answering following 35. Which of the following is evidence that the
questions (g = 10 m/s2) acceleration is uniform?
(A) The displacement x is always non-negative
30. The time in which the ball strikes the floor of (B) The velocity is always increasing
elevator is given by (C) The velocity becomes zero at t = 1.5 s
(A) 2.13 s (B) 2.0 s (D) Equal intervals of time correspond to equal
(C) 1.0 s (D) 3.12 s intervals of velocity

99
MOTION I N ONE DIMEN SIO N PHYSICS -X I-IIT-JEE
36. Which best represents the graph of acceleration 39. After 10 s of the start of motion of both objects A
versus time ?
and B, find the value of velocity of A if
(A) (B) uA = 6 m/s, uB = 12 m/s and at T velocity of A is
t
t 8 m/s and T = 4 s -
(C) (D) (A) 12 m/s (B) 10 m/s
t
t (C) 15 m/s (D) None of these
Passage # 3 (Q.No. 37 to 39)
Passage # 4 (Q.No. 40 to 42)
A situation in which two objects A and B start
Consider a particle moving along x-axis as shown
their motion from same point in same direction.
in figure. Its distance from the origin O is
The graph of their velocities against time is described by the coordinate x, which varies with
shown in figure. uA and uB are the initial time. At a time t1, the particle is at point P, where
velocities of A and B respectively. T is the time its coordinate is x1, and at time t2 it is at point Q,
at which their velocities become equal after start where its coordinate is x2. The displacement
of motion. You cannot use the data of one during the time interval from t1 to t2 is the vector
question while solving another question of the from P to Q: the x-component of this vector is
same set. So all the questions are independent of (x2 – x1) and all other components are zero.
each other. It is convenient to represent the quantity x2 – x1,
then change in x, by means of a notation using
Velocity of A the Greek letter  (capital delta) to designate a
uB
change in any quantity. Thus we write x = x2 –
x1 in which x is not a product but is to be
interpreted as a single symbol representing the
uA Velocity of B
change in the quantity x. Similarly, we denote the
time interval from t1 to t2 as t = t2 – t1.
T Y
t
X1
P Q
37. If the value of T is 4 s then the time after which A X
O
will meet B is -
X2 – X1 = X
(A) 12 s (B) 6 s
X2
(C) 8 s (D) data insufficient

38. Let vA and vB be the velocities of the particles A The average velocity of the particle is defined as
and B respectively at the moment when A and B the ratio of the displacement x to the time
meets after start of the motion. If uA = 5 m/s and interval t. We represent average velocity by the
uB = 15 m/s, then the magnitude of the difference letter v with a bar (v) to signify average value.
of velocities vA and vB is - Thus
(A) 5 m/s (B) 10 m/s x 2  x1 x
v 
(C) 15 m/s (D) data insufficient t 2  t1 t

100
PHYSICS -X I-IIT-JEE MOTION I N ONE DIMEN SIO N

40. A particle moves half the time of its journey with 43. If the plane has a eastward heading, and a 20 m/s
u. The rest of half time it moves with two wind blows towards the southwest, then the
velocities V1 and V2 such that half the distance it plane's speed is -
covers with V1 and the other half with V2. Find (A) 80 m/s
the net average velocity. Assume straight line (B) more than 80 m/s but less than 100 m/s
motion. (C) 100 m/s
u(V1  V2 )  2V1V2 2u (V1  V2 ) (D) more than 100 m/s
(A) (B)
2(V1  V2 ) 2u  V1  V2
44. The pilot maintains an eastward heading while a
u (V1  V2 ) 2V1V2
(C) (D) 20 m/s wind blows northward. The plane's
2 V1 u  V1  V2
velocity is deflected from due east by what
41. A particle moves according to the equation
angle?
x = t2 + 3t + 4. The average velocity in the first 5s
(A) sin–1 (1/5) (B) cos–1 (1/5)
is-
(C) tan–1 (1/5) (D) none of these
–1 –1
(A) 8 ms (B) 7.6 ms
(C) 6.4 ms–1 (D) 5.8 ms–1 45. Because the 20 m/s northward wind persists, the
pilot adjust the heading so that the plane's total
2
t velocity is eastward. By what angle does the new
42. A particle moves from A to B such that x  t  .
2
heading differ from due east ?
Its average speed from t = 0 to t = 2s is-
1 1
(A) 2 ms–1 (B) 1 ms–1 (A) sin–1 (B) cos–1
5 5
(C) zero (D) None of these
1
(C) tan–1 (D) none of these
5
Passage # 5 (Q.No. 43 to 45)
Passage # 6 (Q. No. 46 to 48)
When an airplane flies, its total velocity with A spaceship launched vertically up from Mars
respect to the ground is : has reached a height of 320 m and a velocity of
Vtotal = vplane + vwind 80 m/s at time t = 0. At this instant its controls
Where vplane denotes the plane velocity through are switched off. It continues to move up under
motionless air, and vwind denotes the wind's 2
the influence of Martian gravity (i.e. 3.72 m/s ).
velocity crucially, all the quantities in this At the same instant another spaceship at height
equation are vectors. The magnitude of a velocity 1500 m is moving downward at 25 m/s and
2
vector is often called the "speed". slowing down at rate of 0.8 m/s .
Consider an airplane whose speed through
46. The two space ships will be at same height of
motionless air is 100 m/s. To reach its
other time t = ?
destination, the plane must fly east. (A) 19 sec
The "heading" of a plane is the direction in which (B) 27.4 sec
the nose of the plane points. So, it is the direction (C) 50 sec
(D) will reach at same height only once
in which the engines propel the plane.

101
MOTION I N ONE DIMEN SIO N PHYSICS -X I-IIT-JEE

47. How high above the planet surface will the first Passage # 8 (Q.No. 52 to 54)
meeting take place ? An auto-mobile and a truck starts from rest at the
same instant, with the automobile initially at
(A) 3 km (B) 18 km
some distance behind the truck. The truck has a
(C) 1.14 km (D) 5.14 km constant acceleration of 2.1 m/s
2
and the
2
automobile has acceleration 3.4 m/s . The auto-
48. Which of the graph best explains the motion ? mobile overtakes the truck after the truck has
y y moved 40 m.
1500 1500
52. How far was the automobile behind the truck
(A) 320 (B) 320 initially ?
(A) 24.8 m (B) 30 m (C) 50 m (D) 60 m
t t

y y 53. The distance moved by automobile before it


1500 1500
overtakes the truck–
(A) 54.8 m (B) 64.8 m (C) 74.8 m (D) 68.8 m
(C) 320 (D) 320

t t 54. Which is the best possible graph ?


x x
truck truck
Passage # 7 (Q.No. 49 to 51) (A) (B)
A spaceship ferrying walks to Moon base. It takes t t
automobile automobile
a straight line path from the earth to the moon, a
x truck x
distance of 384,000 km. Suppose it accelerates at
2 (C)
20 m/s for the first 15 min of the trip. Then t (D)
truck
travels with same speed until the last 15 min, automobile t
2
when it decelerates at 20 m/s just coming to rest
Passage # 9 (Q.No. 55 to 57)
as it reaches the moon. Two cars A & B travel in a straight line from
same point. The distance of A from the starting
49. What distance does it travel while decelerating? point is given by xA = t + t2 where  = 2.6 m/s
3 3
(A) 6.1 × 10 km (B) 5.1 × 10 km and  = 1.2 m/s2. The distance B from the starting
3 3 2
(C) 3.1 × 10 km (D) 8.1 × 10 km point is xB = t2 – t3; where  = 2.8 m/s and
3
 = 0.2 m/s .
50. What fraction of the total distance it travelled at 55. Which car is ahead just after they leave the
constant speed ? starting point?
(A) 0.95 (B) 0.23 (C) 0.40 (D) 0.52
(A) A

51. What total time is required for the trip ? (B) B


(A) 170 min (B) 270 min (C) Both are with same velocity
(C) 370 min (D) 470 min (D) Cannot be determined

102
PHYSICS -X I-IIT-JEE MOTION I N ONE DIMEN SIO N

56. At what times are the car at same point ? 58. The particle is moving with constant speed

(A) 2.7 sec (A) In graphs (i) & (iii) (B) In graphs (i) & (iv)

(B) 2.27 sec & 5.73 sec (C) In graphs (i) & (ii) (D) In graphs (i)

(C) 5.73 sec & 2.6 sec

(D) 0, 2.27 sec & 5.73 sec 59. The particle has negative acceleration

(A) In graph (i) (B) In graph (ii)


57. At what times is the distance from A to B neither
(C) In graph (iii) (D) In graph (iv)
increasing nor decreasing ?

(A) 1.66 sec (B) 4.33 sec

(C) 1.66 sec & 1 sec (D) 1 sec & 4.3 sec

Passage # 10 (Q.No. 58 & 59)

Study the following graphs

x x

t t
(i) (ii)

x x

t t
(iii) (iv)

103
MOTION I N ONE DIMEN SIO N PHYSICS-X I-IIT-JEE

EXERCISE # 3
Question Column Match Type Questions v a

1. Match the following two columns : (D) (s)


Column I Column II t t
a 4. A particle is dropped vertically downward under
(p) speed must be gravity. Consider the downward direction as
(A) increasing positive. Ball collides elastically with ground.
t Column – I Column – II
(A) The distance travelled (p)
a by particle varies with
(q) speed must be time as
(B) t decreasing
(B) Velocity of particle (q)
changes with time as
a
(r) speed may be (C) Displacement of (r)
(C) increasing particle depends
t on time as
a (D) Dependency of (s)
(s) speed may be
acceleration on time
(D) decreasing
t is given by
5. For the velocity-time graph shown in figure, in a
time interval from t = 0 to t = 6 s, match the
2. Match the following two columns : following :
Column I Column II v (m/s)
(A) v = –2 î , a = –4 ĵ (p) speed increasing 10

(q) speed decreasing 2 4 6


(B) v = 2 î , a = 2 î + 2 ĵ
t (s)
(C) v = –2 î , a = + 2 ĵ (r) speed constant

(A) v = 2 î , a = –2 î + 2 ĵ (s) speed increasing


Column-I Column-II
3. Column I Column II (A) Change in velocity (p) – 5/3 SI unit
(B) Average acceleration (q) – 20 SI unit
a
v (C) Total displacement (r) – 10 SI unit
t (D) Acceleration at t = 3s (s) – 5 SI unit
(A) (p)
6. x and y-coordinates of a particle moving in x-y
t –a0
plane are x = 1 – 2t + t2 and y = 4 – 4t + t2
v a For the given situation match the following two
columns :
(B) (q)
Column I Column II
t t
(A) y-component of
v a velocity when it (p) +2 SI unit
crosses the y-axis
(C) (r)
(B) x-component of
t t velocity when it (q) – 2SI units
crosses the x-axis

104
PHYSICS-X I-IIT-JEE MOTION I N ONE DIMEN SIO N

(C) Initial velocity of 10. For a particle moving along the x-axis, if
(r) +4 SI units
particle acceleration (constant) is acting along negative x-
(D) Initial acceleration axis, then match the entries of Column I with
(s) None of the above entries of Column II.
of particle
Column I Column II
7. The equation of one dimensional motion of
(A) Initial velocity > 0 (p) Particle may move in
particle is described in column I. At t = 0,
positive x-direction with
particle is at origin and at rest. Match the column increasing speed
I with the statements in column II. (B) Initial velocity < 0 (q) Particle may move in
Column-I Column-II positive x-direction with
2
(A) x = (3t + 2)m (p) velocity of particle at decreasing speed
t = 1 s is 8 m/s (C) x > 0 (r) Particle may move in
(B) v = 8t m/s (q) particle moves with negative x-direction
uniform acceleration with increasing speed
(C) a = 16 t (r) particle moves with (D) x < 0 (s) Particle may move in
negative x-direction
variable acceleration
2 with decreasing speed
(D) v = 6t – 3t (s) particle will
change its direction 11. Figure shows the position-time graph of particle
some time moving along a straight line. Match the entries of
Column I with the entries of Colum II.
8. A particle moves along a straight line such that its x
displacement S varies with time t as A B
S =  + t + t2,
Column I Column II
C D
(A) Acceleration at t = 2s (p)  + 5
(B) Average velocity t
(q) 2
during third second
Column I Column II
(C) Velocity at t = 2s (r)  (A) The particle A is (p) accelerating
(D) Initial displacement (s)  = 2 (B) The particle B is (q) decelerating
(C) The particle C is (r) speeding up
9. For a body projected vertically up with a velocity (D) The particle D is (s) slowing down

v 0 from the ground, match the following 12. A ball is thrown vertically upwards from the top
Column I Column II of a cliff. Take starting position of motion as
 origin and upward direction as positive. Column
(A) v av (Average velocity) (p) Zero for round trip
  I specifies the position velocity, and/or
v1  v 2 acceleration of the particle at any instant.
(q) over any
(B) uav (Average speed) 2 Column II gives their sign, (+) or (–), at that
time interval moment. Match the columns.
v0 Column I Column II
(r) over the total
(C) Tascent 2 (A) When the ball is above the
time of its flight point of projection, its (p) Always positive
v0 displacement is
(D) Tdescent (s) (B) When the ball is above the
g
point of projection, its (q) Always negative
velocity is

105
MOTION I N ONE DIMEN SIO N PHYSICS-X I-IIT-JEE

(C) When the ball is above the back to the bridge. The swimmer and the cork
point of projection, its reach the bridge at the same time. The swimmer
(r) May be positive
acceleration is or may be has been swimming at a constant speed. How fast
negative does the water in the canal flow in km/hr.
(D) When the ball is below the
(s) May be zero
point of 16. Figure shows the graph of the x-co-ordinate of a
13. The displacement versus time curve is given particle going along the x-axis as function of
(shown in figure). Sections OA and BC are time. Find the instantaneous speed of particle at t
= 12.5 s (in m/s)
parabolic. CD is parallel to the time axis x
D
C
s A B A
8m
O 4m
t

Column I Column II O 4s 8s 12s 16s B


(p) Velocity increases with time
(A) OA
linearly
(B) AB (q) Velocity decreases with time 17. Figure shows the graph of velocity versus time
(r) Velocity is independent of for a particle going along x axis. Initially at t = 0,
(C) BC
time particle is at x = 3m. Find position of particle at
t = 2s. (in m)
(D) CD (s) Velocity is zero v(m/s)

10
Question Numerical Type Questions

14. The particle moves with rectilinear motion given 2


the acceleration-displacement (a-S) curve is
O 8 t
shown in figure, the velocity after the particle has
traveled 30 m is n × 10 m/s. If the initial velocity
is 10 m/s. Find n? 18. A Staircase contains three steps each 10 cm high
a(m/s )2 and 20 cm wide. What should be the minimum
horizontal velocity of a ball rolling off the
uppermost plane so as to hit directly the lowest
10
plane. (in m/s)

19. A man is running with a speed 8 m/s constant in


S(m) magnitude and direction passes under a lantern
15 30
hanging at a height 10 m above the ground. The
15. A swimmer jumps from a bridge over a canal and velocity which the edge of the shadow of the
swims 1 km up stream. After that first km, he man's head moves over the ground with if his
passes a floating cork. He continues swimming
height is 2 m is n × 2 m/s. Find n ?
for half an hour and then turns around and swims

106
PHYSICS-X I-IIT-JEE MOTION I N ONE DIMEN SIO N

20. In the figure shown, the velocity of lift is 2m/s 25. The acceleration of particle varies with time as
while string is winding on the motor shaft with shown.
velocity 2m/s and block A is moving downwards
with a velocity of 2m/s, the velocity of block B in a(m/s2)
m/s is-
2 m/s

t(s)
1

–2
B A
2 m/s Calculate the displacement of the particle in the
interval from t = 2s to t = 4s (approx.)
21. A body initially at rest moving along x-axis in
such a way so that its acceleration Vs 26. From a lift moving upwards with a uniform
displacement plot is as shown in figure. What
acceleration a = 2 m/s2, a man throws a ball
will be the maximum velocity of particle in
m/sec. vertically upwards with a velocity v = 12 m/s2
a relative to the lift. The ball comes back to the
1m/s2 man after a time t. Find the value of t in seconds.

27. A balloon rises from rest on the ground with


S constant acceleration 1 m/s2. A stone is dropped
0.5 1m
when the balloon has risen to a height of 39.2 m.
22. Two trains of lengths 180 m are moving on Find the time taken by the stone to reach the
parallel tracks. If they move in the same direction
then they cross each other in 15 s, and if they ground.
move in opposite directions then they cross in 28. A body is thrown up with a velocity 100 m/s. It
1 travels 5 m in the last second of its journey. If
7 s, then sum of their velocities is
2 the same body is thrown up with a velocity 200
n × 6 m/s. Find n?
m/s, how much distance (in meter) will it travel
23. An ant runs from an ant-hill in a straight line so in the last second (g = 10 m/s2) ?
that its velocity is inversely proportional to the
distance from the centre of the ant-hill. When the
29. In quick succession, a large number of balls are
ant is at point A at a distance 1 = 1 m from the
thrown up vertically in such a way that the next
centre of the ant-hill, its velocity v1 = 2 cm/s, the ball is thrown up when the previous ball is at the
time will it take ant to run from point A to point
maximum height. If the maximum height is 5 m.,
B is n × 15 sec, which is at a distance  2 = 2 m
then find the number of the thrown up per second
from the centre of the ant-hill, find n ?
(g = 10 m/s2).
24. To stop a car, first you require a certain reaction
time to begin braking; then the car slows under 30. A police jeep is chasing a culprit going on a
the constant braking deceleration. Suppose that
motorbike. The motorbike crosses a turning at a
the total distance moved by our car during these
two phases is 56.7k m when its initial speed is speed of 72 km/h. The jeep follows it at a speed
80.5 km/h and 24.4 m when its initial speed is of 90 km/h, crossing the turning 10s later than the
48.8 km/h. What the magnitude of the declaration bike.
in m/s2 (approx.)?

107
MOTION I N ONE DIMEN SIO N PHYSICS-X I-IIT-JEE

31. A train starts from station A with uniform 36. Two bodies 1 and 2 are projected simultaneously
acceleration a1 for some distance and then goes
with velocities v1 = 2 m/s and v2 = 4 m/s
with uniform retardation a2 for some more
distance to come to rest at station B. The respectively. up from the top of a cliff of height h
distance between stations A and B is 4 km and = 10 m and the body 2 is projected vertically up
the train takes 1/15 h to complete this journey. If
accelerations are in km per minute unit, then from the bottom of the cliff. If the bodies meet,
1 1 find the time (in s) of meeting of the bodies.
show that  = x. Find the value of x.
a1 a 2
32. In a car race, car A takes 4 s less than car B at the
finish and passes the finishing point with a t
v1
velocity v more than the car B. Assuming that
t=0 s1
the cars start form rest and travel with constant
accelerations a1 = 4 m/s2 and a2 = 1 m/s2
respectively, find the velocity of v in m/s. s2
v2

33. A cat, on seeing a rat a distance d = 5m, starts


with velocity u = 5 m/s and moves with t=0
2
acceleration  = 2.5 m/s in order to catch it,
while the rate with acceleration  starts from rest. 37. A particle moves in a straight line. Its position

For what value of  will the cat overtake the rat ? (in m) as function of time is given by
2
(in m/s ). x = (at2 + b)
What is the average velocity in time interval
34. On a two-lane road, car A is travelling with a t = 3s to t = 5s in m/s ? (Where a and b are
speed of 36 km/h. Two cars B and C approach constants and a = 1 m/s2, b = 1 m)
car A in opposite directions with a speed of 54
38. A particle can move only along x-axis. Three
km/h each. At a certain instant, when the
pairs of initial and final positions of particle at
distance AB is equal to AC, both being 1 km, B
two successive times are given
decides to overtake A before C does. What
minimum acceleration of car B is required to Pair Initial position Final position
avoid an accident ? 1 –3 m +5 m
35. A particle moves vertically with an upward initial 2 –3 m –7 m
speed v0 = 10.5 m/s. If its acceleration varies 3 +7 m –3 m
with time as shown in a-t graph in figure, find the
Find the sum of magnitudes of displacement in the
velocity of the particle at t = 4 s.
pairs which given negative displacement in m.
a(m/s2)
5 39. Acceleration of particle moving rectilinearly
is a = 4 – 2x (where x is position in metre and
t(s)
0 1 2 3 4 a in m/s 2). It is at instantaneous rest at x = 0.

–10 At what position x (in metre) will the particle


again come to instantaneous rest ?

108
PHYSICS-X I-IIT-JEE MOTION I N ONE DIMEN SIO N

40. A big Diwali rocket is projected vertically spreads out as an expanding, brilliant sphere.
upward so as to attain a maximum height of 160 The bottom of this sphere just touches the ground
m. The rocket explodes just as it reaches the top when its radius is 80m. With what speed (in m/s)
of its trajectory sending out luminous particles in are the luminous particles ejected by the
all possible directions all with same speed v. The explosion ?
display, consisting of the luminous particles,

109
MOTION I N ONE DIMEN SIO N PHYSICS-X I-IIT-JEE

EXERCISE # 4
Question Previous Year (JEE Main) 7. An automobile travelling with a speed of
60 km/h, can brake to stop, within a distance of
1. From a building two balls A and B are thrown such
20 m. If the car is going twice as fast i.e.,
that A is thrown upwards and B downwards (both
120 km/h, the stopping distance will be –
vertically) with same velocity. If vA and vB are
[AIEEE-2004]
their respective velocities on reaching the ground, (A) 20 m (B) 40 m
then – [AIEEE-2002]
(C) 60 m (D) 80 m
(A) vB > vA
(B) vA = vB 8. A bullet fired into a fixed target loses half of its
(C) vA > vB velocity after penetrating 3 cm. How much further
(D) Their velocities depends on their masses it will penetrate before coming to rest assuming
that it faces constant resistance to motion ?
2. If a body looses half of its velocity on penetrating
[AIEEE-2005]
3 cm in a wooden block, then how much will it
(A) 3.0 cm (B) 2.0 cm
penetrate more before coming to rest ?
(C) 1.5 cm (D) 1.0 cm
[AIEEE-2002]
(A) 1 cm (B) 2 cm (C) 3 cm (D) 4 cm
9. The relation between time t and distance x is
3. Speeds of two identical cars are u and 4u at a 2
t = ax + bx where a and b are constants. The
specific instant. The ratio of the respective acceleration is - [AIEEE-2005]
distance in which the two cars are stopped from (A) –2abv
2
(B) 2bv
3
that instant by same breaking force is – 3 2
(C) –2av (D) 2av
[AIEEE-2002]
(A) 1 : 1 (B) 1 : 4 (C) 1 : 8 (D) 1 : 16
10. A particle is moving eastwards with a velocity of
4. A car, moving with a speed of 50 km/hr, he can 5m/s. In 10 seconds the velocity changes to 5 ms–
be stopped by brakes after at least 6m. If the same 1 northwards. The average acceleration in this
car is moving at a speed of 100 km/hr, the
time is - [AIEEE-2005]
minimum stopping distance is –
1 –2
[AIEEE-2003] (A) ms towards north-east
2
(A) 18 m (B) 24 m (C) 6 m (D) 12 m
1 –2
5. A particle moves in a straight line with (B) ms towards north
2
retardation proportional to its displacement. Its
(C) zero
loss of kinetic energy for any displacement x is
1 –2
proportional to – [AIEEE-2004] (D) ms towards north-west
2
(A) x2 (B) ex (C) x (D) logex
6. A ball is released from the top of a tower of 11. A particle located at x = 0 at time t = 0, starts
height h meters. It takes T seconds to reach the moving along the positive x-direction with a
ground. What is the position of the ball at T/3 velocity 'v' that varies as v =  x . The
second – [AIEEE-2004] displacement of the particle varies with time as-
(A) h/9 meters from the ground [AIEEE 2006]
(B) 7h/9 meters from the ground 1/2 3
(A) t (B) t
(C) 8h/9 meters from the ground 2
(C) t (D) t
(D) 17h/18 meters from the ground

110
PHYSICS-X I-IIT-JEE MOTION I N ONE DIMEN SIO N

12. A body is at rest at x = 0. At t = 0, it starts 15. An object, moving with a speed of 6.25 m/s, is
moving in the positive x-direction with a constant decelerated at a rate given by :
acceleration. At the same instant another body dv
passes through x = 0 moving in the positive x-  –2.5 v
direction with a constant speed. The position of dt
the first body is given by x1(t) after time ‘t’ and where v is the instantaneous speed. The time
that of second body by x2(t) after the same time taken by the object, to come to rest, would be :
interval. Which of the following graphs correctly [AIEEE-2011]
describes (x1 – x2) as a function of time ‘t’ ? (A) 1 s (B) 2 s
[AIEEE 2008] (C) 4 s (D) 8 s
(x1 – x2) (x1 – x2)
16. From a tower of height H, a particle is thrown
(A) (B) vertically upwards with a speed u. The time taken
t t by the particle, to hit the ground, is n times that
O O
taken by it to reach the highest point of its path.
(x1 – x2) (x1 – x2) The relation between H, u and n is –
[JEE-Main 2014]
(C) (D) (A) gH = (n – 2)2u2
(B) 2gH = nu2 (n – 2)
O
t
O
t (C) gH = (n – 2)u2
(D) 2gH = n2u2
13. A particle has an initial velocity of 3î  4 ĵ and an
acceleration of 0.4î  0.3ˆj . Its speed after 17. A body is thrown vertically upwards. Which one of
10 s is – [AIEEE - 2009] the following graphs correctly represent the velocity
(A) 10 units (B) 7 2 units vs time ? [JEE-Main 2017]
(C) 7 units (D) 8.5 units
14. Consider a rubber ball freely falling from a height v
v
h = 4.9 m onto a horizontal elastic plate. Assume (A) (B)
that the duration of collision is negligible and the t
collision with the plate is totally elastic. Then the
velocity as a function of time and the height as a t
function of time will be - v v
[AIEEE - 2009] (C) (D)
t

(A) t

18. All the graphs below are intended to represent the


same motion. One of them does it incorrectly.
Pick it up. [JEE Main 2018]
(B) Velocity

Time
(A) (B)
(C)

(D)
(C) (D)

111
MOTION I N ONE DIMEN SIO N PHYSICS-X I-IIT-JEE
19. In a car race on straight road, car A takes a time t 23. A person standing on an open ground hears the
less than car B at the finish and passes finishing sound of a jet aeroplane, coming from north at an
point with a speed 'v' more than that of car B. angle 60° with ground level. But he finds the
Both the cars start from rest and travel with aeroplane right vertically above his position. If v
constant acceleration a1 and a2 respectively. Then is the speed of sound, speed of the plane is :
[JEE Main Online 2019]
'v' is equal to – [JEE Main Online 2019]
a1  a 2 (A) 2v / 3 (B) v
(A) t (B) 2a 1a 2 t
2 (C) v/2 (D) 3 v/2
2a1a 2
(C) t (D) a 1a 2 t 24. Ship A is sailing towards north-east with velocity
a1  a 2 
v  30iˆ  50jˆ km/hr where î points east and
20. A particle starts from the origin at time t = 0 and ˆj, north. Ship B is at a distance of 80 km east
moves along the positive x-axis. The graph of
and 150 km north of Ship A and is sailing
velocity with respect to time is shown in figure.
towards west at 10 km/hr. A will be at minimum
What is the position of the particle at time t = 5s ?
distance from B in : [JEE Main Online 2019]
[JEE Main Online 2019]
(A) 4.2 hrs. (B) 2.2 hrs.
v
(m/s) (C) 3.2 hrs. (D) 2.6 hrs.
3
25. A particle starts from origin O from rest and
2
moves with a uniform acceleration along the
1
positive x-axis. Identify all figures that correctly
1 2 3 4 5 6 7 8 9 10 t (s) represent the motion qualitatively. (a = acceleration,
(A) 6 m (B) 9 m (C) 3 m (D) 10 m v = velocity, x = displacement, t = time)
[JEE Main Online 2019]
21. A particle is moving along a circular path with a
constant speed of 10 ms–1. What is the
magnitude of the change is velocity of the a v
(i) (ii)
particle, when it moves through an angle of 60° O t O t
around the centre of the circle?
[JEE Main Online 2019] x x

(A) zero (B) 10 m/s


(iii) (iv)
(C) 10 3 m/s (D) 10 2 m/s O t O t
(A) (i), (ii), (iii) (B) (i)
22. A passenger train of length 60m travels at a speed (C) (i), (ii), (iv) (D) (ii), (iii)
of 80 km/hr. Another freight train of length 120
m travels at a speed of 30 km/hr. The ratio of 26. The stream of a river is flowing with a speed of
times taken by the passenger train to completely 2km/h. A swimmer can swim at a speed of
cross the freight train when : (i) they are moving 4km/h. What should be the direction of the
in the same direction, and (ii) in the opposite swimmer with respect to the flow of the river to
directions is [JEE Main Online 2019] cross the river straight ?
(A) 5 / 2 (B) 25 / 11 (C) 3 / 2 (D) 11 / 5 [JEE Main Online 2019]
(A) 60° (B) 150° (C) 90° (D) 120°

112
PHYSICS-X I-IIT-JEE MOTION I N ONE DIMEN SIO N

27. A ball is thrown vertically up (taken as +z-axis) T(x) T(x)

from the ground. The correct momentum-height


(p-h) diagram is : [JEE Main Online 2019] (A) x
(B) x
 

p p T(x) T(x)

(A) O
h (B) O
h

(C) x
(D)
  x

p p

(C) O h (D) O
h
31. A man (mass = 50 kg) and his son (mass = 20 kg)
are standing on a frictionless surface facing each
other. The man pushes his son so that he starts
28. The position of a particle as a function of time t,
moving at a speed of 0.70 ms–1 with respect to
is given by x (t) = at + bt2 – ct3, where a, b and c
the man. The speed of the man with respect to the
are constants. When the particle attains zero
acceleration, then its velocity will be: surface is : [JEE Main Online 2019]

[JEE Main Online 2019] (A) 0.20 ms–1 (B) 0.14 ms–1
b2 b2 (C) 0.47 ms–1 (D) 0.28 ms–1
(A) a + (B) a +
4c c
32. A particle is moving with speed n = b x along
b2 b2
(C) a + (D) a + positive x-axis. Calculate the speed of the particle
2c 3c
at time t =  (Assume that the particle is at origin
29. A bullet of mass 20 g has an initial speed of 1m/s,
at t = 0). [JEE Main Online 2019]
just before it starts penetrating a mud wall of
thickness 20 cm. if the wall offers a mean b2 b2 b2
(A) (B) (C) b2  (D)
4 2 2
resistance of 2.5 × 10–2 N, the speed of the bullet
after emerging from the other side of the wall is
33. Train A and train B are running on parallel tracks
close to [JEE Main Online 2019]
in the opposite directions with speeds of 36
(A) 0.4 ms–1 (B) 0.1 ms–1
km/hour and 72 km/hour, respectively. A person
(C) 0.3 ms–1 (D) 0.7 ms–1
is walking in train A in the direction opposite to
its motion with a speed of 1.8 km/hr. Speed (in
30. A uniform rod of length  is being rotated in a
ms–1) of this person as observed from train B will
horizontal plane with a constant angular speed
about an axis passing through one of its ends. If be close to : (take the distance between the tracks
the tension generated in the rod due to rotation is as negligible) [JEE Main 2020]
T(x) at a distance x from the axis, then which of
the following graphs depicts it most closely? (A) 30.5 ms–1 (B) 29.5 ms–1
[JEE Main Online 2019] (C) 31.5 ms–1 (D) 28.5 ms–1

113
MOTION I N ONE DIMEN SIO N PHYSICS-X I-IIT-JEE
34. A particle is moving unidirectional on a 37. The speed verses time graph for a particle is
horizontal plane under the action of a constant shown in the figure. The distance travelled (in m)
power supplying energy source. The by the particle during the time interval t = 0 to t =
displacement (s) - time (t) graph that describes 5s will be ______ : [JEE Main 2020]
the motion of the particle is (graphs are drawn 10

schematically and are not to scale) : 8


u
–1 6
[JEE Main 2020] (ms )
4
s s
2
(A) (B) 1 2 3 4 5
time (s) 
t t
s s
38. The velocity (v) and time (t) graph of a body in a
(C) (D) straight line motion is shown in the figure. The
t t point S is at 4.333 seconds. The total distance
covered by the body in 6s is :
35. A Tennis ball is released from a height h and [JEE Main 2020]
after freely falling on a wooden floor it rebounds A B
v(m/s) 4
and reaches height h/2. The velocity versus 2 S D t(in s)
height of the ball during its motion may be 0
1 2 3 4 5 6
–2
represented graphically by : (graph are drawn
schematically and on not to scale)
(A) 12m (B) (49/4) m
[JEE Main 2020]
(C) 11 m (D) (37/3) m
 

h/2
39. A particle moving in the xy plane experiences a
h 
(A) h/2
h( ) (B) h( ) velocity dependent force F  k (v y ˆi  v x ˆj),
where vx and vy are the x and y components of
  
 its velocity v. If a is the acceleration of the
h/2
h particle, then which of the following statements is
(C) h( ) (D) h( )
h/2 h true for the particle ? [JEE Main 2020]
 
(A) Quantity v  a is constant in time.
(B) Kinetic energy of particle is constant in time.
 
36. A small ball of mass is thrown upward with (C) Quantity v  a is constant in lime.

velocity u from the ground. The ball experiences (D) F Arises due to a magnetic field.
a resistive force mkv2 where v is its speed. The 40. A particle starts from the origin at t = 0 with an
maximum height attained by the ball is :
initial velocity of 3.0 î m/s and moves in the x-y
[JEE Main 2020]
plane with a constant acceleration (6.0iˆ  4.0j)
ˆ m
1 ku 2 1  ku 2 
(A) tan 1 (B) ln 1   / s2. The x-coordinate of the particle at the instant
2k g 2k  g 
when its y-coordinate is 32 m is D meters. The
1 ku 2 1  ku 2  value of D is : [JEE Main 2020]
(C) tan 1 (D) ln 1  
k 2g k  2g  (A) 50 (B) 32 (C) 60 (D) 40

114
PHYSICS-X I-IIT-JEE MOTION I N ONE DIMEN SIO N

41. The distance x covered by a particle in one Question Previous Year (JEE Advanced)
2
dimensional motion varies with time t as x = 46. A ball is dropped vertically from a height d above
2
at + 2bt + c. If the acceleration of the particle the ground. It hits the ground and bounces up
depends on x as x–n, where n is an integer, the
vertically to a height d/2. Neglecting subsequent
value of n is __________ . [JEE Main 2020]
motion and air resistance, its velocity v varies
with the height h above the ground as
42. Two spherical balls having equal masses with
[IIT – 2000]
radius of 5cm each are thrown upwards along the
same vertical direction at an interval of 3 s with v v
the same initial velocity of 35 m/s, then these d
(A) h (B) h
balls collide at a height of ___ m. (take g = 10 d
m/s2) [JEE MAIN– 2021]
v v
43. If the velocity of a body related to displacement x is
(C) d (D) d
h h
given by v = 5000  24x m/s , then the
acceleration of the body is ______ m/s2.
[JEE MAIN– 2021]
47. A block is released from rest at t = 0 on a
44. Water drops are falling from a nozzle of a shower frictionless inclined plane. The distance travelled
onto the floor, from a height of 9.8 m. The drops by the block in tn–1 sec to tn sec is represented by
fall at a regular interval of time. When the first drop
Sn
strikes the floor, at that instant, the third drop Sn. Then is - [IIT - 2004]
Sn 1
begins to fall. Locate the position of second drop
2n  1 2n  1
from the floor when the first drop strikes the floor. (A) (B)
2n  1 2n  1
[JEE MAIN– 2021]
(A) 2.45 m (B) 7.35 m 2n 2n
(C) (D)
(C) 2.94 m (D) 4.18 m 2n  1 ( 2n  1)

45. A particle is moving with constant acceleration ‘a’. 48. At t = 0, particle is at rest. Then the maximum
Following graph shows v2 versus x(displacement) speed achieved by particle is - [IIT - 2004]
plot. The acceleration of the particle is _____ m/s2.
10
80 C
acceleration
v2 (m/s2)

60 B (m/s2)
40 A 11
time(s)
20
(A) 110 m/s (B) 650 m/s

0 10 20 30 (C) 55 m/s (D) 550 m/s


x(m)

115
MOTION I N ONE DIMEN SIO N PHYSICS-X I-IIT-JEE

49. If graph of velocity Vs distance is as shown

V0 x
(A) (B)
a a
x
V

x x0 (C) a (D) a
The graph of acceleration Vs distance is x
represented by [IIT - 2005] x

116
PHYSICS-X I-IIT-JEE MOTION I N ONE DIMEN SIO N

ANSWER KEY
EXERCISE-1

Qus. 1 2 3 4 5 6 7 8 9 10 11 12 13 14 15 16 17 18 19 20
Ans. A B C A A A A A C A D C B A D A D D A A
Qus. 21 22 23 24 25 26 27 28 29 30 31 32 33 34 35 36 37 38 39 40
Ans. A B A B A B B C C D A C C C B A B A A A
Qus. 41 42 43 44 45 46 47 48 49 50 51 52 53 54 55 56
Ans. C C A C B C A A B C C A A D A B

EXERCISE-2

Qus. 1 2 3 4 5 6 7 8 9 10 11 12 13 14 15
Ans. A,B,C,D B,C,D A,B A,B,C A,D B,C A,C,D A,C,D A,C A,C A,C,D C,D A,D A,D A,B,C,D
Qus. 16 17 18 19 20 21 22 23 24 25 26 27 28 29 30
Ans. B A,B A,D B,C A,B B,C A,C A,D A,B,C A,B,C,D B,D A,B,C A,C A,B,D A
Qus. 31 32 33 34 35 36 37 38 39 40 41 42 43 44 45
Ans. C D C B D B C B D A A D B C A
Qus. 46 47 48 49 50 51 52 53 54 55 56 57 58 59
Ans. B C C D A C A B A A D D B C

EXERCISE-3

1. (A)(r),(s); (B)(r),(s); (C)(p); (D)(q) 2. (A)(r); (B)(p); (C)(q); (D)(q)

3. (A)(r), (B)(q), (C)(s), (D)(p) 4. (A) (s); (B)(r); (C)(q); (D)(p)

5. (A)(r); (B)(p); (C)(r); (D)(s) 6. (A)  (q) ; (B)  (p) ; (C)  (s) ; (D)  (s)

7. (A)(q); (B)(p),(q); (C)(p),(r);(D)(r),(s) 8. (A)  (q); (B)  (p); (C)  (s) ; (D)  (r)

9. (A)  (p), (q) ; (B)  (r) ; (C)  (s), (D)  (s) 10. (A)(q); (B)(r); (C)(q), (r) ; (D)  (q), (r)

11.(A)  (q), (s) ; (B).  (p), (r) ; (C)  (p), (r) ; (D)  (q), (s)

12.(A)(p) ; (B)  (r), (s) ; (C)  (q) ; (D)  (q) 13. (A)  (p) ; (B)  (r) ; (C)  (q) ; (D)  (s)

14. (2) 15. (1) 16. (2) 17. (9) 18. (2) 19. (5) 20. (6)

21. (1) 22. (8) 23. (5) 24. (6) 25. (7) 26. (2) 27. (4)

28. (5) 29. (1) 30. (1) 31. (2) 32. (8) 33. (5) 34. (1)

35. (3) 36. (5) 37. (8) 38. (14) 39. (4) 40. (2)

117
MOTION I N ONE DIMEN SIO N PHYSICS-X I-IIT-JEE

EXERCISE-4
1. (B) 2. (A) 3. (D) 4. (B) 5. (A) 6. (C) 7. (D)
8. (D) 9. (C) 10. (D) 11. (C) 12. (A) 13. (B) 14. (C)
15. (B) 16. (B) 17. (C) 18. (D) 19. (D) 20. (B) 21. (B)
22. (D) 23. (C) 24. (D) 25. (C) 26. (D) 27. (A) 28. (D)
29. (A) 30. (D) 31. (A) 32. (B) 33. (B) 34. (C) 35. (C)
36. (B) 37. (40) 38. (4) 39. (C) 40. (C) 41. (C) 42. (50)
43. (12) 44. (B) 45. (1) 46. (A) 47. (B) 48. (C) 49. (B)

118

You might also like